Search This Blog

468x60.

728x90

Guyton and Hall Physiology 02

 



































































































































































D) An increased end-systolic volume

E) A shift of the pressure-volume relationship to the left

157. Which of the following is correct about treatment of shock?

A) Dextran solution (if plasma is not available) is helpful as

plasma substitution in cardiogenic shock since it increases

interstitial pressure.

B) Anti-histamine agents in anaphylactic shock help to prevent

development of disseminated intra-vascular coagulation

C) Dextran solution (if whole blood or plasma are not available)

is helpful as plasma substitution since it increases intravascular hydrostatic pressure.

D) Dextran solution (if whole blood or plasma are not available)

is helpful as plasma substitution since it increases intravascular colloid osmotic pressure.

E) Glucocorticoids stimulate release of enzymes from lysosomes

that are crucial for tissue protection and to prevent cellular

deterioration

Answers

1. B) The capillary filtration coefficient is calculated as rate of net fluid

movement across a capillary wall divided by net filtration pressure. Net

filtration pressure = capillary hydrostatic pressure − plasma colloid

osmotic pressure + interstitial colloid osmotic pressure − interstitial

hydrostatic pressure. The rate of net fluid movement across the capillary

wall is 150 ml/min.

Capillary filtration coefficient (Kf

) = Filtration rate/Net filtration pressure

Kf = 150 ml/min / [Pc − Πc + Πi − PI

]

Kf = 150 ml/min / [25 − 25 + 10 − (−5)]

Kf = 150 /15 = 10 ml/min/mm Hg

TMP14 p. 197

2. A) Moving from a supine to a standing position causes an acute fall in

arterial pressure that is sensed by arterial baroreceptors located in the

carotid bifurcation and aortic arch. Activation of the arterial

baroreceptors leads to an increase in sympathetic outflow to the heart

and peripheral vasculature and a decrease in parasympathetic outflow

to the heart. The increase in sympathetic activity to peripheral vessels

results in an increase in total peripheral resistance. The increase in

sympathetic activity to the heart results in an increase in heart rate and

strength of contraction. The decrease in parasympathetic outflow to the

heart also contributes to the increase in heart rate.

TMP14 pp. 222–223

3. E) Administration of a drug that decreases the diameter of arterioles in a

muscle bed increases the vascular resistance. The increased vascular

resistance decreases vascular conductance and blood flow. The

reduction in arteriolar diameter also leads to a decrease in capillary

hydrostatic pressure and capillary filtration rate.

TMP14 pp. 174, 177, 197

4. G) Moving from a supine to a standing position causes an acute fall in

arterial pressure that is sensed by arterial baroreceptors located in the

carotid sinuses and aortic arch. Activation of the baroreceptors results in

a decrease in parasympathetic activity (or vagal tone) and an increase in

sympathetic activity, which leads to an increase in plasma renin activity

(or renin release).

TMP14 pp. 217–223

5. A) The difference between systolic pressure and diastolic pressure is the

pulse pressure. The two major factors that affect pulse pressure are the

stroke volume output of the heart and the compliance of the arterial

tree. In patients with moderate aortic regurgitation (due to incomplete

closure of aortic valve), the blood that is pumped into the aorta

immediately flows back into the left ventricle. The backflow of blood

into the left ventricle increases stroke volume and systolic pressure.The

rapid backflow of blood also results in a decrease in diastolic pressure.

Thus, patients with moderate aortic regurgitation have high systolic

pressure, low diastolic pressure, and high pulse pressure.

TMP14 pp. 184–185

6. F) The increase in local metabolism during exercise increases carbon

dioxide production and decreases tissue oxygen concentration and

tissue pH. The decrease in tissue oxygen concentration, tissue pH, and

increase in carbon dioxide production increase arteriolar diameter

(decrease arteriole resistance) and increase vascular conductance and

blood flow to skeletal muscles.

TMP14 pp. 206–208

7. B) Cognitive stimuli increase cerebral blood flow by decreasing cerebral

vascular resistance. The diameter of cerebral vessels is increased by

various metabolic factors in response to cognitive stimuli. Metabolic

factors that enhance cerebral blood flow include increases in carbon

dioxide, hydrogen ion (decreased pH), and adenosine.

TMP14 pp. 205–207

8. A) Histamine is a vasodilator that is typically released by mast cells and

basophils. Infusion of histamine into a brachial artery would decrease

arteriolar resistance and increase water permeability of the capillary

wall. The decrease in arteriolar resistance would also increase capillary

hydrostatic pressure. The increase in capillary hydrostatic pressure and

water permeability leads to an increase in capillary filtration rate,

interstitial volume, and interstitial hydrostatic pressure.

TMP14 pp. 174, 197

9. C) An increase in shear stress in blood vessels is one of the major stimuli

for the release of nitric oxide by endothelial cells. Nitric oxide increases

blood flow by increasing cyclic guanosine monophosphate.

TMP14 p. 210

10. C) Angiotensin I is formed by an enzyme (renin) acting on a substrate

called angiotensinogen. Angiotensin I is converted to angiotensin II by a

converting enzyme. Angiotensin II also has a negative feedback effect on

juxtaglomerular cells to inhibit renin secretion. Angiotensin II is a

powerful vasoconstrictor and sodium-retaining hormone that increases

arterial pressure. Administration of an ACE inhibitor would increase

plasma renin concentration, decrease angiotensin II formation, and

decrease total peripheral resistance and arterial pressure.

TMP14 pp. 236–238

11. E) A decrease in the diameter of a precapillary arteriole would increase

arteriolar resistance. The increase in arteriolar resistance would lead to a

decrease in vascular conductance, capillary blood flow, hydrostatic

pressure, and filtration rate.

TMP14 pp. 174, 197

12. A) Stenosis of one kidney results in the release of renin and the

formation of angiotensin II from the affected kidney. Angiotensin II

stimulates aldosterone production and increases total peripheral

resistance by constricting most of the blood vessels in the body.

TMP14 p. 239

13. D) Blood flow in a vessel is directly proportional to the fourth power of

the vessel radius. Increasing vessel diameter (or radius) by 100% (2 ×

control) would increase blood flow 2 to the fourth power × normal blood

flow (100 ml/min). Thus, blood flow would increase to 100 ml/min × 16,

or approximately 1600 ml/min.

TMP14 p. 177

14. A) In patent ductus arteriosus, a large quantity of the blood pumped

into the aorta by the left ventricle immediately flows backward into the

pulmonary artery and then into the lung and left atrium. The shunting

of blood from the aorta results in a low diastolic pressure, while the

increased inflow of blood into the left atrium and ventricle increases

stroke volume and systolic pressure. The combined increase in systolic

pressure and decrease in diastolic pressure results in an increase in

pulse pressure.

TMP14 pp. 184–185

15. E) A decrease in tissue oxygen tension is thought to be an important

stimulus for vascular endothelial growth factor and the growth of blood

vessels in solid tumors.

TMP14 pp. 211–213

16. A) The net movement of sodium across a capillary wall is directly

proportional to the wall permeability to sodium, wall surface area, and

concentration gradient across the capillary wall. Thus, increases in

permeability to sodium, surface area, and sodium concentration

gradient wall would all increase the net movement of sodium across the

capillary wall.

TMP14 pp. 195–196

17. D) Constriction of the carotid artery decreases blood pressure at the

level of the carotid sinus. A decrease in carotid sinus pressure leads to a

decrease in carotid sinus nerve impulses to the vasomotor center, which

in turn leads to enhanced sympathetic nervous activity and decreased

parasympathetic nerve activity. The increase in sympathetic nerve

activity results in peripheral vasoconstriction and an increase in total

peripheral resistance and heart rate. The decreased parasympathetic

nerve activity to the heart would also contribute to the increase in heart

rate.

TMP14 pp. 221–223

18. E) Pulse pressure is the difference between systolic pressure and

diastolic pressure. The two major factors that affect pulse pressure are

the stroke volume output of the heart and the compliance of the arterial

tree. An increase in stroke volume increases systolic and pulse pressure,

whereas an increase in compliance of the arterial tree decreases pulse

pressure. Moderate aortic valve stenosis results in a decrease in stroke

volume, which leads to a decrease in systolic pressure and pulse

pressure.

TMP14 pp. 184–185

19. B) A person with atherosclerosis would be expected to have decreased

arterial compliance. The decrease in arterial compliance would lead to

an increase in systolic pressure and pulse pressure.

TMP14 pp. 184–185

20. B) Constriction of the carotid artery reduces blood pressure at the

carotid bifurcation where the arterial baroreceptors are located. The

decrease in arterial pressure activates baroreceptors, which in turn leads

to an increase in sympathetic activity and a decrease in parasympathetic

activity (or vagal tone). The enhanced sympathetic activity results in

constriction of peripheral blood vessels, including the kidneys. The

enhanced sympathetic activity leads to an increase in total peripheral

resistance and a decrease in renal blood flow. The combination of

enhanced sympathetic activity and decreased vagal tone also leads to an

increase in heart rate.

TMP14 pp. 221–223

21. B) Filtration rate is the product of the filtration coefficient (Kf

) and the

net pressure across the capillary wall. The net pressure for fluid

movement across a capillary wall is promoted by increases in capillary

hydrostatic pressure and positive interstitial colloid osmotic pressure,

whereas negative plasma colloid osmotic pressure and a positive

interstitial hydrostatic pressure oppose filtration. Thus, increased

capillary hydrostatic pressure, decreased plasma colloid osmotic pressure,

and increased interstitial colloid osmotic pressure would all promote

filtration. Increased arteriolar resistance would decrease filtration by

decreasing capillary hydrostatic pressure. The filtration coefficient is the

product of capillary surface area and the capillary water permeability. A

decrease in capillary water permeability would decrease the filtration

coefficient and reduce the filtration rate.

TMP14 pp. 196–197

22. E) Solid tumors are metabolically active tissues that need increased

quantities of oxygen and other nutrients. When metabolism in a tissue is

increased for a prolonged period, the vascularity of the tissue also

increases. One of the important factors that increases growth of new

blood vessels is VEGF. Presumably, a deficiency of tissue oxygen or

other nutrients, or both, leads to the formation of VEGF.

TMP14 p. 212

23. A) A decrease in the diameter of a precapillary arteriole increases

arteriolar resistance while decreasing vascular conductance and

capillary blood flow, hydrostatic pressure, filtration rate, interstitial

volume, and interstitial hydrostatic pressure.

TMP14 pp. 174, 197

24. C) Excess secretion of aldosterone results in enhanced tubular

reabsorption of sodium and secretion of potassium. The increased

reabsorption of sodium and water leads to an increase in extracellular

fluid volume, which in turn suppresses renin release by the kidney. The

increase in potassium secretion leads to a decrease in plasma potassium

concentration, or hypokalemia.

TMP14 pp. 237–238

25. B) The two main factors that increase lymph flow are an increase in

capillary filtration rate and an increase in lymphatic pump activity. A

decrease in plasma colloid osmotic pressure increases capillary filtration

rate, interstitial volume and hydrostatic pressure, and lymph flow. In

contrast, an increase in hydraulic conductivity of the capillary wall and

capillary hydrostatic pressure increase capillary filtration rate,

interstitial volume and pressure, and lymph flow. An increase in

arteriole resistance would decrease vascular conductance, capillary

hydrostatic pressure, capillary filtration rate, interstitial volume and

pressure, and lymph flow.

TMP14 pp. 195–203

26. E) According to Poiseuille’s law, flow through a vessel increases in

proportion to the fourth power of the radius. A 4-fold increase in vessel

diameter (or radius) would increase 4 to the fourth power, or 256 times

normal. Thus, flow through the vessel after increasing the vessel 4 times

normal would increase from 100 to 25,600 ml/min.

TMP14 p. 177

27. B) Vascular resistance is equal to arterial pressure divided by blood

flow. In this example, arterial pressure is 125 mm Hg, venous pressure is

5 mm Hg, and blood flow is 1200 ml/min ( plasma flow/hematocrit).

Thus, vascular resistance is equal to 120 divided by (600/.50 or 1200), or

0.10 mm Hg/ml/min.

TMP14 p. 174

28. D) The rate of blood flow is directly proportional to the fourth power of

the vessel radius and to the pressure gradient across the vessel. In

contrast, the rate of blood flow is inversely proportional to the viscosity

of the blood. Thus, an increase in blood viscosity would decrease blood

flow in a vessel.

TMP14 pp. 177–178

29. D) The flow in a vessel is directly proportional to the pressure gradient

across the vessel and to the fourth power of the radius of the vessel. In

contrast, blood flow is inversely proportional to the viscosity of the

blood. Because blood flow is proportional to the fourth power of the

vessel radius, the vessel with the largest radius (vessel D) would have

the greatest flow.

TMP14 p. 177

30. B) The arterial baroreceptors are activated in response to a fall in

arterial pressure. During hemorrhage, the fall in arterial pressure at the

level of the baroreceptors results in enhanced sympathetic outflow from

the vasomotor center and a decrease in parasympathetic nerve activity.

The increase in sympathetic nerve activity leads to constriction of

peripheral blood vessels, increased total peripheral resistance, and

plasma renin activity, angiotensin II and a return of blood pressure

toward normal. The decrease in parasympathetic nerve activity and

sympathetic outflow would result in an increase in heart rate.

TMP14 pp. 221–223, 236–238

31. A) Activation of the baroreceptors leads to an increase in sympathetic

activity, which in turn increases heart rate, strength of cardiac

contraction, and constriction of arterioles and veins. The increase in

venous constriction results in an increase in mean circulatory filling

pressure, venous return, and cardiac output.

TMP14 pp. 221–223

32. E) The conversion of angiotensin I to angiotensin II is catalyzed by a

converting enzyme that is present in the endothelium of the lung vessels

and in the kidneys. The converting enzyme also serves as a kininase that

degrades bradykinin. Thus, a converting enzyme inhibitor not only

decreases the formation of angiotensin II but also inhibits kininases and

the breakdown of bradykinin. Angiotensin II is a vasoconstrictor and a

powerful sodium-retaining hormone. While plasma bradykinin

increases, the major cause for the decrease in arterial pressure in

response to an ACE inhibitor is the decrease in formation of angiotensin

II.

TMP14 pp. 236–238

33. G) When blood pressure falls below 80 mm Hg, carotid and aortic

chemoreceptors are activated to elicit a neural reflex to minimize the fall

in blood pressure. The chemoreceptors are chemosensitive cells that are

sensitive to oxygen lack, carbon dioxide excess, or hydrogen ion excess

(or fall in pH). The signals transmied from the chemoreceptors into the

vasomotor center excite the vasomotor center to increase arterial

pressure.

TMP14 p. 224

34. D) Although sympathetic nerves, angiotensin II, and vasopressin are

powerful vasoconstrictors, blood flow to skeletal muscles under normal

physiological conditions is mainly determined by local metabolic factors

such as tissue adenosine, oxygen, hydrogen, and carbon dioxide

concentrations. Capillary osmotic pressure plays a role in determining

fluid movement across a capillary not blood flow.

TMP14 pp. 206–208

35. E) During exercise, tissue levels of carbon dioxide and lactic acid

increase. These metabolites dilate blood vessels, decrease arteriolar

resistance, and enhance vascular conductance and blood flow.

TMP14 pp. 208–209

36. C) The velocity of blood flow within each segment of the circulatory

system is inversely proportional to the total cross-sectional area of the

segment. Because the capillaries has the highest total cross-sectional area

of all circulatory segments, it has the lowest velocity of blood flow.

TMP14 p. 172

37. B) Filtration rate is the product of the filtration coefficient (Kf

) and the

net pressure across the capillary wall. The net pressure for fluid

movement across a capillary wall = capillary hydrostatic pressure −

plasma colloid osmotic pressure + interstitial colloid osmotic pressure −

interstitial hydrostatic pressure. The net pressure in this question

calculates to be 10 mm Hg, and the Kf

 is 15. Thus, the filtration rate is 15

× 10, or 150 ml/min.

TMP14 p. 197

38. A) Resistance of a vessel = pressure gradient ÷ blood flow of the vessel.

In this example, vessel A has the highest vascular resistance (100 mm

Hg/1000 ml/min, or 0.1 mm Hg/ml/min).

TMP14 pp. 173–174

39. C) The transport of oxygen across a capillary wall is proportional to the

capillary surface area, capillary wall permeability to oxygen, and oxygen

gradient across the capillary wall. Thus, a 2-fold increase in the oxygen

concentration gradient would result in the greatest increase in the

transport of oxygen across the capillary wall. A 2-fold increase in

intercellular clefts in the capillary wall would not have a significant

impact on oxygen transport because oxygen can permeate the

endothelial cell wall.

TMP14 pp. 195–196

40. D) Atrial natriuretic peptide is released from myocytes in the atria in

response to increases in atrial pressure. ANP in turn inhibits the

production of angiotensin II and aldosterone. Stretch of the atria also

results in a nervous reflex to inhibit renal sympathetic nerve activity.

TMP14 p. 224

41. C) The capillaries have the largest total cross-sectional area of all

vessels of the circulatory system. The venules also have a relatively large

total cross-sectional area but not as great as the capillaries, which

explains the large storage of blood in the venous system compared with

that in the arterial system.

TMP14 p. 172

42. E) An increase in atrial pressure increases plasma levels of atrial

natriuretic peptide, which in turn would decrease plasma levels of

angiotensin II and aldosterone and increase sodium excretion.

TMP14 p. 224

43. C) An increase in perfusion pressure to a tissue results in excessive

delivery of nutrients such as oxygen to a tissue. The increase in tissue

oxygen concentration constricts arterioles and returns blood flow and

nutrient delivery toward normal levels.

TMP14 pp. 209–210

44. C) The percentage of total blood volume in the veins is approximately

64%.

TMP14 p. 172

45. B) Constriction of the renal artery increases release of renin, formation

of angiotensin II and aldosterone, and arterial pressure. A 50%

reduction in renal artery pressure would be below the range of renal

autoregulation and would result in a decrease in the glomerular

filtration rate.

TMP14 pp. 239–240

46. B) An increase in plasma colloid osmotic pressure would reduce net

filtration pressure and capillary filtration rate. Increases in capillary

hydrostatic pressure and interstitial colloid osmotic pressure would also

favor capillary filtration. An increase in venous hydrostatic pressure and

arteriolar diameter would tend to increase capillary hydrostatic pressure

and capillary filtration rate.

TMP14 pp. 194–201

47. E) A decrease in sodium intake would result in a decrease in sodium

excretion to maintain sodium balance. Angiotensin II and aldosterone

would increase in response to a chronic decrease in sodium intake,

whereas plasma atrial natriuretic peptide levels would decrease.

TMP14 pp. 236–238

48. E) The rate of lymph flow increases in proportion to the interstitial

hydrostatic pressure and the lymphatic pump activity. An increase in

capillary hydrostatic pressure would increase filtration rate, interstitial

volume, interstitial hydrostatic pressure, and lymph flow. An increase in

plasma colloid osmotic pressure would decrease filtration rate,

interstitial volume, interstitial hydrostatic pressure, and lymph flow. A

decrease in interstitial volume would increase interstitial hydrostatic

pressure and lymph flow. A decrease in arteriolar diameter would

decrease capillary hydrostatic pressure, capillary filtration, interstitial

volume, interstitial hydrostatic pressure and lymph flow.

TMP14 pp. 197–204

49. E) Nitric oxide and prostacyclin are potent vasodilator and natriuretic

substances. Thus, increases in nitric oxide and prostacyclin production

would result in a decrease in arterial pressure. In contrast, angiotensin II

and aldosterone are anti-natriuretic and pro-hypertensive factors. An

increase in the production of these factors would increase arterial

pressure.

TMP14 p. 244

50. B) The liver endothelium contains many open pores, or fenestrae that

allow extremely large molecules such as albumin to pass in or out of the

liver tissues. Under normal conditions, very lile if any albumin cross

the capillary walls of muscle, glomerular, brain, or intestine.

TMP14 p. 195

51. D) The Cushing reaction is a special type of CNS ischemic response

that results from increased pressure of the cerebrospinal fluid around

the brain in the cranial vault. When the cerebrospinal fluid pressure

rises, it decreases the blood supply to the brain and elicits a CNS

ischemic response. The CNS ischemic response includes enhanced

sympathetic activity, decreased parasympathetic activity, and increased

heart rate, arterial pressure, and total peripheral resistance.

TMP14 p. 226

52. B) The factors that determine the net movement of glucose across a

capillary wall include the wall permeability to glucose, the glucose

concentration gradient across the wall, and the capillary wall surface

area. Thus, an increase in the concentration difference of glucose across

the wall would enhance the net movement of glucose.

TMP14 pp. 195–196

53. E) An increase in atrial pressure of 10 mm Hg would tend to decrease

venous return to the heart and increase vena cava hydrostatic pressure.

Plasma colloid osmotic pressure, interstitial colloid osmotic pressure,

arterial pressure, and cardiac output would generally be low to normal

in this patient.

TMP14 pp. 197–200

54. A) An increase in renal arterial pressure results in increases in sodium

and water excretion. Normally, glomerular filtration rate would be

normal or slightly increased in response to an increase in renal artery

pressure. However, in the absence of an intact tubuloglomerular

feedback system, an important renal autoregulatory mechanism, an

increase in renal artery pressure would result in significant increases in

glomerular filtration rate.

TMP14 p. 210

55. C) The vascular compliance is proportional to the vascular

distensibility and the vascular volume of any given segment of the

circulation. The compliance of a systemic vein is 24 times that of its

corresponding artery because it is about 8 times as distensible and has a

volume about 3 times as great.

TMP14 p. 183

56. E) The difference between systolic pressure and diastolic pressure is

called the pulse pressure. The two main factors that affect pulse

pressure are stroke volume and arterial compliance. Pulse pressure is

directly proportional to the stroke volume and inversely proportional to

the arterial compliance. Thus, increases in systolic pressure or stroke

volume would tend to increase pulse pressure.

TMP14 pp. 184–185

57. B) Moving from a supine to a standing position results in pooling of

blood in the lower extremities and a fall in blood pressure. The pooling

of blood in the legs increases venous hydrostatic pressure. The fall in

arterial pressure activates the arterial baroreceptors, which in turn

increases sympathetic nerve activity and decreases parasympathetic

nerve activity. The increase in sympathetic activity constricts renal

vessels and reduces renal blood flow. The heart rate also increases.

TMP14 pp. 221–223

58. B) Moving from a supine to a standing position would tend to cause

pooling of blood in the lower extremities and a reduction in arterial

pressure. Decreased arterial pressure activates arterial the baroreceptor

reflex which then leads to increased sympathetic nervous system

activity, decreased parasympathetic activity, increased heart rate, and

increased cardiac contractility.

TMP14 pp. 221–223

59. B) An increase in arterial pressure to a tissue leads to an increase in

tissue oxygen concentration and a decrease in tissue carbon dioxide

concentration. Both events lead to a decrease in arteriolar diameter,

increased vascular resistance, and decreased vascular conductance.

TMP14 pp. 209–210

60. B) Because oxygen is lipid soluble and can cross the capillary wall with

ease, it has the fastest rate of movement across the capillary wall. The

ability of lipid-insoluble substances such as sodium, albumin, and

glucose to move across a capillary wall depends on the permeability of

the capillary to lipid-insoluble substances. Because the capillary wall is

relatively impermeable to albumin, it has the slowest rate of net

movement across the capillary wall.

TMP14 p. 195

61. A) An increase in capillary wall permeability to water would increase

capillary filtration rate, whereas increases in arteriolar resistance,

plasma colloid osmotic pressure, and interstitial hydrostatic pressure

would all decrease filtration rate. Plasma sodium concentration would

have no effect on filtration.

TMP14 pp. 197–1201

62. D) The tendency for turbulent flow occurs at vascular sites where the

velocity of blood flow is high. The aorta has the highest velocity of blood

flow.

TMP14 pp. 175–176

63. E) Total peripheral vascular resistance = (arterial pressure − right atrial

pressure) ÷ cardiac output. In this example, total peripheral vascular

resistance = 130 mm Hg ÷ 3.5 l/min, or approximately 37 mm Hg/l/min.

TMP14 p. 177

64. C) Interstitial hydrostatic pressure in a muscle capillary bed is normally

negative (−3 mm Hg). Pumping by the lymphatic system is the basic

cause of the negative pressure.

TMP14 pp. 202–203

65. D) The two main factors that affect pulse pressure are stroke volume

and arterial compliance. Decreases in stroke volume decrease pulse

pressure, and an increase in arterial compliance decreases pulse

pressure. Hemorrhage and decreased venous return would decrease

stroke volume and pulse pressure. In patients with patent ductus, stroke

volume and pulse pressure are increased as a result of shunting of blood

from the aorta to the pulmonary artery.

TMP14 pp. 184–186

66. D) The primary mechanism whereby solutes move across a capillary

wall is simple diffusion.

TMP14 p. 196

67. E) Movement of the leg muscles causes blood to flow toward the vena

cava, which reduces venous hydrostatic pressure. A decrease in right

atrial pressure would increase venous return and decrease venous

hydrostatic pressure. Pregnancy and the presence of abdominal tumor

in the abdomen would tend to compress the vena cava and increase

venous hydrostatic pressure in the legs.

TMP14 pp. 188–190

68. A) Nitric oxide is a vasodilator that is believed to play a role in

regulating blood flow. Infusion of a nitric oxide donor into the brachial

artery would increase arteriolar diameter and decrease arteriolar

resistance. The decrease in arteriolar resistance would also result in an

increase in capillary hydrostatic pressure and filtration rate. The

increase in filtration rate leads to an increase in interstitial hydrostatic

pressure and lymph flow.

TMP14 pp. 173–174, 197–200, 210

69. D) In persons with decompensated heart failure, the kidneys retain

sodium and water, which causes a weight gain and an increase in blood

volume. This effect increases the mean systemic filling pressure, which

also stretches the heart. Therefore, a decreased mean systemic filling

pressure does not occur in decompensated heart failure. The excess

blood volume often will overstretch the sarcomeres of the heart, which

will prevent them from achieving their maximal tension. An excess

central fluid volume also results in orthopnea, which is the inability to

breathe properly except in the upright position.

TMP14 pp. 273–274

70. A) During progressive hemorrhagic shock, the vasomotor center often

fails, thus reducing sympathetic output. Decreases in arterial pressure

will reduce urine output. Decreased blood flow throughout the body

causes acidosis because of decreased removal of carbon dioxide. In

progressive shock due to hemorrhage, capillary permeability increases

and mean systemic filling pressure decreases.

TMP14 p. 296

71. C) With an overdose of furosemide there is a large loss of sodium and

water from the body, resulting in dehydration and sometimes shock.

The optimal therapy is to replenish the electrolytes that were lost as a

result of the overdose of the furosemide. Therefore, infusion of a

balanced electrolyte solution is the therapy of choice.

TMP14 pp. 301–302

72. C) Severe vomiting can lead to a large loss of sodium and water from

the body, resulting in dehydration and sometimes shock. The best

therapy is to replenish the depleted sodium and water lost by vomiting.

Therefore, infusion of a balanced electrolyte solution is the therapy of

choice.

TMP14 pp. 301–302

73. C) The formula for resistance to venous return is (mean systemic filling

pressure − right atrial pressure)/cardiac output. In this example, the

mean systemic filling pressure is 7 mm Hg, and the right atrial pressure

is 0 mm Hg. The cardiac output is 5 l/min. Using these values in the

previous formula indicates that the resistance to venous return is 1.4

mm Hg/l/min. Note that this formula only applies to the linear portion

of the venous return curve.

TMP14 pp. 251–253

74. A) During increases in sympathetic output to maximal values, several

changes occur. First, the mean systemic filling pressure increases

markedly, but at the same time the resistance to venous return increases.

Venous return is determined by the following formula: (mean systemic

filling pressure − right atrial pressure)/resistance to venous return.

During maximal sympathetic output, the increase in systemic filling

pressure is greater than the increase in resistance to venous return.

Therefore, in this formula, the numerator has a much greater increase

than the denominator, which results in an increase in the venous return.

TMP14 p. 255

75. C) This problem concerns the Fick principle for determining cardiac

output. The formula for cardiac output is oxygen absorbed per minute

by the lungs divided by the arterial-venous oxygen difference. In this

problem, oxygen consumption of the body is 240 ml/min, and in a

steady-state condition, this would exactly equal the oxygen absorbed by

the lungs. Therefore, by inserting these values into the equation, we see

that the cardiac output will equal 12 l/min.

TMP14 p. 256

76. A) A shift to the right in the cardiac output curve involves an increase

in the normal intrapleural pressure of −4 mm Hg. Changing the

intrapleural pressure to −1 mm Hg will shift the curve to the right.

Changing the mean systemic filling pressure does not change the

cardiac output curve. Taking a patient off of a ventilator, decreasing

intrapleural pressure to −7 mm Hg, and breathing against a negative

pressure shifts the cardiac output curve to the left.

TMP14 p. 250

77. C) Several factors can cause the cardiac output to shift to the right or to

the left. Among those are surgically opening the chest, which makes the

cardiac output curve shift 4 mm Hg to the right, and severe cardiac

tamponade, which increases the pressure inside the pericardium, thus

tending to collapse the heart, particularly the atria. Playing a trumpet or

positive pressure breathing tremendously increases the intrapleural

pressure, thus collapsing the atria and shifting the cardiac output curve

to the right. Breathing against a negative pressure shifts the cardiac

output curve to the left.

TMP14 p. 250

78. E) The plateau level of the cardiac output curve, which is one measure

of cardiac contractility, decreases in several circumstances. Some of

these circumstances include severe cardiac tamponade, which increases

the pressure in the pericardial space, and increasing parasympathetic

stimulation of the heart. Increased sympathetic stimulation of the heart

increases the level of the cardiac output curve by increasing heart rate

and contractility.

TMP14 p. 247

79. B) Cardiac output increases in several conditions because of increased

venous return. A-V fistulae also cause a decreased resistance to venous

return, thus increasing cardiac output. Cardiac output decreases in

patients with hypovolemia, severe aortic regurgitation, and

polycythemia. The hematocrit level is high in polycythemia, which

increases resistance to venous return.

TMP14 pp. 248–250

80. D) Mean systemic filling pressure is a measure of the tightness of fit of

the blood in the circulation. Mean systemic filling pressure is increased

by factors that increase blood volume and decrease the vascular

compliance. Therefore, decreased venous compliance, not increased

compliance, would cause an increase in mean systemic filling pressure.

Norepinephrine administration and sympathetic stimulation cause

arteriolar vasoconstriction and decreased vascular compliance, resulting

in an increase in mean systemic filling pressure. Increased blood volume

and skeletal muscle contraction, which cause a contraction of the

vasculature, also increase this filling pressure.

TMP14 pp. 252–253

81. C) Resistance to venous return (which is equal to cardiac output) is

equal to the mean systemic filling pressure minus the right atrial

pressure divided by venous return. Thus, resistance to venous return is

equal to 12 minus 2 divided by 10 or 10/10 or 1 mm Hg/l per min.

TMP14 pp. 253–254

82. A) Anemia decreases resistance to venous return because of arteriolar

dilation. The following mechanisms increase resistance to venous

return: increased venous resistance, increased arteriolar resistance,

increased sympathetic output, and obstruction of veins.

TMP14 pp. 253–254

83. D) Venous return (or cardiac output) is equal to the mean systemic

filling pressure minus the right atrial pressure divided by resistance to

venous return. Thus, an increase in resistance to venous return

decreases venous return and cardiac output.

TMP14 p. 253

84. E) Cardiac output decreases in several conditions because of decreased

venous return. Cardiac output decreases in hypothyroidism because of

the decreased oxygen use by the peripheral tissues, resulting in

arteriolar vasoconstriction and thus decreased venous return.

Conversely, cardiac output increases with hyperthyroidism. Beriberi

causes increased cardiac output because a lack of the vitamin thiamine

results in peripheral vasodilation. A-V fistulae also cause decreased

resistance to venous return, thus increasing cardiac output. Increased

muscle mass is associated with increased tissue metabolism, decreased

arteriolar resistance and resistance to venous return, and increased

venous return to the heart and thus an increase in cardiac output.

TMP14 pp. 248–249

85. G) In response to increases in cardiac workload, there are increases in

cardiac tissue metabolism and cardiac tissue adenosine concentration.

Adenosine reduces arteriole resistance and increases vascular

conductance.

TMP14 p. 263

86. C) The plateau level of the cardiac output curve, which is one measure

of cardiac contractility, decreases in several circumstances. Some of

these include myocarditis, severe cardiac tamponade that increases the

pressure in the pericardial space, myocardial infarction, and various

valvular diseases such as mitral stenosis. Decreased parasympathetic

stimulation of the heart actually moderately increases the level of the

cardiac output curve by increasing the heart rate.

TMP14 p. 247

87. A) During increases in sympathetic output, the main two organs that

maintain their blood flow are the brain and the heart. During exercise

for 1 hour, the intestinal flow decreases significantly, as do the renal and

pancreatic blood flows. The skeletal muscle blood flow to non-exercising

muscles also decreases at this time. Cerebral blood flow remains close to

its control value.

TMP14 pp. 259–262

88. E) According to the Fick principle,

Cardiac output = Oxygen absorbed by lungs(ml/min) (400) divided by

Arterio-venous oxygen difference (200 – 150 ml/l)

Cardiac output = 400/50 or 8 l/min

TMP14 pp. 256–257

89. A) Although bradykinin, prostaglandins, carbon dioxide, and

potassium ions serve as vasodilators for the coronary artery system, the

major controller of coronary blood flow is adenosine. Adenosine is

formed as adenosine triphosphate degrades to adenosine

monophosphate. Small portions of the adenosine monophosphate are

then further degraded to release adenosine into the tissue fluids of the

heart muscle, and this adenosine vasodilates the coronary arteries.

TMP14 p. 263

90. E) Sympathetic stimulation directly increases the strength of cardiac

contraction and increases the heart rate. In this way, the plateau of the

Starling curve elevates. Surgically opening the chest and undergoing

mechanical ventilation shifts the cardiac output curve to the right.

Cardiac tamponade rotates the curve downward, and parasympathetic

stimulation depresses the curve.

TMP14 pp. 247–248

91. E) Ischemia causes the cardiac muscle to release lactic acid, which

stimulates pain nerve endings in cardiac muscle, sending impulses

through sensory afferent nerve fibers into the central nervous system.

TMP14 pp. 263–264, 268

92. D) Several factors cause arteriolar vasodilation during exercise,

including increases in potassium ion concentration, plasma nitric oxide

concentration, plasma adenosine concentration, and plasma osmolality.

Although histamine causes arteriolar vasodilation, histamine release

does not normally occur during exercise.

TMP14 pp. 259–260

93. A) At the beginning of exercise, increases in sympathetic stimulation of

the heart strengthens the heart and increases the heart rate. Coronary

and cerebral blood flow are spared from any decrease. Reverse stress

relaxation does not occur. Venous constriction, not dilation, occurs.

TMP14 p. 261

94. D) When a sudden occlusion occurs in one of the larger coronary

arteries, cardiac tissue adenosine concentration increases as a result of

ATP breakdown. The small anastomoses in cardiac tissue begin to dilate

immediately. Then over the next several days, collateral blood flow

increases to partially restore blood flow to the ischemic tissue.

TMP14 p. 265

95. E) Several drugs have proven to be helpful to patients with myocardial

ischemia. Beta receptor blockers (not stimulators) inhibit the

sympathetic effects on the heart and are very helpful. ACE inhibition

prevents the production of angiotensin II and thus decreases the

afterload effect on the heart. Nitroglycerin causes nitric oxide release,

resulting in coronary vasodilation. Isometric exercise increases blood

pressure markedly and can be harmful, and increased dietary calcium

would be of lile benefit.

TMP14 pp. 268–269

96. E) During exercise, the sympathetic output increases markedly, which

causes arteriolar constriction in many places of the body, including nonexercising muscle. The increased sympathetic output also causes

venoconstriction throughout the body. During exercise, there also is an

increased release of norepinephrine and epinephrine by the adrenal

glands. These changes all help maintain blood pressure during exercise.

Arterioles in the exercising muscle dilate to increase blood flow to meet

the metabolic needs of the tissue.

TMP14 pp. 260–261

97. D) When increased quantities of blood flows into the heart, the

resulting stretch in the heart wall results in an increase in force of

contraction (Frank-Starling law of the heart) and stroke volume. Stretch

of the sinus node in the wall of the right atrium has a direct effect on the

rhythmicity of the node to increase the heart rate. Stretch of right atrium

also initiates a nervous reflex called the Bainbridge reflex that increases

heart rate.

TMP14 p. 245

98. C) Increased sympathetic stimulation excites the cardiac myocytes and

makes them much more susceptible to fibrillation. High (not low)

potassium increases fibrillation tendency. An increase (not a decrease) in

ventricular diameter allows the cardiac muscle to be out of the

refractory period when the cardiac impulse next arrives and can

increase the tendency to fibrillate. A low adenosine level probably only

causes some coronary constriction. Decreased parasympathetics allow

the heart rate to increase and has lile to do with fibrillation.

TMP14 p. 268

99. A) In a patient with angina due to myocardial ischemia, oxygen use by

the heart must be minimized. Oxygen use can be minimized with ACE

inhibition, which decreases angiotensin II formation. This reduces the

arterial pressure and decreases myocardial tension and oxygen use. The

use of beta sympathetic blockers (not stimulation) inhibits the effects of

excess sympathetic output on the heart, thus reducing wall tension and

oxygen use. Isometric exercise should be avoided because of the large

increase in arterial pressure that occurs. Chelation therapy with EDTA

and increased dietary calcium have lile to do with cardiac function.

TMP14 pp. 268–269

100. C) The major causes of death after myocardial infarction include a

decrease in cardiac output that prevents tissues of the body from

receiving adequate nutrition and oxygen delivery and prevents removal

of waste materials. Other causes of death are pulmonary edema, which

reduces the oxygenation of the blood, fibrillation of the heart, and

rupture of the heart. Cardiac contractility decreases after a myocardial

infarction.

TMP14 p. 266

101. E) During sympathetic stimulation, venous reservoirs constrict,

venous vascular resistance also increases, arterioles constrict (which

increases their resistance), and the heart rate increases. The epicardial

coronary vessels have a large number of alpha receptors, but the

subendocardial vessels have more beta receptors. Therefore,

sympathetic stimulation causes at least a slight constriction of the

epicardial vessels. This results in a slight decrease in epicardial flow.

TMP14 pp. 262–264

102. E) Several factors change during compensated heart failure to stabilize

the circulatory system. Because of increased sympathetic output, the

heart rate increases during compensated heart failure. The kidneys

retain sodium and water, which increases blood volume and thus right

atrial pressure. The increased blood volume that results causes an

increase in mean systemic filling pressure, which will help to increase

the cardiac output. Dyspnea usually will occur only in the early stages

of compensated failure.

TMP14 pp. 272–273

103. E) In unilateral left heart failure, the kidneys retain sodium and water

and thus increase blood volume, and the pulmonary veins, in turn,

become congested. Therefore, mean pulmonary filling pressure,

pulmonary wedge pressure, and left atrial pressure increase. In contrast,

in right heart failure, right atrial pressure increases, and edema of the

lower extremities, including the feet and ankles, occurs.

TMP14 pp. 275–276

104. A) In compensated heart failure, an increased release of angiotensin II

also occurs, which causes direct renal sodium retention and also

stimulates aldosterone secretion that will, in turn, causes further

increases in sodium retention by the kidneys. Because of the low arterial

pressure that occurs in compensated heart failure, the sympathetic

output increases. One of the results is a sympathetic vasoconstriction

(not vasodilation) of the afferent arterioles of the kidney. This decreases

the glomerular hydrostatic pressure and the glomerular filtration rate,

resulting in an increase in sodium and water retention in the body. The

excess sodium in the body increases osmolality, which increases the

release of antidiuretic hormone, which causes renal water retention (but

not sodium retention).

TMP14 p. 276

105. C) During acute pulmonary edema, the increased fluid in the lungs

diminishes the oxygen content in the blood. This decreased oxygen

weakens the heart even further and causes arteriolar dilation in the

body. This results in increases in venous return of blood to the heart,

which cause further leakage of the fluid in the lungs and further

decreases in oxygen content in the blood. It is important to interrupt this

vicious circle to save a patient’s life. This can be interrupted by placing

tourniquets on all four limbs, which effectively removes blood volume

from the chest. The patient can also breathe oxygen, and a

bronchodilator can be administered. Furosemide can be administered to

reduce some of the fluid volume in the body and especially in the lungs.

One thing you do not want to do is infuse whole blood or an electrolyte

solution in this patient because it may exacerbate the pulmonary edema

that is already present.

TMP14 p. 277

106. D) Cardiogenic shock results from a weakening of the cardiac muscle

many times after coronary thrombosis, which can result in a vicious

circle because of low cardiac output resulting in a low diastolic pressure.

This causes a decrease in coronary flow, which decreases the cardiac

strength even more. Therefore, arterial pressure, particularly diastolic

pressure, must be increased in patients with cardiogenic shock with

either vasoconstrictors or volume expanders. In this patient, the best

answer is to infuse plasma. Placing tourniquets on all four limbs

decreases the central blood volume, which would worsen the condition

of the patient in shock.

TMP14 p. 275

107. B) This patient has a resting cardiac output of 4 l/min, and his cardiac

reserve is 300% of this resting cardiac output or 12 l/min. This gives a

total maximum cardiac output of 16 l/min. Therefore, the cardiac reserve

is the percentage increase that the cardiac output can be elevated over

the resting cardiac output.

TMP14 pp. 277–278

108. B) Several factors cause sodium retention during heart failure,

including aldosterone release, decreased glomerular filtration rate, and

increased angiotensin II release. A decrease in mean arterial pressure

also results in decreases in glomerular hydrostatic pressure and causes a

decrease in renal sodium excretion. During heart failure, blood volume

increases, resulting in an increased cardiac stretch. In particular, the

atrial pressure increases, causing a release of atrial natriuretic factor,

resulting in an increase in renal sodium excretion.

TMP14 pp. 276–277

109. D) There is a vicious circle of cardiac deterioration in cardiogenic

shock. A weakened heart causes a decreased cardiac output, which

decreases arterial pressure. The decreased arterial pressure, particularly

the decrease in diastolic pressure, decreases the coronary blood flow

and further weakens the heart and thus further decreases cardiac

output. The therapy of choice for a patient in cardiogenic shock is to

increase the arterial pressure either with a vasoconstrictor drug or with

a volume-expanding drug. Placing tourniquets on the four limbs,

withdrawing a moderate amount of blood, or administering furosemide

decreases the thoracic blood volume and thus worsens the condition of

the patient in cardiogenic shock.

TMP14 p. 275

110. A) In unilateral right heart failure, the right atrial pressure increases,

and the overall cardiac output decreases, which results in a decrease in

arterial pressure and urinary output. However, left atrial pressure does

not increase but in fact decreases.

TMP14 p. 275

111. B) During compensated heart failure, many factors combine to

increase cardiac output so it returns to normal. The kidneys decrease

their urinary output of sodium and water to increase the blood volume.

This action, when combined with a depressed cardiac output curve,

increases right atrial pressure. Mean systemic filling pressure increases

(not decreases), and the venous return of blood back toward the heart

thus increases right atrial pressure. Heart rate is normal, and sweating

and dyspnea are absent in the chronic stages of compensated failure.

TMP14 pp. 272–273, 278–279

112. A) Reduction of fluid in the lungs can prevent rapid deterioration in

patients with acute pulmonary edema. Furosemide causes venodilation,

which reduces thoracic blood volume and acts as a powerful diuretic.

These both reduce excess fluid in the lungs. Blood can actually be

removed in moderate quantities from the patient to decrease the volume

of blood in the chest. Patients should also breathe oxygen to increase the

oxygen levels in the blood. However, they should never be given a

volume expander, such as saline, plasma, whole blood, or dextran,

because it could worsen the pulmonary edema. Norepinephrine would

be of lile help in treating pulmonary edema.

TMP14 p. 277

113. B) In compensated heart failure, mean systemic filling pressure

increases because of hypervolemia, and cardiac output is often at

normal values. The patient has air hunger, called dyspnea, and excess

sweating occurs in the early phases of compensated heart failure.

However, right atrial pressure becomes elevated to very high values in

these patients and is a hallmark of this disease.

TMP14 pp. 272–273, 278–279

114. B) Mean systemic pressure is increased by factors that increase blood

volume or decrease vascular capacity. Sympathetic inhibition and

venous dilation both decrease the mean systemic filling pressure. In

congestive heart failure, the kidneys retain great quantities of sodium

and water, resulting in an increase in blood volume, which causes large

increases in mean systemic filling pressure.

TMP14 pp. 272–274

115. A) During compensated heart failure, release of angiotensin II and

aldosterone is increased, causing the kidneys retain sodium and water,

which increases the blood volume in the body and the venous return of

blood to the heart. This situation results in an increase in right atrial

pressure. Increased sympathetic output during compensated heart

failure increases heart rate. Air hunger, called dyspnea, occurs during

any type of exertion. The patient also has orthopnea, which is the air

hunger that occurs from being in a recumbent position.

TMP14 pp. 271–273

116. B) During decompensated heart failure, cardiac output decreases

because of weakness of the heart and edema of the cardiac muscle.

Pressures in the pulmonary capillary system increase, including the

pulmonary capillary pressure and the mean pulmonary filling pressure.

Depletion of norepinephrine in the endings of the cardiac sympathetic

nerves is another factor that causes weakness of the heart.

TMP14 pp. 273–274, 279–280

117. D) In decompensated heart failure, the kidneys retain sodium and

water, which causes weight gain and an increase in blood volume. This

situation increases the mean systemic filling pressure, which also

stretches the heart. Therefore, decreased mean systemic filling pressure

does not occur in decompensated heart failure. The excess blood volume

often overstretches the sarcomeres of the heart, which prevents them

from achieving their maximal tension. An excess central fluid volume

also results in orthopnea, which is the inability to breathe properly

except in the upright position.

TMP14 pp. 273–274, 279–280

118. C) The mean electrical axis of the QRS of this patient is shifted

rightward to 170 degrees, which indicates that the right side of the heart

is involved. Both aortic stenosis and mitral regurgitation cause a

leftward shift of the QRS axis. Mitral stenosis does not affect the left

ventricle, but in severe enough circumstances, it could cause an increase

in pulmonary artery pressure, which would cause an increase in

pulmonary capillary pressure at the same time. Tricuspid stenosis does

not affect the right ventricle. Therefore, pulmonary valve stenosis is the

only condition that fits this set of symptoms.

TMP14 pp. 286–287, 290

119. A) The fourth heart sound occurs at the end of diastole and is caused

by inrushing of blood into the ventricles due to atrial contraction. The

first heart sound is caused by closing of the A-V valves. The closing of

the aortic and pulmonary valves at the end of systole causes the second

heart sound. This initiates a vibration throughout the ventricles, aorta,

and pulmonary artery. The third heart sound is caused by inrushing of

blood into the ventricles in the early to middle part of diastole.

TMP14 p. 284

120. B) Blowing murmurs of relatively high pitch are usually murmurs

associated with valvular insufficiency. The key pieces of data to identify

this murmur are the systolic and diastolic pressures. Aortic valve

regurgitation typically has a high pulse pressure, which is the systolic –

diastolic pressure and in this case is 100 mm Hg. Also notice that the

diastolic pressure decreases to very low values of 40 mm Hg as the

blood leaks back into the left ventricle.

TMP14 pp. 286–287

121. E) Left ventricular hypertrophy occurs when the left ventricle either

has to produce high pressure or when it pumps extra volume with each

stroke. During aortic regurgitation, extra blood leaks back into the

ventricle during the diastolic period. This extra volume must be

expelled during the next heartbeat. During mitral regurgitation, some

blood gets pumped out into the aorta, while at the same time blood

leaks back into the left atrium. Therefore, the left ventricle is pumping

extra volume with each heartbeat. During aortic stenosis, the left

ventricle must contract very strongly, producing high wall tension to

increase the aortic pressure to the values high enough to expel blood

into the aorta. During mitral stenosis, the ventricle is normal because the

atrium produces the extra pressure to get blood through the stenotic

mitral valve.

TMP14 pp. 286–287

122. E) Several diastolic murmurs can be heard easily with a stethoscope.

During diastole, aortic and pulmonary valve regurgitation occur

through the insufficient valves causing the heart murmur at this time.

Tricuspid and mitral stenosis are diastolic murmurs because blood flows

through the restricted valves during the diastolic period. Patent ductus

arteriosus is heard in both systole and diastole.

TMP14. pp. 289–290

123. C) Aortic stenosis has a very high ventricular systolic pressure.

Diastolic filling of the ventricle requires a much higher left atrial

pressure. However, tricuspid stenosis and regurgitation, pulmonary

valve regurgitation, and pulmonary stenosis are associated with an

increase in right atrial pressure and should not affect pressure in the left

atrium.

TMP14 pp. 286–287

124. B) This patient has a QRS axis of −45 degrees, indicating a leftward

axis shift. In other words, the left side of the heart is enlarged. In aortic

valve stenosis, the left side of the heart is enlarged because of the extra

tension the left ventricular walls must exert to expel blood out the aorta.

Therefore, these symptoms fit with a patient with aortic stenosis. In

pulmonary valve stenosis, the right side of the heart hypertrophies, and

in mitral valve stenosis, there is no left ventricular hypertrophy. In

tricuspid valve regurgitation, the right side of the heart enlarges, and in

tricuspid valve stenosis, no ventricular hypertrophy occurs.

TMP14 pp. 286–287

125. C) This patient has a heart murmur heard maximally in the

“pulmonary area of cardiac auscultation.” The high pitch indicates

regurgitation. The rightward axis shift indicates that the right side of the

heart has hypertrophied. The two choices that have a rightward axis

shift are pulmonary valve regurgitation and tetralogy of Fallot. In

tetralogy of Fallot, the arterial blood oxygen content is low, which is not

the case with this patient. Therefore, pulmonary valve regurgitation is

the correct answer.

TMP14 pp. 283–288

126. A) Right ventricular hypertrophy occurs when the right heart has to

pump a higher volume of blood or pump it against a higher pressure.

Tetralogy of Fallot is associated with right ventricular hypertrophy

because of the increased pulmonary valvular resistance, and this also

occurs during pulmonary artery stenosis. Tricuspid insufficiency causes

an increased stroke volume by the right heart, which causes

hypertrophy. However, tricuspid stenosis does not affect the right

ventricle.

TMP14 pp. 290–291

127. E) Mitral stenosis is heard during diastole only. Aortic stenosis,

tricuspid valve regurgitation, interventricular septal effect, and patent

ductus arteriosus are clearly heard during systole. However, patent

ductus arteriosus is also heard during diastole.

TMP14 p. 286

128. A) In tetralogy of Fallot, there is an interventricular septal defect as

well as stenosis of either the pulmo nary artery or the pulmonary valve.

Therefore, it is very difficult for blood to pass into the pulmonary artery

and into the lungs to be oxygenated. Instead the blood partially shunts

to the left side of the heart, thus bypassing the lungs. This situation

results in low arterial oxygen content.

TMP14 pp. 290–291

129. B) The first heart sound by definition is always associated with the

closing of the A-V valves. The heart sounds are usually not associated

with opening of any of the valves but with the closing of the valves and

the associated vibration of the blood and the walls of the heart. One

exception is an opening snap in some mitral valves.

TMP14 p. 283

130. B) In tetralogy of Fallot, an interventricular septal defect and increased

resistance in the pulmonary valve or pulmonary artery cause partial

blood shunting toward the left side of the heart without going through

the lungs. This situation results in a severely decreased arterial oxygen

content. The interventricular septal defect causes equal systolic

pressures in both cardiac ventricles, which causes right ventricular

hypertrophy and a wall thickness very similar to that of the left

ventricle.

TMP14 p. 290

131. C) Mitral regurgitation and aortic stenosis are murmurs heard during

the systolic period. A ventricular septal defect murmur is normally

heard only during the systolic phase. Tricuspid valve stenosis and

patent ductus arteriosus murmurs are heard during diastole. However,

a patent ductus arteriosus murmur is also heard during systole.

TMP14 pp. 286, 289–290

132. E) The third heart sound is associated with inrushing of blood into the

ventricles in the early to middle part of diastole. The next heart sound,

the fourth heart sound, is caused by inrushing of blood in the ventricles

caused by atrial contraction. The first heart sound is caused by the

closing of the A-V valves, and the second heart sound is caused by the

closing of the pulmonary and aortic valves.

TMP14 pp. 283–284

133. A) A number of things occur in progressive shock, including increased

capillary permeability, which allows fluid to leak out of the vasculature,

thus decreasing the blood volume. Other deteriorating factors include

vasomotor center failure, peripheral circulatory failure, decreased

cellular mitochondrial activity, and acidosis throughout the body.

Usually, urine output strikingly decreases; therefore, the increased

urinary output answer is incorrect. Tissue pH decreases, and reverse

stress relaxation of the veins occurs.

TMP14 pp. 296–298

134. A) Sympathomimetic drugs are given to counteract hypotension

during a number of conditions. These conditions include spinal cord

injury in which the sympathetic output is interrupted.

Sympathomimetic drugs are also given during very deep anesthesia,

which decreases the sympathetic output, and during anaphylactic shock

that results from histamine release and the accompanying

vasodilatation. Sympathomimetic drugs, such as norepinephrine,

increase blood pressure by causing a vasoconstriction. Shock caused by

excess vomiting, hemorrhage, or excessive administration of diuretics

results in fluid volume depletion, resulting in decreased blood volume

and decreased mean systemic filling pressure. Administering a balanced

electrolyte solution best counteracts this condition.

TMP14 pp. 299–301

135. D) Too deep a level of anesthesia can decrease sympathetic tone and

reduce arterial pressure enough to induce shock. To replace the

sympathetic tone that was lost, the optimal therapy is infusion of a

sympathomimetic drug. Infusion of red blood cells, plasma, or

electrolytes would be of lile benefit.

TMP14 pp. 299–301

136. D) The patient received an influenza inoculation and quickly went

into shock, which leads one to believe that he may be in anaphylactic

shock. Anaphylactic shock is a state of extreme vasodilation because of

histamine release. Antihistamines would be somewhat helpful, but they

are very slow acting, and the patient could die in the meantime.

Therefore, a very rapid-acting agent must be used, such as a

sympathomimetic drug.

TMP14 pp. 299–301

137. E) In compensated hemorrhagic shock, a number of factors prevent

the progression of the shock, including increased heart rate. Also

occurring is reverse stress relaxation in which the vasculature,

particularly the veins, constrict around the available blood volume.

Increased ADH release also occurs, which causes water retention from

the kidney but also vasoconstriction of the arterioles. A CNS ischemic

response also occurs if the blood pressure drops to very low values,

causing an increase in sympathetic output. Increased absorption of

interstitial fluid through the capillaries also occurs, which increases the

volume in the vasculature.

TMP14 p. 295

138. E) Spinal anesthesia, especially when the anesthesia extends all the

way up the spinal cord, can block the sympathetic nervous outflow from

the spinal cord. This can be a very potent cause of neurogenic shock.

The therapy of choice is to replace the sympathetic tone that was lost in

the body. The best way to in crease the sympathetic tone is by infusing a

sympathomimetic drug.

TMP14 pp. 299–301

139. A) This patient has obviously lost a lot of blood because of the

motorcycle wreck. The most advantageous therapy is to replace what

was lost in the accident. This would be whole blood, which is much

superior to a plasma infusion, because the patient is also receiving red

blood cells that have a much superior oxygen-carrying capacity than the

plasma component of blood. Sympathetic nerves are firing very rapidly

in this condition, and an infusion of a sympathomimetic agent would be

of lile advantage.

TMP14 p. 301

140. C) In hemorrhagic shock, anaphylactic shock, and neurogenic shock,

the venous return of blood to the heart markedly decreases. However, in

septic shock, the cardiac output increases in many patients because of

vasodilation in affected tissues and a high metabolic rate causing

vasodilation in other parts of the body.

TMP14 p. 300

141. E) This patient has been hemorrhaging, and the optimal therapy is to

replace the blood he has lost. Unfortunately, no blood is available, and

therefore we must choose next best therapy, which is increasing the

volume of his blood. Thus, plasma infusion is the next best therapy

because its high colloid osmotic pressure will help the infused fluid stay

in the circulation much longer than would a balanced electrolyte

solution.

TMP14 p. 301

142. B) Intestinal obstruction often causes severe reduction in plasma

volume. Obstruction causes a distention of the intestine and partially

blocks the venous blood flow in the intestines. This partial blockage

results in an increased intestinal capillary pressure, which causes fluid

to leak from the capillary into the walls of the intestines and into the

intestinal lumen. The leaking fluid has a high protein content very

similar to that of the plasma, which reduces the total plasma protein and

the plasma volume. Therefore, the therapy of choice would be to replace

the fluid lost by infusing plasma.

TMP14 pp. 299, 301

143. A) In progressive shock, because of the poor blood flow, the pH in the

tissues throughout the body decreases. Many vessels become blocked

because of local blood agglutination, which is called “sludged blood.”

Patchy areas of necrosis also occur in the liver. Mitochondrial activity

decreases and capillary permeability increases. There is also an

increased release of hydrolases by the lysosomes and a decrease in

cellular metabolism of glucose.

TMP14 pp. 296–298

144. A) Anaphylaxis is an allergic condition that results from an antigenantibody reaction that takes place after exposure of an individual to an

antigenic substance. The basophils and mast cells in the pericapillary

tissues release histamine or histamine-like substances. The histamine

causes venous dilation, dilation of arterioles, and greatly increased

capillary permeability with rapid loss of fluid and protein into the tissue

spaces. This response reduces venous return and often results in

anaphylactic shock.

TMP14 pp. 300–301

145. B) This patient has a resting cardiac output of 4.8 l/min, and a cardiac

output of 19.2 l/min after maximum exercise. The cardiac reserve [(19.2 –

4.8)/4.8 =3.0)] is the percentage increase that the cardiac output can be

elevated over the resting cardiac output. Therefore, her cardiac reserve is

300% (3-fold) greater than her resting cardiac output.

TMP14 pp. 277–278

146. C) Immediately after heart damage, a compensatory activation of the

sympathetic nervous system develops, which helps aenuate the

decreased cardiac output (heart muscle is damaged) and improve

cardiovascular dynamics (e.g. mean systemic filling pressures, venous

return). Damming of blood develops in the failing heart due to

depressed contraction. Parasympathetics are not activated as a

compensatory mechanism. The renin-angiotensin system activates later.

TMP14 pp. 271–273

147. B) Cardiac reserve is always reduced in heart failure, even when

compensatory mechanisms and/or treatments have been effective in

restoring cardiovascular hemodynamics at rest. Cardiac failure is the

failure of the heart to pump enough blood to satisfy the needs of the body,

which may develop with low or high cardiac output. A low cardiac

output will result in low blood flow into the kidneys, reduced

glomerular filtration, and a low urinary output.

TMP14 pp. 271–273, 277–278

148. C) The pumping ability of the heart is compromised in low-output

heart failure. Moderate fluid retention will improve venous return and

“prime” the partially damaged heart in order to increase cardiac output.

Based on the Frank-Starling law of the heart (the student should be

familiar with this mechanism), this will result in an increased preload of

the heart. Afterload and isovolumetric contraction are unchanged.

Aortic pressure would be preserved. Peripheral edema may develop if

fluid retention progresses.

TMP14 p. 272

149. A) The severe damage of the heart and the failure to increase CO

enough to satisfy the needs of the body will lead to a progressive fluid

retention (constant low flow to the kidneys will continuously activate

mechanisms of fluid retention), increased mean filling pressures, and

increased right atrial pressure. The progressive fluid retention in an

already weak heart will lead to a deleterious cycle that will progresses

until the heart is so overstretched and/or edematous that it fails

completely.

TMP14 pp. 273–274, 279–280

150. C) Mitral regurgitation produces a systolic murmur. The history of

dilated cardiomyopathy suggests mitral regurgitation. The lack of

cyanosis and age of the patients eliminates tetralogy of Fallot as a likely

diagnosis. The characteristics of the murmur eliminates patent ductus

arteriosus (machinery murmur), mitral and tricuspid stenosis (diastolic

murmurs).

TMP14 pp. 286, 288

151. D) The diastolic murmur of mitral stenosis is due to the narrow

opening of the mitral valve, making it difficult for the blood to move

from the left atria to the left ventricle. A consequence of mitral stenosis is

an increase in pressure of pulmonary vessels, but this is not a cause of

the murmur.

TMP14 pp. 286, 288

152. C) Abnormalities of the aortic valve (outflow tract of the left ventricle)

associates with significant increases in LV pressure and as such,

increases in afterload (which does not occur in mitral

stenosis/regurgitation). Preload is unchanged in aortic stenosis.

Isovolumetric systolic period is lost in aortic and in mitral regurgitation.

Tricuspid stenosis does not associate with loss of left ventricular

isovolumetric diastolic period.

TMP14 pp. 286–287

153. B) A right-to-left shunt as observed in tetralogy of Fallot results in

almost 2/3 of the blood passing from the right ventricle into aorta

without oxygenation, bypassing the lungs (right-to-left shunt). As a

result, oxygen in the blood is low and the infant develops cyanosis (blue

baby) and rapidly shows a significant enlargement of the right ventricle.

Patent ductus arteriosus (a left-to-right shunt) will not show cyanosis at

this stage and the blood is usually hyper-oxygenated. An interatrial

septal defect associates with a left-to-right shunt. Tricuspid stenosis

does not cause low oxygen in blood or associate with enlargement of the

right ventricle.

TMP14 pp. 289–290

154. A) Without intervention (i.e. volume replacement) or after entering

into the progressive phase, cardiac function progressively declines,

which constitutes the most important factor for the progression towards

irreversibility of shock. Autoregulation cannot prevent (or reverse) the

irreversible phase of shock.

TMP14 pp. 295–297

155. C) Based on the information included that states a massive heart

aack, a significant delay to initiate treatment, and the lack of responses

to treatments, the patient is most likely at the irreversible stage of shock

from cardiogenic origin (not hemorrhagic). More fluids will not change

the course of shock at this stage. Diminished delivery of oxygen will

lead to tissue acidosis, and tissue ischemia facilitates release of toxins

that further contribute to the generalized cellular deterioration.

TMP14 pp. 296–299

156. C) The patient is in ventricular fibrillation, thus in cardiac/circulatory

arrest. There is no effective mechanical activity in the left ventricle.

TMP14 pp. 301–302

157. D) Dextran is a large polysaccharide that do not pass through the

capillary pores and serves as a colloid osmotic agent (as do plasma

proteins). Anti-histamine agents target histamine, which is massively

released in anaphylactic shock. Glucocorticoids stabilize lysosomes and

help to prevent their disruption and, consequently, the release of

enzymes to the cellular cytoplasm.

TMP14 p. 301


Unit V: The Body Fluids and

Kidneys

1. All of the following changes would tend to cause interstitial fluid

edema in a tissue EXCEPT one. Which one is the EXCEPTION?

A) Increased venous resistance

B) Increased venous pressure

C) Decreased arteriolar resistance

D) Increased capillary filtration coefficient

E) Increased plasma protein concentration

2. Calculate the approximate extracellular fluid osmolarity of a patient

after administration of 2.0 l of 5% glucose solution, assuming

complete metabolism of the glucose, osmotic equilibrium and no

excretion of water or electrolytes. Also assume the following initial

conditions prior to infusing the glucose solution:

Body weight = 50 kg

Plasma sodium concentration = 170 mmol/l

Plasma osmolarity = 360 mOsm/l

Intracellular fluid volume = 40% of body weight

Extracellular fluid volume = 20% of body weight

Molecular wt of glucose = 180 g/mole

A) 264 mOsm/l

B) 282 mOsm/l

C) 306 mOsm/l

D) 319 mOsm/l

E) 338 mOsm/l

F) 355 mOsm/l

G) 360 mOsm/l

3. Calculate the approximate extracellular fluid volume of a patient after

administration of 3.0 l of 5% glucose solution, assuming complete

metabolism of the glucose, osmotic equilibrium, and no excretion of

water or electrolytes. Also assume the following initial conditions

prior to infusing the glucose solution:

Body weight = 50 kg

Plasma sodium concentration = 170 mmol/l

Plasma osmolarity = 360 mOsm/l

Intracellular fluid volume = 40% of body weight

Extracellular fluid volume = 20% of body weight

Molecular wt of glucose = 180 g/mole

A) 8.0 l

B) 10.7 l

C) 11.7 l

D) 130 l

E) 20.3 l

F) 21.3 l

G) 30.0 l

4. A patient with cirrhosis experiences a doubling of his serum

creatinine over a 6-month period after sustained heavy ingestion of a

nonsteroidal antiinflammatory drug (NSAID) for his arthritis. Which

of the following is the best explanation for his increased serum

creatinine?

A) Increased efferent arteriolar resistance which reduced

glomerular filtration rate (GFR)

B) Decreased Bowman’s capsule pressure which reduced GFR

C) Increased afferent arteriolar resistance which reduced GFR

D) Increased glomerular capillary filtration coefficient which

reduced GFR

E) Increased renal prostaglandins due to the NSAID

F) Increased nitric oxide formation due to the NSAID

5. Administration of empagliflozin, an inhibitor of sodium-glucose cotransporter 2 (SGLT2), would be expected to cause which of the

following sets of changes compared with normal?

GFR Resistance Afferent Arteriole Renal Blood Flow

A)

B)

C)

D)

E)

F)

6. Given the following measurements, calculate the approximate

filtration fraction:

Glomerular capillary hydrostatic pressure = 60 mm Hg

Colloid osmotic pressure in the glomerular capillaries = 30 mm

Hg

Bowman’s space hydrostatic pressure = 20 mm Hg

Glomerular capillary filtration coefficient (Kf) = 10 ml/min/mm

Hg

Renal plasma flow = 600 ml/min

Hematocrit = 0.4

A) 10 mm Hg

B) 100 ml/min

C) 0.100

D) 0.167

E) 0.200

F) 0.333

7. Which of the following statements is incorrect?

A) Creatinine concentration in the urine is normally higher than in

the glomerular filtrate.

B) Urea concentration in the urine is normally higher than in the

glomerular filtrate.

C) The proximal tubules normally reabsorb almost all of the

glucose filtered by the glomerular capillaries.

D) concentration in the urine is normally higher than in the

glomerular filtrate.

E) Organic acids and bases are secreted mainly by the proximal

tubules.

F) Sodium concentration remains relatively constant as tubular

fluid flows along the proximal tubule.

8. A patient with diabetes mellitus has a glomerular filtration rate of 100

ml/min, a urine flow rate of 4.0 ml/min, and a urine glucose

concentration of 2 mg/ml. If he has a kidney transport maximum for

glucose of 200 mg/min, what would be his approximate rate of

glucose excretion?

A) 0 mg/min

B) 8 mg/min

C) 100 mg/min

D) 180 mg/min

E) 300 mg/min

F) Urinary excretion rate of glucose cannot be determined from

these data

9. As tubular fluid passes along a juxtamedullary nephron of a person

with severe central diabetes insipidus and essentially no antidiuretic

hormone, where is the osmolarity lowest?

A) Bowman’s capsule (glomerular filtrate)

B) Fluid leaving the proximal tubule and entering the loop of Henle

C) Fluid leaving the descending thin limb and entering the

ascending thin limb of the loop of Henle

D) Fluid leaving the thick ascending segment of the loop of Henle

and entering the early distal tubule

E) Fluid in the cortical collecting tubules

F) Fluid leaving the collecting ducts (urine)

10. If GFR = 60 ml/min, urine flow rate = 2.0 ml/min, plasma K+

concentration = 4.0 mmol/l, and urine K+

 concentration = 80 mmol/l,

what is the approximate rate of K+

 excretion?

A) 0.08 mmol/min

B) 0.16 mmol/min

C) 0.32 mmol/min

D) 16 mmol/min

E) 160 mmol/min

F) Excretion rate of K+

 cannot be determined from these data

11. If the glomerular filtration rate (GFR) of a patient is reduced to 50% of

normal and sustained at that level, you would expect to find

__________ renal creatinine excretion rate, __________ renal

creatinine clearance, and __________ serum creatinine concentration

6 weeks after the decrease in GFR compared with normal. Assume

steady-state conditions and that the patient has maintained the same

diet.

A) Decreased, decreased, increased

B) Decreased, no change, increased

C) No change, increased, increased

D) No change, no change, increased

E) No change, decreased, increased

F) Decreased, no change, decreased

12. If glomerular filtration rate suddenly decreases by 50%, from 80

ml/min to 40 ml/min and tubular fluid reabsorption simultaneously

decreases from 78 ml/min to 40 ml/min, which of the following

changes in urinary excretion rate will occur (assuming that the

changes in GFR and tubular fluid reabsorption are maintained)?

A) Urine flow rate will decrease to zero

B) Urine flow rate will not change

C) Urine flow rate will decrease by 50%

D) Urine flow rate will increase by 50%

13. Calculate the approximate total renal plasma flow given the

following data:

Urine PAH concentration = 200 µg/ml

Urine flow rate = 2 ml/min

Arterial plasma paraaminohippuric acid (PAH) concentration =

1.0 µg/ml

Renal venous PAH concentration = 0.2 µg/ml

Hematocrit = 0.4

A) 120 ml/min

B) 200 ml/min

C) 400 ml/min

D) 500 ml/min

E) 667 ml/min

F) 833 ml/min

14. Which of the following occurs in type A intercalated cells of the

collecting tubules?

A) Secretion of H+

, reabsorption of , and reabsorption of K+

B) Secretion of H+

, reabsorption of , and secretion of K+

C) Secretion of K+

, reabsorption of Na+

, and reabsorption of

D) Reabsorption of H+

, secretion of , and secretion of K+

E) Reabsorption of H+

, secretion of , and reabsorption of K+

15. Which of the following statements is incorrect?

A) Beta-adrenergic stimulation would tend to cause hypokalemia

by shifting potassium from the extracellular fluid into the cells

B) A powerful diuretic that inhibits proximal tubule or loop of

Henle sodium reabsorption would tend to increase potassium

secretion by the collecting tubules

C) Injection of excess insulin into a patient would tend to cause

hypokalemia

D) Strenuous, sustained exercise may tend to cause significant

hyperkalemia

E) Increased extracellular fluid osmolarity would tend to cause

hypokalemia

16. Which of the following statements is incorrect?

A) Urea transporters UT-A1 and UTA-3 in the collecting ducts are

activated by antidiuretic hormone (ADH)

B) Urea reabsorption in the inner medullary collecting duct is

greater than in the distal tubule during dehydration

C) Increased ADH markedly increases urea reabsorption by the

cortical collecting tubule

D) The inner medullary collecting tubule reabsorbs more urea

during antidiuresis than the thick ascending limb of Henle’s loop

E) The cortical collecting tubule is less permeable to urea than is

the inner medullary collecting duct during antidiuresis

F) Passive diffusion of urea into the thin loops of Henle is

facilitated by the urea transporter UT-A2

17. In a dehydrated person with normal kidneys and high ADH levels,

which part of the nephron normally reabsorbs the smallest amount of

water (see figure below of a renal tubule)?

A) A

B) B

C) C

D) D

E) E

F) F

18. Acute metabolic alkalosis tends to ______K+

 secretion by the cortical

collecting tubules and ______ plasma K+

 concentration.

A) decrease, decrease

B) decrease, increase

C) increase, increase

D) increase, decrease

E) cause no change in, increase

F) cause no change in, cause no change in

19. Which of the following statements is incorrect?

A) Carbonic anhydrase inhibitors tend to cause metabolic acidosis.

B) Thiazide diuretics inhibit the Na-Cl co-transporter in the distal

tubules.

C) Osmotic diuretics tend to increase potassium secretion.

D) Aldosterone antagonists (e.g., spironolactone) tend to cause

hypokalemia.

E) Sodium channel blockers (e.g., amiloride) inhibit sodium

transport across the luminal membrane of the collecting tubules.

F) Loop diuretics (e.g., furosemide) tend to cause hypokalemia.

20. Glomerular filtration rate (creatinine clearance) in a patient with

uncontrolled type 2 diabetes has decreased from 100 ml/min to 50

ml/min over the past 4 years. She has poorly controlled hypertension,

and her plasma pH is 7.16. Which of the following changes,

compared with before she developed renal disease, would you expect

to find, assuming steady-state conditions and no changes in protein

or electrolyte intake?

Sodium

Excretion

Rate

Creati-nine

Excretion Rate

Plasma Creati-nine

Concen-tration

Plasma Bicarbonate

Concen-tration

Ammo-nium

Excre-tion Rate

A)

B)

C)

D)

E)

F)

21. A patient has the following laboratory values:

Arterial pH = 7.18

Plasma

Plasma chloride concentration = 100 mEq/l

Arterial = 28 mm Hg

Plasma Na+

 concentration = 141 mEq/l

What is the most likely cause of his acid-base disturbance?

A) Emphysema

B) Renal tubular acidosis

C) Severe diarrhea

D) Methanol poisoning

E) Ingestion of excess sodium bicarbonate

22. Atrial natriuretic peptide causes which of the following effects?

A) Reduced renal tubular sodium reabsorption

B) Reduced renin secretion

C) Increased renal sodium excretion

D) Only A and C

E) A, B, and C

23. If creatinine clearance = 100 ml/min, urine flow rate = 1.0 ml/min,

plasma Na+

 concentration = 140 mmol/l, and urine Na+

 concentration

= 80 mmol/l, what is the approximate rate of Na+

 excretion?

A) 0.08 mmol/min

B) 0.16 mmol/min

C) 16 mmol/min

D) 160 mmol/min

E) Excretion rate of Na+

 cannot be calculated from these data

24. If a person maintains a high (150 mmol/day) potassium diet, which

part of the nephron would be expected to reabsorb the most

potassium? Choose the appropriate nephron site in the figure below.

A) A

B) B

C) C

D) D

E) E

F) F

25. Which of the following would tend to cause hypokalemia by shifting

potassium from the extracellular fluid into the intracellular fluid?

A) Metabolic alkalosis

B) Insulin deficiency

C) Aldosterone deficiency

D) Beta-adrenergic receptor blockade

E) Increased extracellular fluid osmolarity

26. What is the theoretical maximum clearance rate possible for a

substance X that is freely filtered, actively secreted by the renal

tubules, and completely cleared from the plasma given the following

data?

Glomerular filtration rate = 100 ml/min

Plasma concentration of a substance X = 2 mg/ml

Urine flow= 5 ml/min

Renal plasma flow = 800 ml/min

A) 5 ml/min

B) 100 ml/min

C) 200 ml/min

D) 500 ml/min

E) 800 ml/min

F) 1000 ml/min

27. Which of the following solutions, when infused intravenously,

would result in an increase in extracellular fluid volume, a decrease

in intracellular fluid volume, and an increase in total body water after

osmotic equilibrium?

A) 1 l of 0.9% sodium chloride (NaCl) solution

B) 1 l of 0.45% NaCl solution

C) 1 l of 3% NaCl solution

D) 1 l of 5% dextrose solution

E) 1 l of pure water

28. Partial obstruction of a major vein draining a tissue would tend to

__________ lymph flow rate, __________ interstitial fluid hydrostatic

pressure, and __________ interstitial fluid protein concentration in

the tissue drained by that vein.

A) increase, increase, increase

B) increase, increase, decrease

C) increase, decrease, decrease

D) decrease, decrease, decrease

E) decrease, increase, increase

F) decrease, increase, decrease

29. A 36-year-old woman reports headaches and frequent urination.

Laboratory values reveal the following information.

Urine specific gravity = 1.003

Urine protein = negative

Plasma sodium (Na+) = 165 mmol/l

Plasma potassium (K+) = 4.4 mmol/l

Plasma creatinine = 1.4 mg/dl

Blood pressure = 88/40 mm Hg

Heart rate = 115 beats/min

What is the most likely cause of her elevated plasma Na+

 concentration?

A) Primary aldosteronism

B) Diabetes mellitus

C) Diabetes insipidus

D) Simple dehydration caused by insufficient water intake and heavy

exercise

E) Barer syndrome

F) Liddle syndrome

30. After receiving a kidney transplant, a patient has severe

hypertension (170/110 mm Hg). A renal arteriogram indicates severe

renal artery stenosis in his single remaining kidney, with a reduction

in glomerular filtration rate (GFR) to 25% of normal. Which of the

following changes, compared with normal, would be expected in this

patient, assuming steady-state conditions?

A) A large increase in plasma sodium concentration

B) A reduction in urinary sodium excretion to 25% of normal

C) A reduction in urinary creatinine excretion to 25% of normal

D) An increase in serum creatinine to about four times normal

E) Normal renal blood flow in the stenotic kidney due to

autoregulation

Questions 31–33

The figure above represents various states of abnormal hydration. In each

diagram, the normal state (orange and lavender) is superimposed on the

abnormal state (dashed lines) to illustrate the shifts in the volume (width of

rectangles) and total osmolarity (height of rectangles) of the extracellular and

intracellular fluid compartments. Use this figure to answer Questions 31–33.

31. Which diagram represents the changes (after osmotic equilibrium) in

extracellular and intracellular fluid volume and osmolarity after the

infusion of 1% dextrose?

A) A

B) B

C) C

D) D

32. Which diagram represents the changes (after osmotic equilibrium) in

extracellular and intracellular fluid volume and osmolarity in a

patient with the syndrome of inappropriate antidiuretic hormone

(ADH; i.e., excessive secretion of ADH)?

A) A

B) B

C) C

D) D

33. Which diagram represents the changes (after osmotic equilibrium) in

extracellular and intracellular fluid volumes and osmolarities after

the infusion of 3% NaCl?

A) A

B) B

C) C

D) D

34. Which of the following tends to decrease potassium secretion by the

cortical collecting tubule?

A) Increased plasma potassium concentration

B) A diuretic that decreases proximal tubule sodium reabsorption

C) A diuretic that inhibits the action of aldosterone (e.g.,

spironolactone)

D) Acute alkalosis

E) High sodium intake

35. Because the usual rate of phosphate filtration exceeds the transport

maximum for phosphate reabsorption, which statement is true?

A) All the phosphate that is filtered is reabsorbed

B) More phosphate is reabsorbed than is filtered

C) Phosphate in the tubules can contribute significantly to titratable

acid in the urine

D) The “threshold” for phosphate is usually not exceeded

E) Parathyroid hormone must be secreted for phosphate

reabsorption to occur

Questions 36 and 37

Use the following clinical laboratory test results to answer Questions 36

and 37.

Urine flow rate = 1 ml/min

Urine inulin concentration = 100 mg/ml

Plasma inulin concentration = 2 mg/ml

Urine urea concentration = 50 mg/ml

Plasma urea concentration = 2.5 mg/ml

36. What is the GFR?

A) 25 ml/min

B) 50 ml/min

C) 100 ml/min

D) 125 ml/min

E) None of the above

37. What is the net urea reabsorption rate?

A) 0 mg/min

B) 25 mg/min

C) 50 mg/min

D) 75 mg/min

E) 100 mg/min

38. In normal kidneys, which of the following is true of the osmolarity of

renal tubular fluid that flows through the early distal tubule in the

region of the macula densa?

A) Usually isotonic compared with plasma

B) Usually hypotonic compared with plasma

C) Usually hypertonic compared with plasma

D) Hypertonic, compared with plasma, in antidiuresis

39. Which of the following changes would be expected in a patient with

diabetes insipidus due to a lack of ADH secretion, assuming free

access to water and normal thirst mechanisms for controlling water

intake?

Plasma Osmolarity

Concentration

Plasma Sodium

Concentration

Plasma Renin

Concentration

Urine

Volume

A) ↔ ↔ ↓ ↑

B) ↔ ↔ ↑ ↑

C) ↑ ↑ ↑ ↑

D) ↑ ↑ ↔ ↔

E) ↓ ↓ ↓ ↔

40. A 26-year-old woman recently decided to adopt a healthier diet and

eat more fruits and vegetables. As a result, her potassium intake

increased from 80 to 160 mmol/day. Which of the following

conditions would you expect to find 2 weeks after she increased her

potassium intake, compared with before the increase?

Potassium Excretion

Rate

Sodium Excretion

Rate

Plasma Aldosterone Concentration

Plasma Potassium

Concentration

A) ↔ ↔ ↑ Large increase (>1 mmol/l)

B) ↔ ↓ ↑ Small increase (<1 mmol/l)

C) ↑ 2× ↔ ↑ Small increase (<1 mmol/l)

D) ↑ 2× ↑ ↓ Large increase (>1 mmol/l)

E) ↑ 2× ↑ ↔ Large increase (>1 mmol/l)

41. When the dietary intake of K+

 increases, body K+

 balance is

maintained by an increase in K+

 excretion primarily by which of the

following?

A) Decreased glomerular filtration of K+

B) Decreased reabsorption of K+

 by the proximal tubule

C) Decreased reabsorption of K+

 by the thick ascending limb of the

loop of Henle

D) Increased K+

 secretion by the late distal and collecting tubules

E) Shift of K+

 into the intracellular compartment

42. Which of the following would cause the greatest decrease in GFR in

a person with otherwise normal kidneys?

A) Decrease in renal arterial pressure from 100 to 80 mm Hg in a

normal kidney

B) 50% increase in glomerular capillary filtration coefficient

C) 50% increase in proximal tubular sodium reabsorption

D) 50% decrease in afferent arteriolar resistance

E) 50% decrease in efferent arteriolar resistance

F) 5 mm Hg decrease in Bowman’s capsule pressure

43. An 8-year-old boy is brought to your office with extreme swelling of

the abdomen. His parents indicate that he had a very sore throat a

“month or so” ago and that he has been “swelling up” since that

time. He appears to be edematous, and when you check his urine,

you find that large amounts of protein are being excreted. Your

diagnosis is nephrotic syndrome subsequent to glomerulonephritis.

Which of the following changes would you expect to find, compared

with normal?

Thoracic Lymph

Flow

Interstitial Fluid Protein

Concen-tration

Interstitial Fluid Hydrostatic

Pressure

Plasma Renin

Concentration

A) ↑ ↓ ↑ ↑

B) ↑ ↓ ↑ ↔

C) ↑ ↓ ↔ ↑

D) ↓ ↑ ↔ ↔

E) ↓ ↓ ↓ ↓

44. A patient with severe hypertension (blood pressure 185/110 mm Hg)

is referred to you. A renal magnetic resonance imaging scan shows a

tumor in the kidney, and laboratory findings include a very high

plasma renin activity of 12 ng angiotensin I/ml/h (normal = 1). The

diagnosis is a renin-secreting tumor. Which of the following changes

would you expect to find in this patient, under steady-state

conditions, compared with normal?

Plasma Aldosterone Concentration

Sodium Excretion

Rate

Plasma Potassium

Concentration

Renal Blood

Flow

A) ↔ ↓ ↓ ↑

B) ↔ ↔ ↓ ↑

C) ↑ ↔ ↓ ↓

D) ↑ ↓ ↔ ↓

E) ↑ ↓ ↓ ↔

45. The clinical laboratory returned the following values for arterial

blood taken from a patient: plasma pH = 7.28, plasma ,

and plasma partial pressure of carbon dioxide ( ) = 70 mm Hg.

What is this patient’s acid-base disorder?

A) Acute respiratory acidosis without renal compensation

B) Respiratory acidosis with partial renal compensation

C) Acute metabolic acidosis without respiratory compensation

D) Metabolic acidosis with partial respiratory compensation

46. The following laboratory values were obtained in a 58-year-old man:

Urine volume = 4320 ml of urine collected during the preceding

24 hours

Plasma creatinine = 3 mg/100 ml

Urine creatinine = 50 mg/100 ml

Plasma potassium = 4.0 mmol/l

Urine potassium = 30 mmol/l

What is his approximate GFR, assuming that he collected all of his urine

in the 24-hour period?

A) 20 ml/min

B) 30 ml/min

C) 40 ml/min

D) 50 ml/min

E) 60 ml/min

F) 80 ml/min

G) 100 ml/min

Questions 47 and 48

A 65-year-old man had a heart aack and experiences cardiopulmonary

arrest while being transported to the emergency department. Use the

following laboratory values obtained from arterial blood to answer

Questions 47 and 48.

Plasma pH = 7.12

Plasma = 60 mm Hg

Plasma concentration = 19 mEq/l

47. Which of the following options best describes his acid-base disorder?

A) Respiratory acidosis with partial renal compensation

B) Metabolic acidosis with partial respiratory compensation

C) Mixed acidosis: combined metabolic and respiratory acidosis

D) Mixed alkalosis: combined respiratory and metabolic alkalosis

48. In this patient, which of the following laboratory results would be

expected, compared with normal?

A) Increased renal excretion of bicarbonate ( )

B) Decreased urinary titratable acid

C) Increased urine pH

D) Increased renal excretion of ammonia (NH4

+)

49. What would cause the greatest degree of hyperkalemia?

A) Increase in potassium intake from 60 to 180 mmol/day in a

person with normal kidneys and a normal aldosterone system

B) Chronic treatment with a diuretic that inhibits the action of

aldosterone

C) Decrease in sodium intake from 200 to 100 mmol/day

D) Chronic treatment with a diuretic that inhibits loop of Henle

Na+-2Cl − -K+

 co-transport

E) Chronic treatment with a diuretic that inhibits sodium

reabsorption in the collecting ducts

50. Which of the following changes would be expected in a patient with

Liddle syndrome (i.e., excessive activity of amiloride-sensitive

sodium channel in the collecting tubule) under steady-state

conditions, assuming that the intake of electrolytes remained

constant?

Plasma Renin Concentration

Blood

Pressure

Sodium Excretion

Concentration

Plasma

AldosteroneConcentration

A) ↔ ↑ ↓ ↔

B) ↑ ↑ ↔ ↑

C) ↑ ↑ ↓ ↓

D) ↓ ↑ ↔ ↓

E) ↓ ↑ ↓ ↓

F) ↓ ↓ ↑ ↑

51. A patient is referred for treatment of hypertension. After testing, you

discover that he has a very high level of plasma aldosterone, and

your diagnosis is Conn’s syndrome. Assuming no change in

electrolyte intake, which of the following changes would you expect

to find, compared with normal?

Plasma

pH

Plasma K+ Concentration

Urine K+

Excretion

Urine Na

+

Excretion

Plasma Renin Concentration

A) ↑ ↓ ↔ ↔ ↓

B) ↓ ↓ ↔ ↔ ↓

C) ↑ ↓ ↑ ↓ ↓

D) ↑ ↑ ↔ ↓ ↑

E) ↑ ↑ ↑ ↑ ↑

52. Which change tends to increase GFR?

A) Increased afferent arteriolar resistance

B) Decreased efferent arteriolar resistance

C) Increased glomerular capillary filtration coefficient

D) Increased Bowman’s capsule hydrostatic pressure

E) Decreased glomerular capillary hydrostatic pressure

53. Which of the following changes, compared with normal, would you

expect to find 3 weeks after a patient ingested a toxin that caused

sustained impairment of proximal tubular NaCl reabsorption?

Assume that there has been no change in diet or ingestion of

electrolytes.

Glomerular Filtration Rate Afferent Arteriolar Resistance Sodium Excretion

A) ↔ ↔ ↑

B) ↔ ↔ ↑

C) ↓ ↑ ↑

D) ↓ ↑ ↔

E) ↑ ↓ ↔

54. A patient has the following laboratory values: arterial pH = 7.13,

plasma , plasma chloride concentration = 118 mEq/l,

arterial = 28 mm Hg, and plasma Na+

 concentration = 141 mEq/l.

What is the most likely cause of his acidosis?

A) Salicylic acid poisoning

B) Diabetes mellitus

C) Diarrhea

D) Emphysema

55. The GFR of a 26-year-old man with glomerulonephritis decreases by

50% and remains at that level for one month. For which substance

would you expect to find the greatest increase in plasma

concentration?

A) Creatinine

B) K+

C) Glucose

D) Na+

E) Phosphate

F) H+

56. Which changes would you expect to find after administering a

vasodilator drug that caused a 50% decrease in afferent arteriolar

resistance and no change in arterial pressure?

A) Decreased renal blood flow, decreased GFR, and decreased

peritubular capillary hydrostatic pressure

B) Decreased renal blood flow, decreased GFR, and increased

peritubular capillary hydrostatic pressure

C) Increased renal blood flow, increased GFR, and increased

peritubular capillary hydrostatic pressure

D) Increased renal blood flow, increased GFR, and no change in

peritubular capillary hydrostatic pressure

E) Increased renal blood flow, increased GFR, and decreased

peritubular capillary hydrostatic pressure

57. If the average hydrostatic pressure in the glomerular capillaries is 50

mm Hg, the hydrostatic pressure in the Bowman’s space is 12 mm

Hg, the average colloid osmotic pressure in the glomerular capillaries

is 30 mm Hg, and there is no protein in the glomerular ultrafiltrate,

what is the net pressure driving glomerular filtration?

A) 8 mm Hg

B) 32 mm Hg

C) 48 mm Hg

D) 60 mm Hg

E) 92 mm Hg

58. In a patient who has chronic, uncontrolled diabetes mellitus, which

set of conditions would you expect to find, compared with normal?

Titratable Acid Excretion NH+ Excretion Excretion Plasma

A) ↔ ↑ ↓ ↔

B) ↓ ↑ ↔ ↓

C) ↑ ↑ ↔ ↑

D) ↑ ↑ ↓ ↓

E) ↓ ↓ ↓ ↓

F) ↔ ↑ ↓ ↔

59. Intravenous infusion of 1 l of 0.45% NaCl solution (molecular weight

of NaCl = 58.5) would cause which of the following changes, after

osmotic equilibrium?

Intra-cellular Fluid

Volume

Intra-cellular Fluid

Osmolarity

Extra-cellular Fluid

Volume

Extracellular Fluid

Osmolarity

A) ↑ ↑ ↑ ↑

B) ↑ ↓ ↑ ↓

C) ↔ ↑ ↑ ↑

D) ↓ ↑ ↑ ↑

E) ↓ ↓ ↓ ↓

60. Lines A, B, and C on the figure above show the relative filterability

by the glomerular capillaries of dextran molecules as a function of

their molecular radius and electrical charges. Which lines on the

graph best describe the electrical charges of the dextrans?

A) A = polycationic; B = neutral; C= polyanionic

B) A = polycationic; B = polyanionic; C = neutral

C) A = polyanionic; B = neutral; C = polycationic

D) A = polyanionic; B = polycationic; C = polycationic

E) A = neutral; B = polycationic; C = polyanionic

F) A = neutral; B = polyanionic; C = polycationic

61. If distal tubule fluid creatinine concentration is 5 mg/100 ml and

plasma creatinine concentration is 1.0 mg/100 ml, what is the

approximate percentage of the water filtered by the glomerular

capillaries that remains in the distal tubule?

A) 5%

B) 10%

C) 20%

D) 50%

E) 80%

F) 95%

62. Which change tends to increase peritubular capillary fluid

reabsorption?

A) Increased blood pressure

B) Decreased filtration fraction

C) Increased efferent arteriolar resistance

D) Decreased angiotensin II

E) Increased renal blood flow

63. A 32-year-old man reports frequent urination. He is overweight (280

lb [127 kg], 5 feet 10 inches [178 cm] tall). After measuring the 24-

hour creatinine clearance, you estimate his GFR to be 150 ml/min. His

plasma glucose level is 300 mg/dl. Assuming that his renal transport

maximum for glucose is normal, as shown in the figure above, what

would be this patient’s approximate rate of urinary glucose

excretion?

A) 0 mg/min

B) 100 mg/min

C) 150 mg/min

D) 225 mg/min

E) 300 mg/min

F) Information provided is inadequate to estimate the glucose

excretion rate

64. An adrenal tumor that causes excess aldosterone secretion would

tend to __________ plasma K+

 concentration, __________ plasma pH,

__________ renin secretion, and __________ blood pressure.

A) decrease, decrease, decrease, decrease

B) decrease, increase, decrease, increase

C) decrease, decrease, decrease, increase

D) decrease, increase, increase, increase

E) increase, increase, decrease, increase

F) increase, decrease, decrease, increase

65. Which of the following tends to increase potassium secretion by the

cortical collecting tubule?

A) A diuretic that inhibits the action of aldosterone (e.g.,

spironolactone)

B) A diuretic that decreases loop of Henle sodium reabsorption

(e.g., furosemide)

C) Decreased plasma potassium concentration

D) Acute metabolic acidosis

E) Low sodium intake

66. A diabetic patient has chronic renal disease and is referred to your

nephrology clinic. According to his family physician, his creatinine

clearance has decreased from 100 ml/min to 40 ml/min during the

past 4 years. His glucose level has not been well controlled, and his

plasma pH is 7.14. Which changes, compared with before the

development of renal disease, would you expect to find, assuming

steady-state conditions and no change in electrolyte intake?

Sodium

Excretion Rate

Creati-nine

Excretion Rate

Plasma Creati-nine

Concen-tration

Plasma HCO3

Concen-tration

NH4

+

Excretion Rate

A) ↓ ↓ ↑ ↑ ↑

B) ↔ ↔ ↑ ↓ ↑

C) ↔ ↔ ↑ ↓ ↔

D) ↔ ↓ ↑ ↓ ↔

E) ↓ ↓ ↓ ↓ ↑

F) ↓ ↓ ↓ ↓ ↓

67. A 20-year-old woman comes to your office because of rapid weight

gain and marked fluid retention. Her blood pressure is 105/65 mm

Hg, her plasma protein concentration is 3.6 g/dl (normal = 7.0), and

she has no detectable protein in her urine. Which changes would you

expect to find, compared with normal?

Thoracic Lymph

Flow

Interstitial Fluid Protein

Concentration

Capillary

Filtration

Interstitial Fluid

Pressure

A) ↓ ↓ ↓ ↓

B) ↓ ↑ ↔ ↔

C) ↑ ↓ ↑ ↑

D) ↑ ↓ ↑ ↔

E) ↑ ↑ ↑ ↑

68. A 48-year-old woman reports severe polyuria (producing about 0.5 l

of urine each hour) and polydipsia (drinking two to three glasses of

water every hour). Her urine contains no glucose, and she is placed

on overnight water restriction for further evaluation. The next

morning, she is weak and confused, her sodium concentration is 160

mEq/l, and her urine osmolarity is 80 mOsm/l. Which of the

following is the most likely diagnosis?

A) Diabetes mellitus

B) Diabetes insipidus

C) Primary aldosteronism

D) Renin-secreting tumor

E) Syndrome of inappropriate ADH

69. Which substance is filtered most readily by the glomerular

capillaries?

A) Albumin in plasma

B) Neutral dextran with a molecular weight of 25,000

C) Polycationic dextran with a molecular weight of 25,000

D) Polyanionic dextran with a molecular weight of 25,000

E) Red blood cells

70. A 22-year-old woman runs a 10-km race on a hot day and becomes

dehydrated. Assuming that her ADH levels are very high and that

her kidneys are functioning normally, in which part of the renal

tubule is the most water reabsorbed?

A) Proximal tubule

B) Loop of Henle

C) Distal tubule

D) Cortical collecting tubule

E) Medullary collecting duct

71. Furosemide (Lasix) is a diuretic that also produces natriuresis.

Which of the following is an undesirable side effect of furosemide

due to its site of action on the renal tubule?

A) Edema

B) Hyperkalemia

C) Hypercalcemia

D) Decreased ability to concentrate the urine

E) Heart failure

72. A female patient has unexplained hypernatremia (plasma Na+

 = 167

mmol/l) and reports frequent urination and large urine volumes. A

urine specimen reveals that the Na+

 concentration is 15 mmol/l (very

low) and the osmolarity is 155 mOsm/l (very low). Laboratory tests

reveal the following data: plasma renin activity = 3 ng angiotensin

I/ml/h (normal = 1.0), plasma ADH = 30 pg/ml (normal = 3 pg/ml),

and plasma aldosterone = 20 ng/dl (normal = 6 ng/dl). Which of the

following is the most likely reason for her hypernatremia?

A) Simple dehydration caused by decreased water intake

B) Nephrogenic diabetes insipidus

C) Central diabetes insipidus

D) Syndrome of inappropriate ADH

E) Primary aldosteronism

F) Renin-secreting tumor

73. Which change would you expect to find in a dehydrated person

deprived of water for 24 hours?

A) Decreased plasma renin activity

B) Decreased plasma antidiuretic hormone concentration

C) Increased plasma atrial natriuretic peptide concentration

D) Increased water permeability of the collecting duct

74. Juvenile (type 1) diabetes mellitus is often diagnosed because of

polyuria (high urine flow) and polydipsia (frequent drinking) that

occur because of which of the following?

A) Increased delivery of glucose to the collecting duct interferes

with the action of antidiuretic hormone

B) Increased glomerular filtration of glucose increases Na+

reabsorption via the sodium-glucose co-transporter

C) When the filtered load of glucose exceeds the renal threshold, a

rising glucose concentration in the proximal tubule decreases the

osmotic driving force for water reabsorption

D) High plasma glucose concentration decreases thirst

E) High plasma glucose concentration stimulates ADH release

from the posterior pituitary

75. Which of the following would cause the most serious hypokalemia?

A) A decrease in potassium intake from 150 mEq/day to 60

mEq/day

D) An increase in sodium intake from 100 to 200 mEq/day

C) Excessive aldosterone secretion plus high sodium intake

D) Excessive aldosterone secretion plus low sodium intake

E) A patient with Addison’s disease

F) Treatment with a beta-adrenergic blocker

G) Treatment with spironolactone

76. A 26-year-old woman reports that she has had a severe migraine and

has taken six times more than the recommended dose of aspirin for

the past 3 days to relieve her headaches. Her plasma pH is 7.24.

Which of the following would you expect to find (compared with

normal)?

Plasma concentration

Plasma Urine

Excretion

Urine NH4

+

Excretion

Plasma Anion

Gap

A) ↑ ↓ ↑ ↑ ↑

B) ↑ ↑ ↑ ↓ ↑

C) ↓ ↓ ↓ ↓ ↓

D) ↓ ↓ ↓ ↑ ↑

E) ↓ ↓ ↓ ↑ ↓

F) ↓ ↔ ↓ ↓ ↔

77. Under conditions of normal renal function, which of the following

statements is true of the concentration of urea in tubular fluid at the

end of the proximal tubule?

A) It is higher than the concentration of urea in tubular fluid at the

tip of the loop of Henle

B) It is higher than the concentration of urea in the plasma

C) It is higher than the concentration of urea in the final urine in

antidiuresis

D) It is lower than plasma urea concentration because of active

urea reabsorption along the proximal tubule

78. You begin treating a hypertensive patient with a powerful loop

diuretic (e.g., furosemide). Which changes would you expect to find,

compared with pretreatment values, when he returns for a follow-up

examination 2 weeks later?

Urine Sodium Excretion Extracellular Fluid Volume Blood Pressure Plasma Potassium Concentration

A) ↑ ↓ ↓ ↓

B) ↑ ↓ ↔ ↔

C) ↔ ↓ ↓ ↓

D) ↔ ↓ ↔ ↔

E) ↑ ↔ ↓ ↑

79. Which change, compared with normal, would be expected to occur,

under steady-state conditions, in a patient whose severe renal disease

has reduced the number of functional nephrons to 25% of normal?

A) Increased GFR of the surviving nephrons

B) Decreased urinary creatinine excretion rate

C) Decreased urine flow rate in the surviving nephrons

D) Decreased urinary excretion of sodium

E) Increased urine-concentrating ability

80. Which statement is correct?

A) Urea reabsorption in the medullary collecting tubule is less than

in the distal convoluted tubule during antidiuresis

B) Urea concentration in the interstitial fluid of the renal cortex is

greater than in the interstitial fluid of the renal medulla during

antidiuresis

C) The thick ascending limb of the loop of Henle reabsorbs more

urea than the inner medullary collecting tubule during

antidiuresis

D) Urea reabsorption in the proximal tubule is greater than in the

cortical collecting tubule

81. A patient’s urine is collected for 2 hours, and the total volume is 600

ml during this time. Her urine osmolarity is 150 mOsm/l, and her

plasma osmolarity is 300 mOsm/l. What is her “free water clearance”?

A) +5.0 ml/min

B) +2.5 ml/min

C) 0.0 ml/min

D) −2.5 ml/min

E) −5.0 ml/min

Questions 82–85

For Questions 82–85, choose the appropriate nephron site in the above

figure.

82. In a patient with severe central diabetes insipidus caused by a lack of

ADH secretion, which part of the tubule would have the lowest

tubular fluid osmolarity?

A) A

B) B

C) C

D) D

E) E

83. In a person on a very low potassium diet, which part of the nephron

would be expected to reabsorb the most potassium?

A) A

B) B

C) C

D) D

E) E

84. Which part of the nephron normally reabsorbs the most water?

A) A

B) B

C) C

D) D

E) E

85. In a normally functioning kidney, which part of the tubule has the

lowest permeability to water during antidiuresis?

A) A

B) B

C) C

D) D

E) E

86. Which substances are best suited to measure interstitial fluid

volume?

A) Inulin and heavy water

B) Inulin and 22Na

C) Heavy water and 125I-albumin

D) Inulin and 125I-albumin

E) 51Cr red blood cells and 125I-albumin

87. Long-term administration of furosemide (Lasix) would do what?

A) Inhibit the Na+-Cl − co-transporter in the renal distal tubules

B) Inhibit the Na+-Cl − -K+

 co-transporter in the renal tubules

C) Tend to reduce renal concentrating ability

D) Tend to cause hyperkalemia

E) A and C

F) B and C

G) B, C, and D

88. A patient with normal lungs who has uncontrolled type 1 diabetes

and a plasma glucose concentration of 400 mg/100 ml (normal ∼100

mg/100 ml) would be expected to have which set of blood values?

pH Na

+

(mmol/l) Cl

(mmol/l)

A) 7.66 22 20 143 111

B) 7.52 38 48 146 100

C) 7.29 14 30 143 117

D) 7.25 12 28 142 102

E) 7.07 14 50 144 102

89. Which of the following would be expected to cause a decrease in

extracellular fluid potassium concentration (hypokalemia) at least in

part by stimulating potassium uptake into the cells?

A) α-Adrenergic blockade

B) Insulin deficiency

C) Strenuous exercise

D) Aldosterone deficiency (Addison disease)

E) Metabolic alkalosis

90. If a person has a kidney transport maximum for glucose of 350

mg/min, a GFR of 100 ml/min, a plasma glucose level of 150 mg/dl, a

urine flow rate of 2 ml/min, and no detectable glucose in the urine,

what would be the approximate rate of glucose reabsorption,

assuming normal kidneys?

A) Glucose reabsorption cannot be estimated from these data

B) 0 mg/min

C) 50 mg/min

D) 150 mg/min

E) 350 mg/min

91. Which diuretic inhibits Na+-2Cl − -K+

 co-transport in the loop of Henle

as its primary action?

A) Thiazide diuretic

B) Furosemide

C) Carbonic anhydrase inhibitor

D) Osmotic diuretic

E) Amiloride

F) Spironolactone

92. A selective decrease in efferent arteriolar resistance would __________

glomerular hydrostatic pressure, __________ GFR, and __________

renal blood flow.

A) increase, increase, increase

B) increase, decrease, increase

C) increase, decrease, decrease

D) decrease, increase, decrease

E) decrease, decrease, increase

F) decrease, increase, increase

93. A patient with renal tubular acidosis would be expected to have

which set of blood values?

pH Na+ (mmol/l) Cl

(mmol/l)

A) 7.66 22 20 143 111

B) 7.52 38 48 146 100

C) 7.07 14 50 144 102

D) 7.25 12 28 142 102

E) 7.29 14 30 143 117

94. A patient reports that he is always thirsty, and his breath has an

acetone smell. You suspect that he has diabetes mellitus, and the

diagnosis is confirmed by a urine sample that tests positive for

glucose and a blood sample that shows a fasting blood glucose

concentration of 400 mg/dl. Compared with normal, you would

expect to find which changes in his urine?

Urine pH NH4

+ Excretion Urine volume (ml/24 h) Renal Production

A) ↓ ↓ ↓ ↓

B) ↓ ↑ ↓ ↓

C) ↑ ↓ ↓ ↓

D) ↓ ↑ ↑ ↑

E) ↑ ↑ ↑ ↑

Questions 95–97

A person with normal body fluid volumes weighs 60 kg and has an

extracellular fluid volume of approximately 12.8 l, a blood volume of 4.3 l,

and a hematocrit of 0.4; 57% of his body weight is water. Use this

information to answer Questions 95–71.

95. What is the approximate intracellular fluid volume?

A) 17.1 l

B) 19.6 l

C) 21.4 l

D) 23.5 l

E) 25.6 l

96. What is the approximate plasma volume?

A) 2.0 l

B) 2.3 l

C) 2.6 l

D) 3.0 l

E) 3.3 l

97. What is the approximate interstitial fluid volume?

A) 6.4 l

B) 8.4 l

C) 10.2 l

D) 11.3 l

E) 12.0 l

98. Which nephron segment is the primary site of magnesium

reabsorption under normal conditions?

A) Proximal tubule

B) Descending limb of the loop of Henle

C) Ascending limb of the loop of Henle

D) Distal convoluted tubule

E) Collecting ducts

99. Which changes would you expect to find in a newly diagnosed 10-

year-old patient with type 1 diabetes and uncontrolled

hyperglycemia (plasma glucose = 300 mg/dl)?

Thirst (Water Intake) Urine Volume Glomerular Filtration Rate Afferent Arteriolar Resistance

A) ↑ ↓ ↑ ↓

B) ↑ ↑ ↓ ↑

C) ↑ ↑ ↑ ↓

D) ↓ ↑ ↑ ↑

E) ↓ ↓ ↓ ↓

Questions 100 and 101

To evaluate kidney function in a 45-year-old woman with type 2 diabetes,

you ask her to collect her urine for a 24-hour period. She collects 3600 ml of

urine in that period. The clinical laboratory returns the following results after

analyzing the patient’s urine and plasma samples: plasma creatinine = 4

mg/dl, urine creatinine = 32 mg/dl, plasma potassium = 5 mmol/l, and urine

potassium = 10 mmol/l.

100. What is this patient’s approximate GFR, assuming that she collected

all her urine in the 24-hour period?

A) 10 ml/min

B) 20 ml/min

C) 30 ml/min

D) 40 ml/min

E) 80 ml/min

101. What is the net renal tubular reabsorption rate of potassium in this

patient?

A) 1.050 mmol/min

B) 0.100 mmol/min

C) 0.037 mmol/min

D) 0.075 mmol/min

E) Potassium is not reabsorbed in this example

Questions 102–106

Match each of the patients described in Questions 102–106 with the correct

set of blood values in the following table (the same values may be used for

more than one patient).

pH PCO 2

(mm Hg) Na

+

(mEq/l) Cl

(mEq/l)

A) 7.66 22 20 143 111

B) 7.28 30 65 142 102

C) 7.24 12 29 144 102

D) 7.29 14 30 143 117

E) 7.52 38 48 146 100

F) 7.07 14 50 144 102

102. A patient with severe diarrhea

103. A patient with primary aldosteronism

104. A patient with proximal renal tubular acidosis

105. A patient with diabetic ketoacidosis and emphysema

106. A patient treated chronically with a carbonic anhydrase inhibitor

107. Which change would you expect to find in a patient who developed

acute renal failure after ingesting poisonous mushrooms that caused

renal tubular necrosis?

A) Increased plasma bicarbonate concentration

B) Metabolic acidosis

C) Decreased plasma potassium concentration

D) Decreased blood urea nitrogen concentration

E) Decreased hydrostatic pressure in Bowman’s capsule

108. Which of the following has similar values for both intracellular and

interstitial body fluids?

A) Potassium ion concentration

B) Colloid osmotic pressure

C) Sodium ion concentration

D) Chloride ion concentration

E) Total osmolarity

109. Which of the following is true of the tubular fluid that passes

through the lumen of the early distal tubule in the region of the

macula densa?

A) It is usually isotonic

B) It is usually hypotonic

C) It is usually hypertonic

D) It is hypertonic in antidiuresis

E) It is hypertonic when the filtration rate of its own nephron

decreases to 50% below normal

110. In a person with normal kidneys and normal lungs who has chronic

metabolic acidosis, you would expect to find all of the following,

compared with normal, EXCEPT:

A) Increased renal excretion of NH4Cl

B) Decreased urine pH

B) Decreased urine excretion

D) Increased plasma concentration

E) Decreased plasma

111. Assume that you have a patient who needs fluid therapy and you

decide to administer by intravenous infusion 2.0 l of 0.45% NaCl

solution (molecular weight NaCl = 58.5). After osmotic equilibrium,

which changes would you expect, compared with before infusion of

the NaCl?

INTRA-CELLULAR

VOLUME

INTRA-CELLULAR

OSMOLARITY

ExTRA-CELLULAR

VOLUME

ExTRA-CELLULAR

OSMOLARITY

A) ↑ ↑ ↑ ↑

B) ↑ ↓ ↑ ↓

C) ↔ ↑ ↑ ↑

D) ↓ ↑ ↑ ↑

E) ↓ ↓ ↓ ↓

112. If the renal clearance of substance X is 300 ml/min and the

glomerular filtration rate is 100 ml/min, it is most likely that

substance X is

A) Filtered freely but not secreted or reabsorbed

B) Bound to plasma proteins

C) Secreted

D) Reabsorbed

E) Bound to tubular proteins

F) Clearance of a substance cannot be greater than the GFR

113. Which change tends to increase urinary calcium (Ca2+) excretion?

A) Extracellular fluid volume expansion

B) Increased plasma parathyroid hormone concentration

C) Decreased blood pressure

D) Increased plasma phosphate concentration

E) Metabolic alkalosis

114. Which change would you expect to find in a patient consuming a

high-sodium diet (200 mEq/day) compared with the same patient on

a normal-sodium diet (100 mEq/day), assuming steady-state

conditions?

A) Increased plasma aldosterone concentration

B) Increased urinary potassium excretion

C) Decreased plasma renin activity

D) Decreased plasma atrial natriuretic peptide

E) An increase in plasma sodium concentration of at least 5 mmol/l

115. What would tend to decrease GFR by more than 20% in a normal

kidney?

A) Decrease in renal arterial pressure from 100 to 85 mm Hg

B) 50% decrease in afferent arteriolar resistance

C) 50% decrease in efferent arteriolar resistance

D) 50% increase in the glomerular capillary filtration coefficient

E) Decrease in plasma colloid osmotic pressure from 28 to 20 mm

Hg

116. Acute metabolic acidosis tends to _____ intracellular K+

concentration and _____ K+

 secretion by the cortical collecting

tubules.

A) Increase, increase

B) Increase, decrease

C) Decrease, increase

D) Decrease, decrease

E) Cause no change in, increase

F) Cause no change in, cause no change in

117. Which statement is true?

A) ADH increases water reabsorption from the ascending loop of

Henle

B) Water reabsorption from the descending loop of Henle is

normally less than that from the ascending loop of Henle

C) Sodium reabsorption from the ascending loop of Henle is

normally less than that from the descending loop of Henle

D) Osmolarity of fluid in the early distal tubule would be less than

300 mOsm/l in a dehydrated person with normal kidneys and

increased ADH levels

E) ADH decreases the urea permeability in the medullary

collecting tubules

118. In a person on a high-potassium (200 mmol/day) diet, which part of

the nephron would be expected to secrete the most potassium?

A) Proximal tubule

B) Descending loop of Henle

C) Ascending loop of Henle

D) Early distal tubule

E) Collecting tubules

119. Which of the following would you expect to find in a patient who

has chronic diabetic ketoacidosis?

A) Decreased renal excretion, increased NH4

+

 excretion,

increased plasma anion gap

B) Increased respiration rate, decreased arterial , decreased

plasma anion gap

C) Increased NH4

+

 excretion, increased plasma anion gap,

increased urine pH

D) Increased renal production, increased NH4

+

 excretion,

decreased plasma anion gap

E) Decreased urine pH, decreased renal excretion, increased

arterial

120. Using the indicator dilution method to assess body fluid volumes in

a 40-year-old man weighing 70 kg, the inulin space is calculated to be

16 l, and 125I-albumin space is 4 l. If 60% of his total body weight is

water, what is his approximate interstitial fluid volume?

A) 4 l

B) 12 l

C) 16 l

D) 26 l

E) 38 l

F) 42 l

121. What would tend to decrease plasma potassium concentration by

causing a shift of potassium from the extracellular fluid into the cells?

A) Strenuous exercise

B) Aldosterone deficiency

C) Acidosis

D) β-adrenergic blockade

E) Insulin excess

122. A 26-year-old construction worker is brought to the emergency

department with a change in mental status after working a 10-hour

shift on a hot summer day (average outside temperature was 97°F

[36°C]). The man had been sweating profusely during the day but did

not drink fluids. He has a fever of 102°F [39°C], a heart rate of 140

beats/min, and a blood pressure of 100/55 mm Hg in the supine

position. Upon examination, he has no perspiration, appears to have

dry mucous membranes, and is poorly oriented to person, place, and

time. Assuming that his kidneys were normal yesterday, which set of

hormone levels describes his condition, compared with normal?

A) High ADH, high renin, low angiotensin II, low aldosterone

B) Low ADH, low renin, low angiotensin II, low aldosterone

C) High ADH, low renin, high angiotensin II, low aldosterone

D) High ADH, high renin, high angiotensin II, high aldosterone

E) Low ADH, high renin, low angiotensin II, high aldosterone

123. A 23-year-old man runs a 10-km race in July and loses 2 l of fluid by

sweating. He also drinks 2 l of water during the race. Which changes

would you expect, compared with normal, after he absorbs the water

and assuming osmotic equilibrium and no excretion of water or

electrolytes?

INTRA-CELLULAR

VOLUME

INTRA-CELLULAR

OSMOLARITY

ExTRA-CELLULAR

VOLUME

ExTRA-CELLULAR

OSMO-LARITY

A) ↓ ↑ ↓ ↑

B) ↓ ↓ ↓ ↓

C) ↔ ↓ ↔ ↓

D) ↔ ↑ ↓ ↑

E) ↑ ↓ ↓ ↓

F) ↑ ↓ ↑ ↓

124. Which change would tend to increase Ca2+

 reabsorption in the renal

tubule?

A) Extracellular fluid volume expansion

B) Increased plasma parathyroid hormone concentration

C) Increased blood pressure

D) Decreased plasma phosphate concentration

E) Metabolic acidosis

125. A young man is found comatose, having taken an unknown

number of sleeping pills an unknown time before. An arterial blood

sample yields the following values: pH = 7.02, = 14 mEq/l, and

= 68 mm Hg. Which of the following describes this patient’s acidbase status most accurately?

A) Uncompensated metabolic acidosis

B) Uncompensated respiratory acidosis

C) Simultaneous respiratory and metabolic acidosis

D) Respiratory acidosis with partial renal compensation

E) Respiratory acidosis with complete renal compensation

126. In a person with chronic respiratory acidosis who has partial renal

compensation, you would expect to find which changes, compared

with normal? ______ urinary excretion of NH4

+

; ______ plasma

concentration; and _____ urine pH.

A) Increased, increased, decreased

B) Increased, decreased, decreased

C) No change in, increased, decreased

D) No change in, no change in, decreased

E) Increased, no change in, increased

127. Increases in both renal blood flow and GFR are caused by which

mechanism?

A) Dilation of the afferent arterioles

B) Increased glomerular capillary filtration coefficient

C) Increased plasma colloid osmotic pressure

D) Dilation of the efferent arterioles

128. A 55-year-old male patient with hypertension has had his blood

pressure reasonably well controlled by administration of a thiazide

diuretic. At his last visit (6 months ago), his blood pressure was

130/75 mm Hg, and his serum creatinine was 1 mg/100 ml. He has

been exercising regularly for the past 2 years but recently has

reported knee pain and began taking large amounts of a nonsteroidal

antiinflammatory drug. When he arrives at your office, his blood

pressure is 155/85 mm Hg, and his serum creatinine is 2.5 mg/100 ml.

What best explains his increased serum creatinine level?

A) Increased efferent arteriolar resistance that reduced GFR

B) Increased afferent arteriolar resistance that reduced GFR

C) Increased glomerular capillary filtration coefficient that reduced

GFR

D) Increased angiotensin II formation that decreased GFR

E) Increased muscle mass due to the exercise

129. An older adult patient reports muscle weakness and lethargy. A

urine specimen reveals a Na+

 concentration of 600 mmol/l and an

osmolarity of 1200 mOsm/l. Additional laboratory tests provide the

following information: plasma Na+

 concentration = 167 mmol/l,

plasma renin activity = 4 ng angiotensin I/ml/h (normal = 1 ml/h),

plasma ADH = 60 pg/ml (normal = 3 pg/ml), and plasma aldosterone

= 15 ng/dl (normal = 6 ng/dl). What is the most likely reason for this

patient’s hypernatremia?

A) Dehydration caused by decreased fluid intake

B) Syndrome of inappropriate ADH

C) Nephrogenic diabetes insipidus

D) Primary aldosteronism

E) Renin-secreting tumor

Answers

1. E) Increased plasma protein concentration would reduce the net force

favoring capillary filtration and would oppose edema formation. All of the

other changes would increase capillary filtration rate and tend to cause

edema.

TMP14 pp. 316–318.

2. E) The total body water would initially be 60% of body weight or ∼30 l.

The initial total mOsm in the body fluids would be 30 l × 360 mOsm/l, or

10,800 mOsm. After adding the glucose solution, total body water would

be 30 l + 2 l or 32 l. If we assume that all of the glucose is metabolized, the

final mOsm would also be 10,800. Therefore, the extracellular and

intracellular osmolarity would be 10,800 mOsm/32 l, or approximately 338

mOsm/l.

TMP14 pp. 312–314

3. B) The initial total body water would be 60% of body weight or ∼30 L. The

initial total mOsm in the body fluids would be 30 l × 360 mOsm/l, or 10,800

mOsm. The initial extracellular fluid volume would be 20% of body

weight, or ∼10 l. The initial total mOsm in the extracellular fluid would be

360 mOsm/l × 10 l, or 3600 mOsm. After adding the glucose solution, total

body water would be 30 l + 2 l or 32 l. If we assume that all of the glucose is

metabolized, the final mOsm would also be 10,800. Therefore, the

extracellular and intracellular osmolarity would be 10,800 mOsm/32 l, or

approximately 338 mOsm/l. The extracellular fluid would continue to have

a total of 3600 mOsm. Therefore, the final extracellular fluid volume would

be 3600 mOsm/338 mOsm per l, or ∼10.7 l.

TMP14 pp. 312–314

4. C) A doubling of serum creatinine implies a reduction in glomerular

filtration rate (GFR). Non-steroidal anti-inflammatory drugs (NSAIDS)

inhibit prostaglandin synthesis, which would tend increase afferent

arteriolar and reduce GFR.

TMP14 pp. 337–342, 362.

5. C) Inhibition of sodium-glucose co-transporter 2 (SGLT2) would reduce

glucose and sodium chloride reabsorption in the proximal tubules, causing

increased sodium chloride delivery to the macula densa, which would, in

turn, cause a feedback-mediated vasoconstriction of afferent arterioles and

reductions in glomerular filtration rate and renal blood flow.

TMP14 pp. 339–340, 345–346

6. D) Filtration fraction (FF) = glomerular filtration rate (GFR)/renal plasma

flow. GFR = Kf x (PG - ΠC – PB

) where Kf is the glomerular capillary

filtration coefficient, PG is glomerular hydrostatic pressure, ΠC is

glomerular capillary colloid osmotic pressure, and PB

 is Bowman’s space

hydrostatic pressure. Therefore, GFR= 10 × (60 − 20 − 10) = 100 ml/min.

Since renal plasma flow is 600 ml/min, FF= 100 ml/min/600 ml/min = 0.167.

TMP14 pp. 333–336.

7. D) Bicarbonate is more avidly reabsorbed in the proximal tubules than

water, and therefore concentration decreases along the proximal tubules

and has a lower concentration in the urine than in the glomerular filtrate.

Approximately 85% of the filtered load of is normally reabsorbed in

the proximal tubules. All of the other statements are correct.

TMP14 pp. 354, 410–412.

8. B) With a glomerular filtration rate of 100 ml/min and a plasma glucose

concentration of 4 mg/ml, the filtered load of glucose would be 400

mg/min. Since the transport maximum for glucose in this case is 200

mg/min, the maximum rate of glucose reabsorption is 200 mg/min.

Glucose excreted rate is therefore the difference between the filtered load

of glucose (400 mg/min) and the glucose reabsorption rate (200 mg/min),

or 200 mg/min.

TMP14 pp. 346–347

9. F) Fluid in the ascending loop of Henle becomes dilute as electrolytes are

reabsorbed and water remains in the tubule. When antidiuretic hormone

(ADH) levels are very low, as occurs in central diabetes insipidus, fluid in

the distal and collecting tubules, and the collecting ducts is further diluted

by the reabsorption of sodium chloride and the failure to reabsorb water.

This lead to a very dilute urine (see figure below).

TMP14 p. 366

10. B) Potassium excretion in this case is equal to urine concentration of K+

(80 mmol/l) multiplied by the urine flow rate (2.0 ml/min, or 0.002 l/min)

which is 0.16 mmol/min.

TMP14 pp. 363–364

11. E) A 50% reduction in glomerular filtration rate (GFR) would initially

cause a reduction in creatinine excretion rate. However, within a few days,

the filtered load and excretion of creatinine would return to normal as

serum creatinine concentration increased to approximately twice the

normal level under steady-state conditions (see figure in next column).

Creatinine clearance is approximately equal to GFR and would also be

reduced by approximately 50%.

TMP14 pp. 361–362

12. A) Urine excretion rate is equal to glomerular filtration rate (GFR) minus

tubular reabsorption rate. In this example, the final is 40 ml/min, and the

tubular reabsorption rate is 40 ml/min. Therefore, the urine excretion rate

is zero.

TMP14 p. 343

13. D) Total renal plasma flow (RPF) is equal to the clearance of

paraaminohippuric acid (PAH) divided by the renal PAH extraction ratio

(EPAH).

Clearance of PAH (CPAH) = (UPAH × V) / APAH = (200 µg/ml × 2 ml/min)/1.0

µg/ml = 400 ml/min (EPAH) = (APAH – VPAH)/APAH = (1.0 µg/ml − 0.2

µg/ml)/1.0 µg/ml = 0.8

RPF = 400 ml/min / 0.8 = 500 ml/min

Where UPAH is urine PAH concentration, APAH is arterial PAH concentration,

VPAH is renal venous PAH concentration, and V is urine flow rate.

TMP14 pp. 362–363

14. A) Type A intercalated cells of the collecting tubules secrete H+

 by a

hydrogen-ATPase transporter and by a hydrogen-potassium-ATPase

transporter. They also reabsorb and K+

 (see figure on next page).

TMP14 pp. 352–354

15. E) Increased extracellular fluid osmolarity would tend to cause

hyperkalemia, rather than hypokalemia due to cell dehydration, which raise

intracellular potassium concentration and promotes potassium diffusion

into the extracellular fluid. All of the other statements are correct.

TMP14 pp. 383–384

16. C) The cortical collecting tubule is relatively impermeable to urea, and

very lile reabsorption occurs in this tubular segment, even in the presence

of ADH. All of the other statements are correct.

TMP14 pp. 370–371

17. C) The thick ascending loop of Henle is relatively impermeable to water

even in the presence of high levels of ADH. The other tubular segments

reabsorb significant amounts of water.

TMP14 pp. 368–369, 372–373

18. D) Acute metabolic alkalosis tends to shift K+

 from the extracellular fluid

into the cells, including the renal tubular cells, contributing to increased K+

secretion and decreased plasma K+

 concentration (hypokalemia).

TMP14 pp. 384, 389

19. D) Aldosterone antagonists such as spironolactone tend to cause

hyperkalemia rather than hypokalemia by shifting potassium from the

intracellular to the extracellular fluid and by inhibiting potassium

secretion in the principal cells of collecting tubules. All of the other

statements are correct.

TMP14 pp. 384, 387

20. C) Under steady-state conditions and no changes in protein or electrolyte

intake, excretion rate of sodium would not change and would be equal to

sodium intake. Creatinine excretion rate would also be unchanged in the

steady-state due to increased plasma creatinine concentration, which

would return the filtered load of creatinine to normal despite a 50%

reduction in glomerular filtration rate (creatinine clearance). Uncontrolled

diabetes mellitus can also result in metabolic acidosis, which would reduce

plasma bicarbonate concentration and stimulate a compensatory increased

in renal ammonium production and increased ammonium excretion rate.

TMP14 pp. 420, 428–429.

21. D) The patient has a low pH (7.18) and a low plasma ,indicating

metabolic acidosis. The plasma anion gap is

This is far above normal (8-16 mEq/l), indicating unmeasured anions and

excess nonvolatile acids. Therefore, the most likely explanation for the

patient’s metabolic acidosis is methanol poisoning. Patients with

emphysema would have respiratory acidosis. Patients with diarrhea or

renal tubular acidosis would have metabolic acidosis due to bicarbonate

loss and normal anion gap with hyperchloremia. Ingestion of excess

sodium bicarbonate would cause metabolic alkalosis.

TMP14 pp. 418–420

22. E) Atrial natriuretic peptide decreases renal sodium reabsorption through

direct effects on the renal tubules as well as indirectly by inhibiting renin

secretion. Both of these effects contribute to increased renal sodium

excretion.

TMP14 p. 360

23. A) Urine excretion rate of Na+

 is equal to urine flow rate (1.0 ml/min or

0.001 l/min) multiplied by urine Na+

 concentration (80 mmol/l), or 0.08

mmol/min.

TMP14 pp. 361–363

24. A) Approximately 65% of the filtered load of potassium is reabsorbed in

the proximal tubule. Variations in renal excretion of potassium during

change in potassium intake are achieved mainly by changes in potassium

secretion in collecting tubules. With high potassium intake, the proximal

tubule still reabsorbs a high fraction of the filtered load of potassium.

TMP14 pp. 384–386

25. A) Metabolic alkalosis shifts potassium from the extracellular fluid into

the cells and contributes to hypokalemia. Insulin deficiency, aldosterone

deficiency, beta-adrenergic blockade, and increased extracellular fluid

osmolarity all cause a shift of potassium from the cells to the extracellular

fluid.

TMP14 pp. 383–384

26. E) Theoretically, if all of the plasma flowing through the kidneys was

cleared of a substance, the clearance rate would be equal to the total renal

plasma flow. In this example, renal plasma flow is equal to 800 ml/min.

TMP14 pp. 362–363

27. C) A 3% NaCl solution is hypertonic, and when infused intravenously, it

would increase extracellular fluid volume and osmolarity, thereby causing

water to flow out of the cell. This action would decrease intracellular fluid

volume and further increase extracellular fluid volume. The 0.9% NaCl

solution and 5% dextrose solution are isotonic and therefore would not

reduce intracellular fluid volume. Pure water and the 0.45% NaCl solution

are hypotonic, and when infused, they would increase both intracellular

and extracellular fluid volumes.

TMP14 pp. 311–314

28. B) Partial obstruction of a major vein draining a tissue would increase

capillary hydrostatic pressure in the tissue, which, in turn, would raise

capillary fluid filtration and cause increases interstitial fluid volume,

interstitial fluid hydrostatic pressure, and lymph flow. The increased

lymph flow would “wash out” proteins from the interstitial fluid,

decreasing interstitial fluid protein concentration.

TMP14 pp. 316–317

29. C) The hypernatremia (plasma Na+

 = 165 mmol/l) associated with a low

blood pressure (88/44 mm Hg) suggests dehydration. The frequent

urination and low urine specific gravity (1.003, which implies a urine

osmolarity of about 100-120 mOsm/l) despite hypernatremia and

dehydration suggests diabetes insipidus due to either insufficient secretion

of ADH (central diabetes insipidus) or failure of the kidneys to respond to

ADH (nephrogenic diabetes insipidus).

TMP14 pp. 315–316, 374, 432

30. D) A severe renal artery stenosis that reduces GFR to 25% of normal

would also decrease renal blood flow but would cause only a transient

decrease in urinary creatinine excretion. The transient decrease in

creatinine excretion would increase serum creatinine (to about four times

normal), which would restore the filtered creatinine load to normal and

therefore return urinary creatinine excretion to normal levels under

steady-state conditions. Urinary sodium secretion would also decrease

transiently but would be restored to normal so that intake and excretion of

sodium are balanced. Plasma sodium concentration would not change

significantly because it is carefully regulated by the ADH–thirst

mechanism.

TMP14 pp. 362, 428–429

31. B) A 1% solution of dextrose is hypotonic, and when infused, it would

increase both intracellular and extracellular fluid volumes while

decreasing the osmolarity of these compartments.

TMP14 pp. 311–313

32. B) Excessive secretion of ADH would increase renal tubular reabsorption

of water, thereby increasing extracellular fluid volume and reducing

extracellular fluid osmolarity. The reduced osmolarity, in turn, would

cause water to flow into the cells and raise intracellular fluid volume. In

the steady state, both extracellular and intracellular fluid volumes would

increase, and osmolarity of both compartments would decrease.

TMP14 pp. 314, 375–376

33. C) A 3% solution of NaCl is hypertonic, and when infused into the

extracellular fluid, it would raise osmolarity, thereby causing water to flow

out of the cells into the extracellular fluid until osmotic equilibrium is

achieved. In the steady state, extracellular fluid volume would increase,

intracellular fluid volume would decrease, and osmolarity of both

compartments would increase.

TMP14 pp. 311–313

34. C) Aldosterone stimulates potassium secretion by the principal cells of

the collecting tubules. Therefore, blockade of the action of aldosterone

with spironolactone would inhibit potassium secretion. Other factors that

stimulate potassium secretion by the cortical collecting tubule include

increased potassium concentration, increased cortical collecting tubule

flow rate (as would occur with high sodium intake or a diuretic that

reduces proximal tubular sodium reabsorption), and acute alkalosis.

TMP14 pp. 386–389

35. C) Phosphate excretion by the kidneys is controlled by an overflow

mechanism. When the transport maximum for reabsorbing phosphate is

exceeded, the remaining phosphate in the renal tubules is excreted in the

urine and can be used to buffer hydrogen ions and form titratable acid.

Phosphate normally begins to spill into the urine when the concentration

of extracellular fluid rises above a threshold of 0.8 mmol/l, which is usually

exceeded.

TMP14 pp. 391–392

36. B) GFR is equal to inulin clearance, which is calculated as the urine inulin

concentration (100 mg/ml) × urine flow rate (1 ml/min)/plasma inulin

concentration (2 mg/ml), which is equal to 50 ml/min.

TMP14 pp. 360–361

37. D) The net urea reabsorption rate is equal to the filtered load of urea (GFR

[50 ml/min] × plasma urea concentration [2.5 mg/ml]) − urinary excretion

rate of urea (urine urea concentration [50 mg/ml] × urine flow rate [1

ml/min]). Therefore, net urea reabsorption = (50 ml/min × 2.5 mg/ml) − (50

mg/ml × 1 ml/min) = 75 mg/min.

TMP14 pp. 360–363

38. B) As water flows up the ascending limb of the loop of Henle, solutes are

reabsorbed, but this segment is relatively impermeable to water;

progressive dilution of the tubular fluid occurs so that the osmolarity

decreases to approximately 100 mOsm/l by the time the fluid reaches the

early distal tubule. Even during maximal antidiuresis, this portion of the

renal tubule is relatively impermeable to water and is therefore called the

diluting segment of the renal tubule.

TMP14 pp. 372–373

39. C) In the absence of ADH secretion, a marked increase in urine volume

occurs because the late distal and collecting tubules are relatively

impermeable to water. As a result of increased urine volume, there are

dehydration, increased plasma osmolarity and high plasma sodium

concentration, and increased thirst which leads to increased water intake.

The dehydration reduces extracellular fluid volume increases renin

secretion and plasma renin concentration.

TMP14 pp. 375–376, 378–379

40. C) When potassium intake is doubled (from 80 to 160 mmol/day),

potassium excretion also approximately doubles within a few days, and

the plasma potassium concentration increases only slightly. Increased

potassium excretion is achieved largely by increased secretion of

potassium in the cortical collecting tubule. Increased aldosterone

concentration plays a significant role in increasing potassium secretion and

in maintaining a relatively constant plasma potassium concentration

during increases in potassium intake. Sodium excretion does not change

markedly during chronic increases in potassium intake.

TMP14 pp. 386–389

41. D) Most of the daily variation in potassium excretion is caused by

changes in potassium secretion in the late distal tubules and collecting

tubules. Therefore, when the dietary intake of potassium increases, the

total body balance of potassium is maintained primarily by an increase in

potassium secretion in these tubular segments. Increased potassium intake

has lile effect on GFR or on reabsorption of potassium in the proximal

tubule and loop of Henle. Although high potassium intake may cause a

slight shift of potassium into the intracellular compartment, a balance

between intake and output must be achieved by increasing the excretion of

potassium during high potassium intake.

TMP14 pp. 384–386

42. E) A 50% decrease in efferent arteriolar resistance would cause a

substantial decrease in GFR. A decrease in renal arterial pressure from 100

to 80 mm Hg in a normal kidney would cause only a slight reduction in

GFR in a normal kidney because of autoregulation. All of the other

changes would tend to increase GFR.

TMP14 pp. 333–336

43. A) The patient described has protein in the urine (proteinuria) and

reduced plasma protein concentration as a result of glomerulonephritis

caused by an untreated streptococcal infection (“strep throat”). The

reduced plasma protein concentration, in turn, decreased the plasma

colloid osmotic pressure and resulted in leakage from the plasma to the

interstitium. The extracellular fluid edema raised interstitial fluid pressure

and interstitial fluid volume, causing increased lymph flow and decreased

interstitial fluid protein concentration. Increasing lymph flow causes a

“washout” of the interstitial fluid protein as a safety factor against edema.

The decreased blood volume would tend to lower blood pressure and

stimulate the secretion of renin by the kidneys, raising the plasma renin

concentration.

TMP14 pp. 316–319

44. C) In a patient with a very high rate of renin secretion, there would also

be increased formation of angiotensin II, which in turn would stimulate

aldosterone secretion. The increased levels of angiotensin II and

aldosterone would cause a transient decrease in sodium excretion, which

would cause expansion of the extracellular fluid volume and increased

arterial pressure. The increased arterial pressure, as well as other

compensations, would return sodium excretion to normal so that intake

and output are balanced. Therefore, under steady-state conditions, sodium

excretion would be normal and equal to sodium intake. The increased

aldosterone concentration would cause hypokalemia (decreased plasma

potassium concentration), whereas the high level of angiotensin II would

cause renal vasoconstriction and decreased renal blood flow.

TMP14 pp. 358, 387

45. B) This patient has respiratory acidosis because the plasma pH is lower

than the normal level of 7.4, and the plasma is higher than the normal

level of 40 mm Hg. The elevation in plasma bicarbonate concentration

above normal (∼24 mEq/l) is due to partial renal compensation for the

respiratory acidosis. Therefore, this patient has respiratory acidosis with

partial renal compensation.

TMP14 pp. 415–416

46. D) GFR is approximately equal to creatinine clearance, which is

calculated as the urine creatinine concentration (50 mg/100 ml) × urine

flow rate (3 ml/min)/plasma creatinine concentration (3 mg/100 ml), which

is equal to 50 ml/min. Urine flow rate = 4320 ml/24 h = 4320 ml/1440 min =

3 ml/min.

TMP14 pp. 360–361

47. C) Because the patient has a low plasma pH (normal = 7.4), he has

acidosis. The fact that his plasma bicarbonate concentration is also low

(normal = 24 mEq/l) indicates that he has metabolic acidosis. However, he

also appears to have respiratory acidosis because his plasma is high

(normal = 40 mm Hg). The rise in is due to his impaired breathing as a

result of cardiopulmonary arrest. Therefore, the patient has a mixed

acidosis with combined metabolic and respiratory acidosis.

TMP14 pp. 415–416

48. D) An important compensation for respiratory acidosis is increased renal

production of NH4

+

 and increased NH4

+

 excretion. In acidosis, urinary

excretion of would be reduced, as would urine pH, and urinary

titratable acid would be slightly increased as a compensatory response to

the acidosis.

TMP14 pp. 412–414

49. B) Inhibition of aldosterone causes hyperkalemia by two mechanisms: (1)

shifting potassium out of the cells into the extracellular fluid and (2)

decreasing cortical collecting tubular secretion of potassium. Increasing

potassium intake from 60 to 180 mmol/day would cause only a very small

increase in plasma potassium concentration in a person with normal

kidneys and normal aldosterone feedback mechanisms. A reduction in

sodium intake also has very lile effect on plasma potassium

concentration. Chronic treatment with a diuretic that inhibits loop of

Henle Na+

-2Cl −

-K+

 co-transport would tend to cause potassium loss in the

urine and hypokalemia. However, chronic treatment with a diuretic that

inhibits sodium reabsorption in the collecting ducts, such as amiloride,

would have lile effect on plasma potassium concentration.

TMP14 pp. 383–384, 386–389

50. D) Excessive activity of the amiloride-sensitive sodium channel in the

collecting tubules would cause a transient decrease in sodium excretion

and expansion of extracellular fluid volume, which in turn would increase

arterial pressure and decrease renin secretion, leading to decreased

aldosterone secretion. Under steady-state conditions, sodium excretion

would return to normal so that intake and renal excretion of sodium are

balanced. One of the mechanisms that re-establishes this balance between

intake and output of sodium is the rise in arterial pressure that induces a

“pressure natriuresis.”

TMP14 pp. 394, 433

51. A) Primary excessive secretion of aldosterone (Conn syndrome) would be

associated with marked hypokalemia and metabolic alkalosis (increased

plasma pH). Because aldosterone stimulates sodium reabsorption and

potassium secretion by the cortical collecting tubule, there could be a

transient decrease in sodium excretion and an increase in potassium

excretion, but under steady-state conditions, both urinary sodium and

potassium excretion would return to normal to match the intake of these

electrolytes. However, the sodium retention and the hypertension

associated with aldosterone excess would tend to reduce renin secretion.

TMP14 pp. 384, 387, 398, 417

52. C) The glomerular capillary filtration coefficient is the product of the

hydraulic conductivity and surface area of the glomerular capillaries.

Therefore, increasing the glomerular capillary filtration coefficient tends to

increase GFR. Increased afferent arteriolar resistance, decreased efferent

arteriolar resistance, increased Bowman’s capsule hydrostatic pressure,

and decreased glomerular hydrostatic pressure tend to decrease GFR.

TMP14 pp. 333–336

53. D) Impairment of proximal tubular NaCl reabsorption would increase

NaCl delivery to the macula densa, which in turn would cause a

tubuloglomerular feedback–mediated increase in afferent arteriolar

resistance. The increased afferent arteriolar resistance would decrease the

GFR. Initially there would be a transient increase in sodium excretion, but

after 3 weeks, steady-state conditions would be achieved. Sodium

excretion would equal sodium intake, and no significant change would

occur in urinary sodium excretion.

TMP14 pp. 339–340

54. C) The patient has a lower than normal pH and is therefore acidotic.

Because the plasma bicarbonate concentration is also lower than normal,

the patient has metabolic acidosis with respiratory compensation (i.e.,

is lower than normal). The plasma anion gap is in the

normal range, suggesting that the metabolic acidosis is not caused by

excess nonvolatile acids such as salicylic acid or ketoacids caused by

diabetes mellitus. Therefore, the most likely cause of the metabolic acidosis

is diarrhea, which would cause a loss of in the feces and would be

associated with a normal anion gap and a hyperchloremic (increased

chloride concentration) metabolic acidosis.

TMP14 pp. 415–416, 419–420

55. A) A 50% reduction of GFR would approximately double the plasma

creatinine concentration because creatinine is not reabsorbed or secreted,

and its excretion depends largely on glomerular filtration. Therefore, when

GFR decreases, the plasma concentration of creatinine increases until the

renal excretion of creatinine returns to normal. Plasma concentrations of

glucose, potassium, sodium, and hydrogen ions are closely regulated by

multiple mechanisms that keep them relatively constant even when GFR

falls to very low levels. Plasma phosphate concentration is also maintained

near normal until GFR falls to below 20% to 30% of normal.

TMP14 pp. 429–430

56. C) A 50% reduction in afferent arteriolar resistance with no change in

arterial pressure would increase renal blood flow and glomerular

hydrostatic pressure, thereby increasing GFR. At the same time, the

reduction in afferent arteriolar resistance would raise peritubular capillary

hydrostatic pressure.

TMP14 pp. 333–336, 356–357

57. A) The net filtration pressure at the glomerular capillaries is equal to the

sum of the forces favoring filtration (glomerular capillary hydrostatic

pressure) minus the forces that oppose filtration (hydrostatic pressure in

Bowman’s space and glomerular colloid osmotic pressure). Therefore, the

net pressure driving glomerular filtration is 50 − 12 − 30 = 8 mm Hg.

TMP14 p. 333

58. D) Uncontrolled diabetes mellitus results in increased blood acetoacetic

acid levels, which in turn cause metabolic acidosis and decreased plasma

and pH. The acidosis causes several compensatory responses,

including increased respiratory rate, which reduces plasma ; increased

renal NH+

 production, which leads to increased NH+

 excretion; and

increased phosphate buffering of hydrogen ions secreted by the renal

tubules, which increases titratable acid excretion.

TMP14 pp. 415–417

59. B) Infusion of a hypotonic solution of NaCl would initially increase

extracellular fluid volume and decrease extracellular fluid osmolarity. The

reduction in extracellular fluid osmolarity would cause osmotic flow of

fluid into the cells, thereby increasing intracellular fluid volume and

decreasing intracellular fluid osmolarity after osmotic equilibrium.

TMP14 pp. 312–313

60. A) For any given molecular radius, positively charged molecules (cations)

are filtered more readily than negatively charged molecules (anions)

because negative charges on the proteins of the basement membrane and

podocytes of the glomerular capillaries tend to repel large negatively

changed molecules (e.g., polycationic dextrans, curve C). Large positively

charged molecules (curve A) are filtered more readily.

TMP14 pp. 332–333

61. C) Because water is reabsorbed by the renal tubules but creatinine is not

reabsorbed, the concentration of creatinine in the renal tubular fluid will

increase as fluid flows from the proximal to the distal tubule. An increase

in the concentration from 1.0 mg/100 ml in the proximal tubule to 5.0

mg/100 ml in the distal tubule means that only about one fifth (20%) of the

water that was in the proximal tubules remains in the distal tubule.

TMP14 pp. 349–350

62. C) Peritubular capillary fluid reabsorption is determined by the balance of

hydrostatic and colloid osmotic forces in the peritubular capillaries.

Increased efferent arteriolar resistance reduces peritubular capillary

hydrostatic pressure and therefore increases the net force favoring fluid

reabsorption. Increased blood pressure tends to raise peritubular capillary

hydrostatic pressure and reduce fluid reabsorption. Decreased filtration

fraction increases the peritubular capillary colloid osmotic pressure and

tends to reduce peritubular capillary reabsorption. Decreased angiotensin

II causes vasodilatation of efferent arterioles, raising peritubular capillary

hydrostatic pressure, decreasing reabsorption, and decreasing tubular

transport of water and electrolytes. Increased renal blood flow also tends

to raise peritubular capillary hydrostatic pressure and decrease fluid

reabsorption.

TMP14 pp. 355–357

63. C) The filtered load of glucose in this example is determined as follows:

GFR (150 ml/min) × plasma glucose (300 mg/dl) = 450 mg/min. The

transport maximum for glucose in this example is 300 mg/min. Therefore,

the maximum rate of glucose reabsorption is 300 mg/min. The urinary

glucose excretion is equal to the filtered load (450 mg/min) minus the

tubular reabsorption of glucose (300 mg/min), or 150 mg/min.

TMP14 pp. 346–347, 360–361

64. B) Excess aldosterone increases sodium reabsorption and potassium

secretion by the principal cells of the collecting tubules, causing sodium

retention, increased blood pressure, and decreased renin secretion while

increasing excretion of potassium and tending to decrease plasma

potassium concentration. Excess aldosterone also causes a shift of

potassium from the extracellular fluid into the cells, further reducing

plasma potassium concentration. Aldosterone excess also stimulates

hydrogen ion secretion and bicarbonate reabsorption by the intercalated

cells and tends to increase plasma pH (alkalosis). Therefore, the classic

manifestations of excess aldosterone secretion are hypokalemia,

hypertension, alkalosis, and low renin levels.

TMP14 pp. 351–352, 384

65. B) Potassium secretion by the cortical collecting ducts is stimulated by (1)

aldosterone, (2) increased plasma potassium concentration, (3) increased

flow rate in the cortical collecting tubules, and (4) alkalosis. Therefore, a

diuretic that inhibits aldosterone, decreased plasma potassium

concentration, acute acidosis, and low sodium intake would all tend to

decrease potassium secretion by the cortical collecting tubules. A diuretic

that decreases loop of Henle sodium reabsorption, however, would tend to

increase the flow rate in the cortical collecting tubule and therefore

stimulate potassium secretion.

TMP14 pp. 386–389

66. B) This patient with diabetes mellitus and chronic renal disease has a

reduction in creatinine clearance to 40% of normal, implying a marked

reduction in GFR. He also has acidosis, as evidenced by a plasma pH of

7.14. The decrease in creatinine clearance would cause only a transient

reduction in sodium excretion and creatinine excretion rate. As the plasma

creatinine concentration increased, the urinary creatinine excretion rate

would return to normal, despite the sustained decrease in creatinine

clearance (creatinine excretion rate/plasma concentration of creatinine).

Diabetes is associated with increased production of acetoacetic acid, which

would cause metabolic acidosis and decreased plasma concentration,

as well as a compensatory increase in renal NH4

+

 production and

increased NH4

+

 excretion rate.

TMP14 pp. 417, 428–429

67. C) A reduction in plasma protein concentration to 3.6 g/dl would increase

the capillary filtration rate, thereby raising interstitial fluid volume and

interstitial fluid hydrostatic pressure. The increased interstitial fluid

pressure would, in turn, increase the lymph flow rate and reduce the

interstitial fluid protein concentration (“washout” of interstitial fluid

protein).

TMP14 pp. 316–319

68. B) The most likely diagnosis for this patient is diabetes insipidus, which

can account for the polyuria and the fact that her urine osmolarity is very

low (80 mOsm/l) despite overnight water restriction. In many patients

with diabetes insipidus, the plasma sodium concentration can be

maintained relatively close to normal by increasing fluid intake

(polydipsia). When water intake is restricted, however, the high urine flow

rate leads to rapid depletion of extracellular fluid volume and severe

hypernatremia, as occurred in this patient. The fact that she has no glucose

in her urine rules out diabetes mellitus. Neither primary aldosteronism nor

a renin-secreting tumor would lead to an inability to concentrate the urine

after overnight water restriction. Syndrome of inappropriate ADH would

cause excessive fluid retention and increased urine osmolarity.

TMP14 pp. 374–376, 379–380

69. C) The filterability of solutes in the plasma is inversely related to the size

of the solute (molecular weight). Also, positively charged molecules are

filtered more readily than are neutral molecules or negatively charged

molecules of equal molecular weight. Therefore, the positively charged

polycationic dextran with a molecular weight of 25,000 would be the most

readily filtered substance of the choices provided. Red blood cells are not

filtered at all by the glomerular capillaries under normal conditions.

TMP14 pp. 332–333

70. A) In normally functioning kidneys, approximately two thirds of the

water filtered by the glomerular capillaries is reabsorbed in the proximal

tubule. Although dehydration increases ADH levels and water

reabsorption by the distal tubules, collecting tubules, and collecting ducts

and this action contributes importantly to decreased water excretion in

dehydration, the total amount of water that remains in these tubular

segments is small compared with the amount of water in the proximal

tubules (see the figure below).

TMP14 pp. 372–373

71. D) Furosemide (Lasix) inhibits the Na+

-2Cl −

-K+

 co-transporter in the

ascending limb of the loop of Henle. This action not only causes marked

natriuresis and diuresis but also reduces the urine-concentrating ability.

Furosemide does not cause edema; in fact, it is often used to treat severe

edema and heart failure. Furosemide also increases the renal excretion of

potassium and calcium and therefore tends to cause hypokalemia and

hypocalcemia rather than increasing the plasma concentrations of

potassium and calcium.

TMP14 pp. 350–351, 388-389, 421–422

72. B) Hypernatremia can be caused by excessive sodium retention or water

loss. The fact that the patient has large volumes of dilute urine suggests

excessive urinary water excretion. Of the two possible disturbances listed

that could cause excessive urinary water excretion (nephrogenic diabetes

insipidus and central diabetes insipidus), nephrogenic diabetes insipidus

is the most likely cause. Central diabetes insipidus (decreased ADH

secretion) is not the correct answer because plasma ADH levels are

markedly elevated. Simple dehydration due to decreased water intake is

unlikely because the patient is excreting large volumes of dilute urine.

TMP14 pp. 315–316, 374

73. D) Dehydration due to water deprivation decreases extracellular fluid

volume, which in turn increases renin secretion and decreases plasma

atrial natriuretic peptide. Dehydration also increases the plasma sodium

concentration, which stimulates the secretion of ADH. The increased ADH

increases water permeability in the collecting ducts. The ascending limb of

the loop of Henle is relatively impermeable to water, and this low

permeability is not altered by water deprivation or increased levels of

ADH.

TMP14 pp. 374–376

74. C) High urine flow occurs in type 1 diabetes because the filtered load of

glucose exceeds the renal threshold, resulting in an increase in glucose

concentration in the tubule, which decreases the osmotic driving force for

water reabsorption. Increased urine flow reduces extracellular fluid

volume and stimulates the release of ADH.

TMP14 pp. 346–347, 377

75. C) Excess aldosterone and a high-salt diet could cause serious

hypokalemia because aldosterone stimulates potassium secretion by the

renal tubules (and therefore tends to increase potassium excretion), as well

as causing a shift of potassium from the extracellular fluid into the cells. A

high-salt diet would exacerbate the hypokalemia because this would

increase collecting tubular flow rate, which would tend to further increase

renal potassium secretion. Treatment with spironolactone or a betaadrenergic blocker or Addison disease (adrenal insufficiency) would tend

to increase plasma potassium concentration. Changes in sodium and

potassium intakes over the ranges indicated would have minimal effects

on plasma potassium concentration.

TMP14 pp. 384–388

76. D) Ingestion of excess aspirin (acetylsalicylic acid) would tend to cause

metabolic acidosis, which would lead to decreases in plasma ,

decreased (due to respiratory compensation), decreased urine

excretion and increased NH4

+

 excretion (renal compensation), and

increased anion gap due to increased unmeasured anions.

TMP14 pp. 417, 419–420

77. B) Approximately 30% to 40% of the filtered urea is reabsorbed in the

proximal tubule. However, the tubular fluid urea concentration increases

because urea is not nearly as permeant as water in this nephron segment.

Urea concentration increases further in the tip of the loop of Henle because

water is reabsorbed in the descending limb of the loop of Henle. Under

conditions of antidiuresis, urea is further concentrated as water is

reabsorbed and as fluid flows along the collecting ducts. Therefore, the

final urine concentration of urea is substantially greater than the

concentration in the proximal tubule or in the plasma.

TMP14 pp. 370–371

78. C) Diuretics that inhibit loop of Henle sodium reabsorption are used to

treat conditions associated with excessive fluid volume (e.g., hypertension

and heart failure). These diuretics initially cause an increase in sodium

excretion that reduces extracellular fluid volume and blood pressure, but

under steady-state conditions, the urinary sodium excretion returns to

normal, due in part to the fall in blood pressure. One of the important

adverse effects of loop diuretics is hypokalemia that is caused by the

inhibition of Na+

-2Cl −

-K+

 co-transport in the loop of Henle and by the

increased tubular flow rate in the cortical collecting tubules, which

stimulates potassium secretion.

TMP14 pp. 388–389, 421–422

79. A) A reduction in the number of functional nephrons to 25% of normal

would cause a compensatory increase in GFR and urine flow rate of the

surviving nephrons and decreased urine concentrating ability. Under

steady-state conditions, the urinary creatinine excretion rate and sodium

excretion rate would be maintained at normal levels. (For further

information, see TMP14, Table 32-6.)

TMP14 pp. 428–430

80. D) Approximately 40% to 50% of the filtered urea is reabsorbed in the

proximal tubule. The distal convoluted tubule and the cortical collecting

tubules are relatively impermeable to urea, even under conditions of

antidiuresis; therefore, lile urea reabsorption takes place in these

segments. Likewise, very lile urea reabsorption takes place in the thick

ascending limb of the loop of Henle. Under conditions of antidiuresis, the

concentration of urea in the renal medullary interstitial fluid is markedly

increased because of reabsorption of urea from the collecting ducts, which

contributes to the hyperosmotic renal medulla.

TMP14 pp. 370–371

81. B) Free water clearance is calculated as urine flow rate (600 ml/2 h, or 5

ml/min) − osmolar clearance (urine osmolarity × urine flow rate/plasma

osmolarity). Therefore, free water clearance is equal to +2.5 ml/min.

TMP14 p. 374

82. E) In the absence of ADH, the late distal tubule and collecting tubules are

not permeable to water. Therefore, the tubular fluid, which is already

dilute when it leaves the loop of Henle (∼ 100 mOsm/l), becomes further

diluted as it flows through the late distal tubule and collecting tubules as

electrolytes are reabsorbed. Therefore, the final urine osmolarity in the

complete absence of ADH is less than 100 mOsm/l.

TMP14 p. 372, Figure 29-8

83. A) About 65% of the filtered potassium is reabsorbed in the proximal

tubule, and another 20% to 30% is reabsorbed in the loop of Henle.

Although most of the daily variation in potassium excretion is caused by

changes in potassium secretion in the distal and collecting tubules, only a

small percentage of the filtered potassium load can be reabsorbed in these

nephron segments. (For further information, see TMP14, Figure 30-2.)

TMP14 pp. 384–385

84. A) The proximal tubule normally absorbs approximately 65% of the

filtered water, with much smaller percentages being reabsorbed in the

descending loop of Henle and in the distal and collecting tubules. The

ascending limb of the loop of Henle is relatively impermeable to water and

therefore reabsorbs very lile water.

TMP14 pp. 349–354

85. C) The thick ascending limb of the loop of Henle is relatively

impermeable to water even under conditions of maximal antidiuresis. The

proximal tubule and descending limb of the loop of Henle are highly

permeable to water under normal conditions, as well as during

antidiuresis. Water permeability of the late distal and collecting tubules

increases markedly during antidiuresis because of the effects of increased

levels of ADH.

TMP14 pp. 372–373

86. D) Interstitial fluid volume is equal to extracellular fluid volume minus

plasma volume. Extracellular fluid volume can be estimated from the

distribution of inulin or 22Na, whereas plasma volume can be estimated

from 125I-albumin distribution. Therefore, interstitial fluid volume is

calculated from the difference between the inulin distribution space and

the 125I-albumin distribution space.

TMP14 pp. 308–310, Table 25-3

87. F) Furosemide (Lasix) is a “loop” diuretic that inhibits the Na+

-Cl −

-K+

 cotransporter in the thick ascending loop of Henle, thus reducing urineconcentrating ability; increasing renal excretion of Na+

, Cl −

, and K+

; and

tending to cause hypokalemia.

TMP14 pp. 421–422

88. D) Uncontrolled type 1 diabetes would tend to cause metabolic acidosis

(decreases in plasma pH and ) due to increased metabolism of fat and

production of acetoacetic acid, which, in turn, would be associated with

increased anion gap. The normal respiratory compensation would

decrease plasma .

TMP14 p. 420

89. E) Metabolic alkalosis is associated with hypokalemia due to a shift of

potassium from the extracellular fluid into the cells (see table below). Betaadrenergic blockade, insulin deficiency, strenuous exercise, and

aldosterone deficiency all cause hyperkalemia due to a shift of potassium

out of the cells into the extracellular fluid.

TMP14 pp. 383–384, Table 5-1

Table 5-1 Factors That Can Alter Potassium Distribution Between Intracellular

and Extracellular Fluids

Factors That Shift K+ Into Cells (Decrease

Extracellular[K+

])

Factors That Shift K- Out of Cells (Increase

Extracellular[K+

])

Insulin Insulin deficiency (diabetes mellitus)

Aldosterone Aldosterone deficiency(Addison disease)

β-Adrenergic stimulation β-Adrenergic blockade

Alkalosis Acidosis

Cell lysis

Strenuous exercise

Increased extracellular fluid

osmolarity

90. D) In this example, the filtered load of glucose is equal to GFR (100

ml/min) × plasma glucose (150 mg/dl), or 150 mg/min. If there is no

detectable glucose in the urine, the reabsorption rate is equal to the filtered

load of glucose, or 150 mg/min.

TMP14 pp. 346–347, 361

91. B) Furosemide is a powerful inhibitor of the Na+

-2Cl −

-K+

 co-transporter

in the loop of Henle. Thiazide diuretics primarily inhibit NaCl

reabsorption into the distal tubule, whereas carbonic anhydrase inhibitors

decrease bicarbonate reabsorption in the tubules. Amiloride inhibits

sodium channel activity, whereas spironolactone inhibits the action of

mineralocorticoids in the renal tubules. Osmotic diuretics inhibit water

and solute reabsorption by increasing osmolarity of the tubular fluid.

TMP14 p. 422

92. E) Decreased efferent arteriolar resistance would increase renal blood

flow while reducing glomerular hydrostatic pressure, which, in turn,

would tend to decrease the GFR.

TMP14 pp. 335–336

93. E) Renal tubular acidosis results from a defect of renal secretion or H+

, a

defect in reabsorption of , or both. This defect causes metabolic

acidosis associated with decreases in plasma pH and and a normal

anion gap associated with hyperchloremia (increased plasma chloride

concentration). Plasma is reduced because of respiratory compensation

for the acidosis.

TMP14 p. 417

94. D) The patient has classic symptoms of diabetes mellitus: increased thirst,

breath smelling of acetone (due to increased acetoacetic acids in the blood),

high fasting blood glucose concentration, and glucose in the urine. The

acetoacetic acids in the blood cause metabolic acidosis that leads to a

compensatory decrease in renal excretion, decreased urine pH, and

increased renal production of ammonium and . The high level of

blood glucose increases the filtered load of glucose, which exceeds the

transport maximum for glucose, causing an osmotic diuresis (increased

urine volume) due to the unreabsorbed glucose in the renal tubules acts as

an osmotic diuretic.

TMP14 pp. 346–347, 417

95. C) Intracellular fluid volume is calculated as the difference between total

body fluid (0.57 × 60 g = 34.2 kg, or ∼34.2 l) and extracellular fluid volume

(12.8 l), which equals 21.4 l.

TMP14 pp. 309–310

96. C) Plasma volume is calculated as blood volume (4.3 l) × (1.0 −

hematocrit), which is 4.3 × 0.6 = 2.58 l (rounded up to 2.6).

TMP14 pp. 309–310

97. C) Interstitial fluid volume is calculated as the difference between

extracellular fluid volume (12.8 l) and plasma volume (2.6 l), which is

equal to 10.2 l.

TMP14 pp. 309-310

98. C) The primary site of reabsorption of magnesium is in the loop of Henle,

where about 65% of the filtered load of magnesium is reabsorbed. The

proximal tubule normally reabsorbs only about 25% of filtered

magnesium, and the distal and collecting tubules reabsorb less than 5%.

TMP14 p. 392

99. C) A plasma glucose concentration of 300 mg/dl would increase the

filtered load of glucose above the renal tubular transport maximum and

therefore increase urinary glucose excretion. The unreabsorbed glucose in

the renal tubules would also cause an osmotic diuresis, increased urine

volume, and decreased extracellular fluid volume, which would stimulate

thirst. Increased glucose also causes vasodilatation of afferent arterioles,

which increases GFR.

TMP14 pp. 341–342, 346–347, 378–379

100. B) GFR is approximately equal to the clearance of creatinine. Creatinine

clearance = urine creatinine concentration (32 mg/dl) × urine flow rate

(3600 ml/24 h, or 2.5 ml/min) ÷ plasma creatinine concentration (4 mg/dl) =

20 ml/min.

TMP14 pp. 361–362

101. D) The net renal tubular reabsorption rate is the difference between the

filtered load of potassium (GFR × plasma potassium concentration) and the

urinary excretion of potassium (urine potassium concentration × urine

flow rate). Therefore, the net tubular reabsorption of potassium is 0.075

mmol/min.

TMP14 pp. 361–362

102. D) Severe diarrhea would result in loss of in the stool, thereby

causing metabolic acidosis that is characterized by low plasma and

low pH. Respiratory compensation would reduce . The plasma anion

gap would be normal, and the plasma chloride concentration would be

elevated (hyperchloremic metabolic acidosis) in metabolic acidosis caused

by loss in the stool.

TMP14 pp. 415–420

103. E) Primary excessive secretion of aldosterone causes metabolic alkalosis

due to increased secretion of hydrogen ions and reabsorption by the

intercalated cells of the collecting tubules. Therefore, the metabolic

alkalosis would be associated with increases in plasma pH and , with

a compensatory reduction in respiration rate and increased . The

plasma anion gap would be normal, with a slight reduction in plasma

chloride concentration.

TMP14 pp. 417–420

104. D) Proximal tubular acidosis results from a defect of renal secretion of

hydrogen ions, reabsorption of bicarbonate, or both. This defect leads to

increased renal excretion of and metabolic acidosis characterized by

low plasma concentration, low plasma pH, a compensatory increase

in respiration rate and low , and a normal anion gap with an increased

plasma chloride concentration.

TMP14 pp. 415–420

105. F) A patient with diabetic ketoacidosis and emphysema would be

expected to have metabolic acidosis (due to excess ketoacids in the blood

caused by diabetes), as well as increased plasma due to impaired

pulmonary function. Therefore, the patient would be expected to have

decreased plasma pH, decreased , increased , and an increased

anion gap due to the addition of ketoacids to the

blood.

TMP14 pp. 416–420

106. D) Secretion of hydrogen ions and reabsorption of depend critically

on the presence of carbonic anhydrase in the renal tubules. After inhibition

of carbonic anhydrase, renal tubular secretion of hydrogen ions and

reabsorption of would decrease, leading to increased renal excretion

of , reduced plasma concentration, and metabolic acidosis. The

metabolic acidosis, in turn, would stimulate the respiration rate, leading to

decreased . The plasma anion gap would be within the normal range.

TMP14 pp. 410–411, 420, 422

107. B) Acute renal failure caused by tubular necrosis would cause the rapid

development of metabolic acidosis due to the kidneys’ failure to rid the

body of the acid waste products of metabolism. The metabolic acidosis

would lead to decreased plasma concentration. Acute renal failure

would also lead to a rapid increase in blood urea nitrogen concentration

and a significant increase in plasma potassium concentration due to the

kidneys’ failure to excrete electrolytes or nitrogenous waste products.

Necrosis of the renal epithelial cells causes them to slough away from the

basement membrane and plug up the renal tubules, thereby increasing

hydrostatic pressure in Bowman’s capsule and decreasing GFR.

TMP14 pp. 425, 430

108. E) Intracellular and extracellular body fluids have the same total

osmolarity under steady-state conditions because the cell membrane is

highly permeable to water. Therefore, water flows rapidly across the cell

membrane until osmotic equilibrium is achieved. The colloid osmotic

pressure is determined by the protein concentration, which is considerably

higher inside the cell. The cell membrane is also relatively impermeable to

potassium, sodium, and chloride, and active transport mechanisms

maintain low intracellular concentrations of sodium and chloride and a

high intracellular concentration of potassium.

TMP14 pp. 310–311

109. B) Fluid entering the early distal tubule is almost always hypotonic

because sodium and other ions are actively transported out of the thick

ascending loop of Henle, whereas this portion of the nephron is virtually

impermeable to water. For this reason, the thick ascending limb of the loop

of Henle and the early part of the distal tubule are often called the diluting

segment.

TMP14 pp. 350–351

110. D) Chronic metabolic acidosis is, by definition, associated with

decreased plasma concentration. Decreased excretion of NH4Cl and

occurs with renal compensation for the acidosis, and respiratory

compensation for the acidosis increases the ventilation rate, resulting in

decreased plasma .

TMP14 pp. 415-418

111. B) A 0.45% NaCl solution is hypotonic. Therefore, administration of 2.0 l

of this solution would reduce intracellular and extracellular fluid

osmolarity and cause increases in intracellular and extracellular volumes.

TMP14 pp. 312-313

112. C) If the renal clearance is greater than the GFR, this implies that there

must be secretion of that substance into the renal tubules. A substance that

is freely filtered and not secreted or reabsorbed would have a renal

clearance equal to the GFR.

TMP14 p. 364

113. A) In the proximal tubule, calcium reabsorption usually parallels sodium

and water reabsorption. With extracellular volume expansion or increased

blood pressure, proximal sodium and water reabsorption are reduced, and

a reduction in calcium reabsorption also occurs, causing increased urinary

excretion of calcium. Increased parathyroid hormone, increased plasma

phosphate concentration, and metabolic alkalosis all tend to decrease the

renal excretion of calcium.

TMP14 pp. 390–391

114. C) Increasing sodium intake would decrease renin secretion and plasma

renin activity, as well as reduce plasma aldosterone concentration and

increase plasma atrial natriuretic peptide because of a mod est expansion

of extracellular fluid volume. Although a high sodium intake would

initially increase distal NaCl delivery, which would tend to increase

potassium excretion, the decrease in aldosterone concentration would

offset this effect, resulting in no change in potassium excretion under

steady-state conditions. Even very large increases in sodium intake cause

only minimal changes in plasma sodium concentration as long as the

ADH–thirst mechanisms are fully operative.

TMP14 pp. 388–389

115. C) A 50% reduction in efferent arteriolar resistance would cause a large

decrease in GFR—greater than 10%. A decrease in renal artery pressure

from 100 to 85 mm Hg would cause only a slight decrease in GFR in a

normal, autoregulating kidney. A decrease in afferent arteriole resistance,

a decrease in plasma colloid osmotic pressure, or an increase in the

glomerular capillary filtration coefficient would all tend to increase GFR.

TMP14 pp. 333–336, 338–339

116. D) Acute metabolic acidosis reduces intracellular potassium

concentration, which, in turn, decreases potassium secretion by the

principal cells of the collecting tubules. The primary mechanism by which

increased hydrogen ion concentration inhibits potassium secretion is by

reducing the activity of the sodium-potassium adenosine triphosphatase

pump. This action then reduces intracellular potassium concentration,

which, in turn, decreases the rate of passive diffusion of potassium across

the luminal membrane into the tubule.

TMP14 p. 389

117. D) In a dehydrated person, osmolarity in the early distal tubule is

usually less than 300 mOsm/l because the ascending limb of the loop of

Henle and the early distal tubule are relatively impermeable to water, even

in the presence of ADH. Therefore, the tubular fluid becomes

progressively more dilute in these segments compared with plasma. ADH

does not influence water reabsorption in the ascending limb of the loop of

Henle. The ascending limb, however, reabsorbs sodium to a much greater

extent than does the descending limb. Another important action of ADH is

to increase the urea permeability in the medullary collecting ducts, which

contributes to the hyperosmotic renal medullary interstitium in

antidiuresis.

TMP14 pp. 372–373

118. E) Most potassium secretion occurs in the collecting tubules. A highpotassium diet stimulates potassium secretion by the collecting tubules

through multiple mechanisms, including small increases in extracellular

potassium concentration, as well as increased levels of aldosterone.

TMP14 pp. 386–387

119. A) Diabetic ketoacidosis results in a metabolic acidosis that is

characterized by a decrease in plasma bicarbonate concentration, increased

anion gap (due to the addition of unmeasured anions to the extracellular

fluid along with the ketoacids), and a renal compensatory response that

includes increased secretion of NH4

+

. There is also an increased

respiratory rate with a reduction in arterial , as well as decreased urine

pH and decreased renal excretion.

TMP14 pp. 415–420

120. B) Interstitial fluid volume cannot be measured directly, but it can be

calculated as the difference between extracellular fluid volume (inulin

space = 16 l) and plasma volume (125I-albumin space = 4 l). Therefore,

interstitial fluid volume is approximately 12 l.

TMP14 pp. 308–310

121. E) Increased levels of insulin cause a shift of potassium from the

extracellular fluid into the cells. All the other conditions have the reverse

effect of shifting potassium out of the cells into the extracellular fluid.

TMP14 pp. 383–384

122. D) This patient is severely dehydrated as a result of sweating and lack of

adequate fluid intake. The dehydration markedly stimulates the release of

ADH and renin secretion, which in turn stimulates the formation of

angiotensin II and aldosterone secretion.

TMP14 pp. 359, 375

123. E) After running the race and losing both fluid and electrolytes, this

person replaces his fluid volume by drinking 2 l of water. However, he did

not replace the electrolytes. Therefore, he would be expected to experience

a decrease in plasma sodium concentration, resulting in a decrease in both

intracellular and extracellular fluid osmolarity. The decrease in

extracellular fluid osmolarity would lead to an increase in intracellular

volume as fluid diffused into the cells from the extracellular compartment.

Therefore, after drinking the water and absorbing it, the total body volume

would be normal but intracellular volume would be increased and

extracellular volume would be reduced.

TMP14 pp. 312–314

124. B) Increased levels of parathyroid hormone stimulate calcium

reabsorption in the thick ascending loops of Henle and distal tubules.

Extracellular fluid volume expansion, increased blood pressure, decreased

plasma phosphate concentration, and metabolic acidosis are all associated

with decreased calcium reabsorption by the renal tubules.

TMP14 pp. 390–391

125. C) In this example, the acidosis is associated with a reduced plasma

bicarbonate concentration, signifying metabolic acidosis. In addition, the

patient also has an elevated , signifying respiratory acidosis. Therefore,

the patient has simultaneous respiratory and metabolic acidosis.

TMP14 pp. 415–420

126. A) Chronic respiratory acidosis is caused by insufficient pulmonary

ventilation, resulting in an increase in . Acidosis, in turn, stimulates the

secretion of hydrogen ions into the tubular fluid and increased renal

tubular production of NH4

+

, which further contributes to the excretion of

hydrogen ions and the renal production of , thereby increasing

plasma bicarbonate concentration. The increased tubular secretion of

hydrogen ions also reduces urine pH.

TMP14 pp. 415–416

127. A) Dilation of the afferent arterioles leads to an increase in the

glomerular hydrostatic pressure and therefore an increase in GFR, as well

as an increase in renal blood flow. Increased glomerular capillary filtration

coefficient would also raise the GFR but would not be expected to alter

renal blood flow. Increased plasma colloid osmotic pressure or dilation of

the efferent arterioles would both tend to reduce the GFR. Increased blood

viscosity would tend to reduce renal blood flow and GFR.

TMP14 pp. 333–337

128. B) Nonsteroidal antiinflammatory drugs inhibit the synthesis of

prostaglandins, which, in turn, causes constriction of afferent arterioles

that can reduce the GFR. The decrease in GFR, in turn, leads to an increase

in serum creatinine. Increased efferent arteriole resistance and increased

glomerular capillary filtration coefficient would both tend to increase

rather than reduce GFR. Increasing muscle mass due to exercise would

cause very lile change in serum creatinine.

TMP14 pp. 333–338

129. A) In this example, the plasma sodium concentration is markedly

increased but the urine sodium concentration is relatively normal, and

urine osmolarity is almost maximally increased to 1200 mOsm/l. In

addition, there are increases in plasma renin, ADH, and aldosterone,

which is consistent with dehydration caused by decreased fluid intake.

The syndrome of inappropriate ADH would result in a decrease in plasma

sodium concentration, as well as suppression of renin and aldosterone

secretion. Nephrogenic diabetes insipidus, caused by the kidneys’ failure

to respond to ADH, would also be associated with dehydration, but urine

osmolarity would be reduced rather than increased. Primary

aldosteronism would tend to cause sodium and water retention with only

a modest change in plasma sodium concentration and a marked reduction

in the secretion of renin. Likewise, a renin-secreting tumor would be

associated with increases in plasma aldosterone concentration and plasma

renin activity but only a modest change in plasma sodium concentration.

TMP14 pp. 375–376, 378–380


Unit VI: Blood Cells, Immunity,

and Blood Coagulation

The following table of normal test values can be referenced

throughout Unit VI.

Test Normal Values

Erythrocyte count Male: 4.3-5.9 million/µl

Female: 3.5-5.5 million/µl

Hematocrit Male: 41%-53%

Female: 36%-46%

Hemoglobin, blood Male: 13.5-17.5 g/dl

Female: 12.0-16.0 g/dl

Mean corpuscular hemoglobin 25.4-34.6 pg/cell

Mean corpuscular hemoglobin concentration 31%-36% hemoglobin

Mean corpuscular volume

80-100 fl

Reticulocyte count 0.5%-1.5% of red blood cells

Platelet count 150,000-400,000/µl

Leukocyte count and differential

Leukocyte count 4500-11,000/µl

Neutrophils 54%-62%

Eosinophils 1%-3%

Basophils 0-0.75%

Lymphocytes 25%-33%

Monocytes 3%-7%

Partial thromboplastin time (activated) 25-40 seconds

Prothrombin time

11-15 seconds

Bleeding time

2-7 minutes

1. A 40-year-old woman visits the clinic complaining of fatigue.

She had recently been treated for an infection. Her laboratory

values are as follows: red blood cell (RBC) count, 1.8 × 106

/µl;

hemoglobin (Hb), 5.2 g/dl; hematocrit (Hct), 15; white blood

cell (WBC) count, 7.6 × 103

/µl; platelet count, 320,000/µl;

mean corpuscular volume (MCV), 92 fL; and reticulocyte

count, 24%. What is the most likely explanation for this

presentation?

A) Aplastic anemia

B) Hemolytic anemia

C) Hereditary spherocytosis

D) B12

 deficiency

2. What RBC enzyme facilitates transport of carbon dioxide

(CO2

)?

A) Myeloperoxidase

B) Carbonic anhydrase

C) Superoxide dismutase

D) Globin reductase

3. A patient presents with hemoglobin of 7.9 g/dl, hematocrit of

23%, and mean corpuscular volume of 89 fl. Erythropoietin

level is quite low. What is the most likely diagnosis?

A) Sickle cell anemia

B) Pernicious anemia

C) End-stage renal disease

D) Chronic blood loss

4. A patient has a hematocrit of 63% and a reduced

erythropoietin level. Based on this information alone, what is

the most likely diagnosis?

A) Prolonged and continued exposure to high altitude

B) Polycythemia vera

C) Chronic obstructive pulmonary disease with hypoxemia

D) Hemochromatosis

5. A 74-year-old man has a hemoglobin of 6.0 g/dl, hematocrit of

19%, mean corpuscular volume of 117, and large, bizarrely

shaped erythrocytes on peripheral blood smear. His serum

folic acid levels are normal. What is the likely cause of this

disorder?

A) Insufficient intrinsic factor

B) Vitamin K deficiency

C) Hepatic cirrhosis

D) Alpha thalassemia

6. When red blood cells break down, the porphyrin portion of

hemoglobin is converted by macrophages to what

product(s)?

A) Xanthine

B) Branched-chain amino acids

C) Homocysteine

D) Bilirubin

7. How many oxygen atoms can be transported by each

hemoglobin molecule?

A) 2

B) 4

C) 8

D) 16

8. During the second trimester of pregnancy, where is the

predominant site of RBC production in the embryo?

A) Yolk sac

B) Bone marrow

C) Lymph nodes

D) Liver

9. What function do vitamin B12

 and folic acid perform that is

critical to hematopoiesis?

A) Support porphyrin production

B) Serve as cofactors for iron uptake

C) Support terminal differentiation of erythroid and

myeloid cells

D) Support production of thymidine triphosphate

10. A 62-year-old man complains of headaches, visual

difficulties, and chest pains. Physical examination reveals a

red complexion and a large spleen. His complete blood cell

count (CBC) is as follows: Hct, 58%; WBC, 13,300/µl; and

platelets, 600,000/µl. His arterial oxygen saturation is 97% on

room air. Which treatment would you recommend?

A) Chemotherapy

B) Phlebotomy

C) Iron supplement

D) Inhaled oxygen therapy

11. A 38-year-old healthy woman comes to you for a routine

visit. She has spent the past 2 months hiking through the

Himalayas and climbed to the base camp of Mount Everest.

Which results would you expect to see on her CBC?

Hematocrit RBC count WBC count MCV

A) ↑ ↑ ↑ ↑

B) ↑ ↑ ↔ ↑

C) ↑ ↑ ↔ ↔

D) ↑ ↔ ↔ ↔

E) ↔ ↑ ↑ ↔

F) ↑ ↔ ↑ ↑

G) ↔ ↑ ↔ ↑

12. A 24-year-old African American man comes to the

emergency department 3 hours after the onset of severe back

and chest pain. These problems started while he was walking

briskly through a hilly neighborhood. He has had similar

episodes numerous times in the past. He is in obvious pain.

Laboratory studies show the following values:

Hemoglobin = 11 g/dl

Leukocyte count = 22,000/µl3

Reticulocyte count = 3.5%

What is this patient’s diagnosis?

A) Acute blood loss

B) Sickle cell anemia

C) Anemia of chronic disease

D) End-stage renal disease

13. After a person is placed in an atmosphere with low oxygen,

how long does it take for increased numbers of reticulocytes

to develop?

A) 6 hours

B) 12 hours

C) 3 days

D) 5 days

E) 2 weeks

14. A patient presents to your office complaining of extreme

fatigue and shortness of breath on exertion that has gradually

worsened during the past 2 weeks. Physical examination

reveals a well-nourished woman who appears comfortable

but somewhat short of breath. Her vital signs include a pulse

of 120, a respiratory rate of 20, and blood pressure of 120/70.

When she stands up, her pulse increases to 150, and her

blood pressure falls to 80/50. Her hematologic values are as

follows: Hb, 7 g/dl; Hct, 20%; RBC count, 2 × 106

/µl; and

platelet count, 400,000/µl. On a peripheral smear, her RBCs

are microcytic and hypochromic. What is your diagnosis?

A) Aplastic anemia

B) Renal failure

C) Chronic and acute blood loss with iron-deficiency

anemia

D) Sickle cell anemia

E) Megaloblastic anemia

15. Which phagocytes can extrude digestion products and

continue to survive and function for many months?

A) Neutrophils

B) Basophils

C) Macrophages

D) Eosinophils

16. What cells ingest foreign material and micro-organisms

within the sinusoids of the liver?

A) Dendritic cells

B) Megakaryocytes

C) Basophils

D) Kupffer cells

17. A 45-year-old man presents to the emergency department

with a 2-week history of diarrhea that has goen

progressively worse during the past several days. He has

minimal urine output and is admied to the hospital for

dehydration. His stool specimen is positive for parasitic eggs.

Which type of WBC would have an elevated number?

A) Eosinophils

B) Neutrophils

C) T lymphocytes

D) B lymphocytes

E) Monocytes

18. A 24-year-old man came to the emergency department with

a broken leg. A blood test revealed his WBC count to be 22 ×

103

/µl. Five hours later, a second blood test revealed a WBC

count of 7 × 103

/µl. What is the cause of the increased WBC

count in the first test?

A) Increased production of WBCs by the bone marrow

B) Release of preformed, mature WBCs into the circulation

C) Decreased destruction of WBCs

D) Increased production of selectins

19. A 62-year-old man who was known to have a normal blood

cell count and differential count 3 months ago presents with

pallor, bone pain, bruising, and a WBC count of 42,000.

Eighty-five percent of cells in the circulation appear to be

immature granulocytes. What is the diagnosis?

A) Acute lymphocytic leukemia

B) Acute myelocytic leukemia

C) Chronic lymphocytic leukemia

D) Chronic myelocytic leukemia

20. What cell type can interact with IgE and release large

amounts of histamine, bradykinin, serotonin, heparin,

lysosomal enzymes, and other inflammatory mediators?

A) Neutrophils

B) Basophils and mast cells

C) Eosinophils

D) Monocytes

y

21. A 60-year-old man is noted on a routine physical

examination to have a significantly elevated white blood cell

(WBC) count of 22,000/µl3

, and 80% of cells are matureappearing cells with large, round nuclei and scant cytoplasm.

A review of the record shows that the WBC count has been

moderately elevated for at least 18 months. What is the likely

diagnosis?

A) Acute lymphocytic leukemia

B) Chronic lymphocytic leukemia

C) Acute myelocytic leukemia

D) Chronic myelocytic leukemia

22. Where does the transmigration of WBCs occur in response

to infectious agents?

A) Arterioles

B) Lymphatic ducts

C) Capillaries and venules

D) Inflamed arteries

23. An 8-year-old boy frequently comes to the clinic for

persistent skin infections that do not heal within a normal

time frame. He had a normal recovery from the measles. A

check of his antibodies after immunizations yielded normal

antibody responses. A defect in which of the following cells

would most likely be the cause of the frequent and prolonged

infections?

A) B lymphocytes

B) Plasma cells

C) Neutrophils

D) Macrophages

E) CD4 T lymphocytes

24. Which cell type migrates into inflammatory sites to clean up

necrotic tissue and direct tissue remodeling?

A) Neutrophil

B) Macrophage

C) Dendritic cell

D) Eosinophil

25. A 3-year-old child who has had frequent ear infections is

found to have reduced immunoglobulin levels and is

unresponsive to vaccination with tetanus toxoid. However,

the child has normal skin test reactivity (delayed redness and

induration) to a common environmental antigen. Which cell

lineage is not functioning normally?

A) Macrophages

B) Helper T cells

C) Cytotoxic T cells

D) B cells

26. Patients with human immunodeficiency virus (HIV) exhibit

abnormal functioning of which of the following mechanisms?

A) Antibody production only

B) T cell–mediated cytotoxicity only

C) Degranulation of appropriately stimulated mast cells

D) Both antibody production and T cell–mediated

cytotoxicity

27. What is the term for binding of IgG and complement to an

invading microbe to facilitate recognition?

A) Chemokinesis

B) Opsonization

C) Phagolysosome fusion

D) Signal transduction

28. Presentation by a cell of a foreign antigen bound to major

histocompatibility complex (MHC)-I will result in which of

the following?

A) Generation of antibodies

B) Activation of cytotoxic T cells

C) Increase in phagocytosis

D) Release of histamine by mast cells

29. The variable and hypervariable regions of immunoglobulins

that are responsible for antigen binding and specificity reside

in which part(s) of the immunoglobulin molecule?

A) The heavy chains

B) The light chains

C) The hinge regions

g g

D) Both the heavy chains and the light chains

E) The FC region

30. Fluid exudation into the tissue in an acute inflammatory

reaction is due to which of the following?

A) Decreased blood pressure

B) Decreased protein in the interstitium

C) Obstruction of the lymph vessels

D) Increased cloing factors

E) Increased vascular permeability

31. Which immune molecules can bind intact antigen?

A) T cell receptors

B) MHC class I

C) MHC class II

D) Immunoglobulins

32. What mechanisms are primarily responsible for producing a

lip ulcer from herpes simplex virus?

A) Activation of bradykinin by antibodies directed against

herpes antigens

B) Rapid influx of neutrophils and dendritic cells in

response to virally-derived chemotactic factors

C) Presentation of viral antigens by MHC class I and

engagement by cytotoxic T cells

D) Presentation of viral antigens by MHC class II and

engagement by cytotoxic T cells

33. What is the function of IL-2 in the immune response?

A) Binds to and presents antigen

B) Stimulates proliferation of T cells

C) Kills virus-infected cells

D) Is required for an anaphylactic response

34. Which of the following is true about helper T cells?

A) They are activated by the presentation of antigen by an

infected cell

B) They require the presence of a competent B-cell system

C) They destroy bacteria by phagocytosis

D) They are activated by the presentation of antigen by

macrophages or dendritic cells

p g

35. Which of the following applies to cytotoxic T cells?

A) They require the presence of a competent B-lymphocyte

system

B) They require the presence of a competent suppressor Tlymphocyte system

C) They are activated by the presentation of foreign

antigen by an infected cell

D) They destroy bacteria by initiating macrophage

phagocytosis

36. A 9-year-old girl has nasal discharge and itching of the eyes

in the spring every year. An allergist performs a skin test

using a mixture of grass pollens. Within a few minutes, the

girl exhibits a focal redness and swelling at the test site. This

response is most likely due to

A) Antigen–antibody complexes being formed in blood

vessels in the skin

B) Activation of neutrophils due to injected antigens

C) Activation of CD4 helper cells and the resultant

generation of specific antibodies

D) Activation of cytotoxic T lymphocytes to destroy

antigens

37. Activation of the complement system results in which

action?

A) Binding of the invading microbe with IgG

B) Inactivation of eosinophils

C) Decreased tissue levels of complement

D) Generation of chemotactic substances

38. Which statement is true concerning erythroblastosis fetalis

(hemolytic disease of the newborn [HDN])?

A) HDN occurs when an Rh-positive mother has an Rhnegative child

B) HDN is prevented by giving the mother a blood

transfusion

C) A complete blood transfusion after the first birth will

prevent HDN

D) The father of the child must be Rh positive

p

39. A couple requests blood typing of a newborn male (man is

AB, Rh-positive; woman is A, Rh-negative).

Hemagglutination assays show that the child is type O, Rhpositive. Which of the following conclusions regarding the

child’s parentage is valid?

A) The child could be the natural offspring of this couple

B) The mother could be the natural mother, but the father

could not be the natural father

C) The father could be the natural father, but the mother

could not be the natural mother

D) Neither the father nor the mother could be the natural

parents

40. A woman whose blood type is A, Rh positive, and a man

whose blood type is B, Rh positive, come to the clinic with a

3-year-old girl whose blood type is O, Rh negative. What can

be said about the relationship of these two adults to this

child?

A) The woman could be the child’s natural mother, but the

man could not be the natural father

B) The man could be the child’s natural father, but the

woman could not be the natural mother

C) Neither adult could be the natural parent of this child

D) This couple could be the natural parents of this child

41. What is the appropriate treatment for an infant born with

severe HDN (erythroblastosis fetalis)?

A) Passive immunization with anti-Rh(D) immunoglobulin

B) Immunization with Rh(D) antigen

C) Exchange transfusion with Rh(D)-positive blood

D) Exchange transfusion with Rh(D)-negative blood

42. Chronic allograft rejection results primarily from the actions

of what effector cell type?

A) Activated macrophages

B) Helper T lymphocytes

C) Cytotoxic T lymphocytes

D) Dendritic cells

43. Which of the following transfusions will result in an

immediate transfusion reaction?

A) O Rh-negative whole blood to an O Rh-positive patient

B) A Rh-negative whole blood to a B Rh-negative patient

C) AB Rh-negative whole blood to an AB Rh-positive

patient

D) B Rh-negative whole blood to a B Rh-negative patient

44. Given optimal transplantation technique, the following

types of grafts may be expected to succeed, with long-term

graft acceptance.

A) Allografts and xenografts

B) Autografts and allografts

C) Isografts and autografts

D) Allografts and isografts

E) Grafts with at least a 3/6 HLA match

45. After trauma, a person whose blood type is AB, Rh negative

and who has never had a transfusion before receives four

units of appropriately cross-matched type A, Rh positive

blood. What is the likely outcome?

A) Fever, chills, shortness of breath, circulatory shock, and

renal shutdown

B) Rapid hemolysis of the transfused cells

C) Moderate, intracellular hemolysis over a period of a few

weeks

D) Anaphylactic shock

46. Which transfusion will result in an immediate transfusion

reaction? Assume that the patient has never had a

transfusion.

A) Type O Rh-negative packed cells to an AB Rh-positive

patient

B) Type A Rh-positive packed cells to an A Rh-negative

patient

C) Type AB Rh-positive packed cells to an AB Rh-positive

patient

D) Type A Rh-positive packed cells to an O Rh-positive

patient

p

47. Which antigens must be matched optimally between donors

and recipients of solid organ transplants?

A) Class I human leukocyte antigen (HLA) antigens only

B) Class II HLA antigens only

C) Class I and Class II HLA antigens only

D) Class I and Class II HLA antigens and ABO antigens

48. A woman whose blood type is A positive and who has

always been healthy just delivered her second child. The

father’s blood type is O negative. Because the child’s blood

type is O negative (O, Rh negative), what would you expect

to find in this child?

A) Erythroblastosis fetalis due to rhesus incompatibility

B) Erythroblastosis fetalis due to ABO blood group

incompatibility

C) Both A and B

D) The child would not be expected to have HDN

49. A 55-year-old man who has been undergoing stable and

successful anticoagulation with warfarin for recurrent deep

vein thrombosis is treated for pneumonia, and 8 days later he

presents with lower intestinal bleeding. His prothrombin

time is quite prolonged. What is the appropriate therapy?

A) Treatment with tissue plasminogen activator

B) Infusion of calcium citrate

C) Treatment with fresh-frozen plasma and vitamin K

D) Rapid infusion of protamine

50. A 2-year-old boy bleeds excessively from minor injuries and

has previously had bleeding gums. The maternal grandfather

has a bleeding disorder. The child’s physical examination

shows slight tenderness of his knee with fluid accumulation

in the knee joint. You suspect this patient is deficient in

which coagulation factor(s)?

A) Prothrombin activator

B) Factor II

C) Factor VIII or Factor IX

D) Factor X

51. A Viet Nam veteran with chronic, severe hepatitis C has

developed an intractable nosebleed. His platelet count is

180,000, and his international normalized ratio (INR) is

markedly increased at 7.0. In addition to local compression at

the bleeding site what is the appropriate therapy?

A) Coumarin

B) Fresh-frozen plasma

C) Platelet transfusion

D) Fibrinogen infusion

52. Which agent is not effective as an in vitro anticoagulant?

A) Heparin

B) Warfarin (Coumadin)

C) Ethylenediamine tetraacetic acid (EDTA)

D) Sodium citrate

53. Hemophilia A affects which pathway of blood coagulation?

A) Intrinsic pathway only

B) Extrinsic pathway only

C) Intrinsic and extrinsic pathways only

D) Intrinsic, extrinsic, and common pathways

54. A homeless person is brought in from a vacant lot with signs

of sepsis and with persistent bleeding from his gums,

phlebotomy sites, and around IV catheters. His white blood

cell count is 15,000 with normal cellular morphology and a

few immature neutrophils (band cells), and his platelet count

is somewhat reduced at 95,000. What is the most likely

diagnosis?

A) Reduced fibrinogen concentration due to liver disease

B) Inhibition of platelet function by methyl alcohol

ingestion

C) Dietary deficiency of vitamin B12 and folic acid

D) Disseminated intravascular coagulation

55. Which coagulation pathway begins with tissue

thromboplastin?

A) Extrinsic pathway

B) Intrinsic pathway

C) Common pathway

p y

D) Fibrin stabilization

56. A 56-year-old electrician has severe but undiagnosed gluten

sensitivity with persistent diarrhea and weight loss. He has

excessive bleeding after a dental extraction and is found to

have a markedly prolonged prothrombin time of 37 seconds.

What is the most likely cause of his excessive bleeding?

A) Hepatocellular disease

B) Vitamin K deficiency

C) Protein malnutrition with inadequate fibrinogen

D) Loss of cloing factors through the gastrointestinal

mucosa

57. Which of the following would best explain a prolonged

bleeding time test?

A) Hemophilia A

B) Hemophilia B

C) Thrombocytopenia

D) Coumadin use

58. Which of the following is appropriate therapy for a massive

pulmonary embolism?

A) Citrated plasma

B) Warfarin

C) Aspirin

D) Tissue plasminogen activator

59. What is the primary mechanism by which heparin prevents

blood coagulation?

A) Antithrombin III activation

B) Binding and inhibition of tissue factor

C) Binding available calcium

D) Inhibition of platelet-activating factor

60. A 14-year-old boy has a life long history of excessive

bleeding, often involving bleeding into his joints

(hemarthrosis). There is a positive history of a similar

disorder in several male relatives. You may assume that he

inherited this disorder from which parent(s)?

A) His father

B) His mother

C) Both parents

D) Penetrance of this disorder is variable and difficult to

interpret

Answers

1. B) This patient has increased production of RBCs as indicated by a

markedly increased reticulocyte count in the seing of significant

anemia (low red blood cell number, hemoglobin, and hematocrit).

The RBCs being produced have a normal size (MCV = 90), and

thus the patient does not have spherocytosis (small RBCs) or

vitamin B12 deficiency (large RBCs). The normal WBC count and

the increased reticulocyte count suggest that the bone marrow is

functioning. The increased reticulocyte count means that a large

number of RBCs are being produced. These laboratory values

support an anemia due to some type of blood loss—in this case an

anemia due to hemolysis.

TMP14 pp. 440–443, 446

2. B) Carbonic anhydrase catalyzes the reaction of CO2

 with water to

allow large amounts of CO2

 to be transported in blood as soluble

bicarbonate ion.

TMP14 p. 439

3. C) In end-stage renal disease, the kidneys cease to make

erythropoietin, leading to severe anemia. The types of anemia in

the other three possible answers lead to red blood cell loss or

destruction and increased erythropoietin due to tissue hypoxemia.

TMP14 p. 442

4. B) In polycythemia vera autonomous, clonal production of red

blood cells results in increased hemoglobin, hematocrit, and red

blood cell count and decreased erythropoietin, because hypoxemic

drive for “epo” production is reduced. High-altitude exposure and

chronic obstructive pulmonary disease are both associated with

tissue hypoxemia that drives increased erythropoietin production.

Hemochromatosis is a disorder of systemic iron overload with

normal red blood cell production.

TMP14 pp. 442, 447

5. A) The clinical presentation is that of a megaloblastic anemia,

which can be caused by a deficiency of either vitamin B12 or folic

acid. Because this patient’s folic acid levels are normal, the cause

must be vitamin B12 deficiency caused by atrophy of the gastric

mucosa resulting in insufficient production of intrinsic factor,

needed for normal gastrointestinal absorption of vitamin B12.

TMP14 p. 443

6. D) Macrophages convert the porphyrin portion of hemoglobin to

bilirubin.

TMP14 p. 445

7. C) Each hemoglobin molecule has four globin chains (in

hemoglobin A, the predominant form in adults, the hemoglobin

molecule includes two alpha and two beta chains). Each globin

chain is associated with one heme group, containing one atom of

iron. Each of the four iron atoms can bind loosely with one

molecule (two atoms) of oxygen. Thus, each hemoglobin molecule

can transport eight oxygen atoms.

TMP14 p. 444

8. D) RBC production begins in the yolk sac for the first trimester.

Production in the yolk sac decreases at the beginning of the

second trimester, and the liver becomes the predominant source of

RBC production. RBC production from the bone marrow increases

during the third trimester and continues throughout life.

TMP14 pp. 439–440

9. D) Cell proliferation requires DNA replication, which requires an

adequate supply of thymidine triphosphate. Both vitamin B12 and

folate are needed to make thymidine triphosphate.

TMP14 p. 443

10. B) This patient has polycythemia vera: increased RBCs, WBCs,

and platelets. Normal arterial oxygen saturation shows that there

is no reason for a compensatory polycythemia. His increased

hematocrit also increases the viscosity of the blood, resulting in

increased afterload for the heart, which is probably the reason for

his chest pain. Thus, phlebotomy (bleeding) is needed to decrease

his elevated blood cell count and blood viscosity.

TMP14 p. 447

11. C) Secondary polycythemia has developed because of exposure

to low oxygen levels. She will have an increased hematocrit level

and RBC count but a normal WBC count. The cells are normal, so

the MCV will be normal.

TMP14 p. 447

12. B) This African American man has sickle cell anemia, as

demonstrated by his decreased hemoglobin concentration,

moderately elevated reticulocyte count, and history of painful

crises. His elevated WBC count suggests a response to stress

hormones, with recruitment of mature leukocytes into the

circulation. Increased oxygen demand from walking in hilly

terrain likely produced tissue ischemia with resultant sickling of

his RBCs.

TMP14 p. 446

13. C) EPO levels increase in response to decreased arterial oxygen

level, with the maximum EPO production occurring within 24

hours. It takes 3 days for new reticulocytes to appear in the

circulation, and after a total of 5 days from the beginning of

hypoxemia, these reticulocytes will be circulating as mature

erythrocytes. Because it takes 1 to 2 days for a reticulocyte to

become an erythrocyte, the correct answer is 3 days until the

person has an increased number of reticulocytes.

TMP14 pp. 441–442

14. C) The blood cell count shows that the patient is anemic. Her

bone marrow is functioning and she has a normal platelet count,

but she is generating a decreased number of abnormal RBCs. The

microcytic (small), hypochromic (decreased intracellular

hemoglobin) RBCs are a classic finding of iron-deficiency anemia,

which suggests that blood loss has been sufficiently sustained to

lead to iron depletion. A decrease in her blood pressure on

standing (postural hypotension) suggests decreased intravascular

volume, indicating that there may also be a component of acute

blood loss. If she had renal failure, she would be anemic with

normal RBCs. People with sickle cell anemia have misshapen

RBCs. Megaloblastic anemia is characterized by macrocytic (large)

RBCs.

TMP14 p. 446

15. C) Macrophages become activated and enlarged at sites of

inflammation and can ingest up to 100 bacteria per macrophage.

They can extrude digested material and remain viable and active

for many months. Basophils are not phagocytic, and eosinophils

are weak phagocytes. Neutrophils respond rapidly to infection or

inflammation and ingest from 3 to 20 bacteria or other particles

before dying.

TMP14 pp. 451–452, 456–457

16. D) Kupffer cells are tissue-specific macrophages that reside in the

hepatic sinusoids.

TMP14 p. 453

17. A) Eosinophils normally constitute about 2% of the total WBC

count, but they are produced in large numbers in people with

parasitic infections.

TMP14 p. 456

18. B) The majority of WBCs are stored in the bone marrow, waiting

for an increased level of cytokines or stress hormones (e.g.,

epinephrine, corticosteroids) to stimulate their release into the

circulation. Trauma to bone can result in a release of WBCs into

the circulation. This increase in WBC count is not primarily due to

any inflammatory response but instead is aributed to mechanical

trauma and associated stress responses.

TMP14 p. 455

19. B) The WBC count of 42,000 is higher than the range usually seen

as a response to infection and suggests leukemia. The patient’s

florid clinical presentation suggests an acute process, and findings

of a normal CBC 3 months previously confirm that this patient has

an acute leukemia. Granulocytes are myeloid cells, and the fact

that they are in the circulation while still being immature is wholly

compatible with leukemia. Thus, the patient has acute myelocytic

(also referred to as “myelogenous” or “myeloid”) leukemia.

TMP14 p. 457

20. B) Cross-linking of IgE receptors on the surface of basophils and

mast cells results in their degranulation, releasing an array of

inflammatory mediators.

TMP14 p. 457

21. B) The white blood cell count has been elevated for more than a

year and the cells that are in excess are mature-appearing cells that

clearly are lymphocytes. Thus, this is chronic lymphocytic

leukemia (CLL). Patients with CLL frequently remain

asymptomatic for long periods, often many years.

TMP14 p. 457

22. C) Transmigration of WBCs occurs through parts of the

vasculature that have very thin walls and minimal vascular

smooth muscle layers. This includes capillaries and venules.

TMP14 p. 451

23. C) Bacterial killing in cutaneous infections is largely mediated by

neutrophils, which produce bactericidal agents, including reactive

oxygen species. For the acquired immune response to function

normally, T and B lymphocytes and plasma cells, along with

macrophages, are needed.

TMP14 pp. 452, 454–455

24. B) Macrophages follow the initial influx of neutrophils into an

inflammatory site. Whereas neutrophils ingest a modest number

of bacteria per cell before dying, macrophages persist at the site,

ingesting and digesting infectious organisms and necrotic material

and producing cytokines that direct tissue remodeling by

fibroblasts and other cell types. Dendritic cells are resident

antigen-presenting cells, whereas eosinophils are weakly

phagocytic cells whose products (e.g., major basic protein) can kill

parasites without the eosinophils ingesting them.

TMP14 p. 455

25. D) The presence of normal skin test reactivity, which is T cell

mediated, indicates normal function of macrophages and other

antigen-presenting cells, helper T cells, and cytotoxic T cells. This

information, and the reduction in antibody production, localizes

the defect to the B-cell lineage.

TMP14 pp. 462, 467–468

26. D) Patients with HIV have specific loss of T-helper cells, resulting

in a loss of T-cell help for both antibody production and activation

and proliferation of cytotoxic T cells. Assuming that mast cells can

be appropriately stimulated (i.e., bear sufficient residual surfacebound IgE and are exposed to relevant antigen), their processes

for degranulation are intact.

TMP14 p. 467

27. B) Phagocytosis of bacteria is enhanced by the presence on their

surfaces of both immunoglobulin and products of the complement

cascade, which in turn bind to surface receptors on phagocytes.

This “tagging” of bacteria and other particles for enhanced

phagocytosis is called opsonization.

TMP14 p. 465

28. B) Presentation of a foreign antigen bound to MHC I on an

infected cell results in activation of the cytotoxic T cells to kill the

infected cell. Presentation of an antigen by MHC II on

macrophages activates helper T cells, which promote antibody

production and support proliferation of both helper and cytotoxic

T cells.

TMP14 p. 466

29. D) Variable and hypervariable regions are present at the ends of

both the heavy and light chains of immunoglobulin, allowing both

chains to participate in antigen binding.

TMP14 pp. 463–464

30. E) Fluid leaks into the tissue because of an increase in capillary

permeability.

TMP14 pp. 465–466

31. D) Only immunoglobulins bind intact antigen. T-cell receptors

and MHC class I and II molecules bind processed peptides but do

not bind intact antigen.

TMP14 pp. 464, 466

32. C) Infected labial cells process viral antigens and present them,

bound to MHC class I molecules. Viral antigens in association

with MHC class I molecules are recognized by antigen-specific

cytotoxic T cells, which then destroy the virally infected cells,

producing a lip ulcer.

TMP14 pp. 466–468

33. B) IL-2 is secreted by helper T cells when the T cells are activated

by specific antigens. IL-2 plays an important role in the

proliferation of helper, cytotoxic, and suppressor T cells.

TMP14 p. 467

34. D) Helper T cells are activated by the presentation of processed

antigens by MHC class II molecules on the surface of antigenpresenting cells (primarily macrophages and dendritic cells).

p g p y p g

Helper T cells activate B cells to form antibodies, but B cells are

not required for activation of helper T cells. Helper T cells help to

enhance phagocytosis by macrophages but they do not have the

ability to phagocytize bacteria themselves.

TMP14 pp. 466–467

35. C) Cytotoxic cells act on infected cells when the cells have the

appropriate antigen located on their surfaces. Cytotoxic T cells are

stimulated by lymphokines generated by activation of helper T

cells. Cytotoxic T cells destroy an infected cell by expressing

proteins that punch large holes in the membrane of the infected

cells. There is no interaction between cytotoxic T cells and normal

B cells.

TMP14 pp. 467–468

36. A) Because the person has previously demonstrated allergic

reactions, the initial cutaneous response reflects the binding of

antigen by specific IgE on the surfaces of basophils and mast cells,

resulting in the release of an array of inflammatory mediators. In

addition, activation of the complement system results in the

production of C3a and C4a, which also activate basophils and

mast cells. The influx of neutrophils activation of T-helper cells

and the accumulation of antigen-specific T lymphocytes would

take some time.

TMP14 pp. 469–470

37. D) Activation of the complement system results in a series of

actions, including opsonization and phagocytosis by neutrophils,

lysis of bacteria, agglutination of organisms, activation of

basophils and mast cells, and chemotaxis. Fragment C5a of the

complement system causes chemotaxis of neutrophils and

macrophages.

TMP14 p. 465

38. D) HDN occurs when an Rh-negative mother gives birth to a

second or subsequent Rh-positive child. Therefore, the father must

be Rh positive. The mother becomes sensitized to the Rh antigens

during the events surrounding the birth of an Rh-positive child.

HDN in future pregnancies can be prevented by treating the

mother with antibodies against Rh antigen around the birth of

each Rh-positive child. This treatment destroys fetal RBCs in the

mother and prevents the mother from being sensitized to the Rh

antigen. A transfusion of the child after birth does not prevent

HDN in future pregnancies because the mother has already been

exposed to the Rh antigen during the birth process.

TMP14 pp. 473–474

39. B) The mother could be heterozygous for blood types A and O

and thus could have a type O child. Rh positivity could have been

determined by an allele from the actual father. The man in this

couple is type AB and thus could not have contributed a blood

type O allele (persons who are type O must receive an O allele

from each parent).

TMP14 p. 472

40. D) Each parent needs only a single allele for either the A or B

antigen or the Rh(D) antigen to express these antigens on their

blood cells and other cell types. Thus, if each parent also carries an

allele for blood type O, as well as a null allele for the Rh(D)

antigen, then the child can be homozygous for the recessive O

allele and the Rh(D)-negative allele.

TMP14 pp. 472–473

41. D) The appropriate treatment is repetitive removal of Rh-positive

blood, replacing it with Rh-negative blood (an exchange of about

400 ml over 90 minutes). This treatment may be performed several

times over a few weeks. Maternal antibodies disappear over 1 to 2

months, so the newborn’s endogenous Rh-positive cells cease to be

a target. Exchange transfusions can actually be initiated in utero

when there is evidence of an active immune reaction against the

fetus’s blood cells.

TMP14 p. 474

42. C) Allograft rejection occurs primarily through the actions of

cytotoxic T cells. T-helper cells promote this reaction but are not

the effector cells. Both macrophages and dendritic cells may

present antigen that promotes the immune response, but the key

effector cells are cytotoxic T cells.

TMP14 pp. 475–476

43. B) Type A blood has A antigen on the red blood cell surface and

serum antibodies to the B antigen. Type B blood has B surface

antigens and anti-A antibodies. Therefore, transfusing type A

blood into a person with type B blood causes the A antibodies in

the type B recipient to react with the donor blood. Transfusion of

Rh-negative blood into an Rh-positive person with the same ABO

type will not result in any reaction.

TMP14 pp. 472–473

44. C) Isografts and autografts are derived from genetically identical

donors or from the recipient’s own tissues, respectively. Therefore,

they can be expected to be readily accepted because they present

no foreign antigens to the recipient. Allografts and xenografts do

result in the presentation of foreign antigens. Any HLA mismatch

indicates that the transplanted tissue or organ is an allograft that

carries known foreign antigens.

TMP14 p. 475

45. C) A person who has never been exposed to the Rh antigen

before would not have anti-Rh antibodies and thus would not

have an immediate transfusion reaction. Rather, such antibodies

would develop over a few weeks, resulting in gradual

opsonization and phagocytosis of the transfused cells with

moderate, intracellular hemolysis.

TMP14 p. 473

46. D) Type A, Rh-positive cells express both the A and Rh antigens.

Persons with type O blood develop antibodies to type A antigen

through dietary and environmental exposure. Thus, transfusion of

type A blood into a type O recipient results in an immediate

transfusion reaction. Reaction to Rh antigens requires prior

exposure through an Rh-incompatible blood transfusion.

Reactions occur between the recipient’s antibodies and donor

antigens as shown in the following table.

TMP14 pp. 472–473

Donor Donor Antigen Recipient Recipient Antibody Reaction

O negative None AB positive None None

A positive A, Rh A negative B None

AB positive A, B, Rh AB positive None None

A positive A, Rh O positive A, B A (antigen) and A (antibody)

47. D) Unmatched donor HLA antigens of both classes are

recognized as foreign by recipient T cells. In addition, blood group

(ABO) antigens are expressed on the cells of solid organs and can

lead to strong, immediate organ rejection.

TMP14 p. 475

48. D) HDN occurs when the mother is Rh negative and the father is

Rh positive, resulting in an Rh-positive child. Because the child is

O, Rh negative, HDN would not be expected to develop.

TMP14 p. 473

49. C) Antibiotic treatment for pneumonia can kill flora in the

gastrointestinal tract that are critical for the production of vitamin

K. Production of several active cloing factors (prothrombin and

factors VII, IX, and X) has been suppressed in this patient by

warfarin inhibition of VKOR c1, which normally chemically

reduces vitamin K so that it can activate the listed cloing factors.

Further quantitative reduction of vitamin K by the death of critical

gut flora has produced excessive anticoagulation and resulted in

bleeding in this patient. Fresh-frozen plasma is infused to provide

active cloing factors immediately, and vitamin K is provided to

promote endogenous production of active cloing factors. Both

are needed in the seing of acute bleeding.

TMP14 pp. 484, 486–487

50. C) Inheritance of a bleeding disorder from the maternal

grandfather is classic for an X-linked disorder—the grandfather

transmits the affected X allele to his daughter, who transmits it to

her son. Hemophilia A and B, resulting in deficiency of cloing

factors VIII and IX, respectively, are both X-linked disorders that

present in the manner described for this 2-year-old boy.

TMP14 p. 485

51. B) In this patient with chronic, severe viral hepatitis, liver

damage has led to hepatic insufficiency, so that he is no longer

able to manufacture sufficient quantities of cloing factors. These

must be provided in this emergent situation by treatment with

fresh-frozen plasma. Fibrinogen production generally remains

adequate in patients with liver failure.

TMP14 p. 484

52. B) Warfarin functions by interfering with the production of active

cloing factors. It is not effective in inhibiting cloing factors

in vitro.

TMP14 pp. 486–487

53. A) Hemophilia A is caused by a deficiency of cloing factor VIII,

which functions in the intrinsic cloing pathway only.

TMP14 pp. 482, 485

54. D) In sepsis and particularly septic shock, bacteria and bacterial

toxins such as endotoxin, plus the release of large amounts of

tissue factor from traumatized or dying tissue, cause wide spread

activation of the cloing system. This produces innumerable small

clots that occlude small vessels and exacerbate tissue damage.

Paradoxically, this consumes cloing factors to an extent that the

clinical result is bleeding from multiple sites. Even if a toxin that

could inhibit platelet function had been ingested, this usually is

manifested by cutaneous petechiae (small, punctate hemorrhages)

and bruising rather than widespread bleeding.

TMP14 p. 486

55. A) The extrinsic pathway begins with the release of tissue

thromboplastin in response to vascular injury or contact between

traumatized extravascular tissue and blood. Tissue

thromboplastin is composed of phospholipids from the

membranes of the damaged tissues.

TMP14 p. 481

56. B) Severe diarrhea can lead to fat malabsorption. Because vitamin

K is a fat-soluble vitamin, this can result in vitamin K deficiency.

Vitamin K is required for the production of active cloing factors.

Thus, vitamin K deficiency can cause defective blood coagulation

and excessive bleeding.

TMP14 p. 484

57. C) Bleeding time, measured by determining how long it takes for

bleeding to stop from a small, standardized cut, reflects platelet

aggregation and activation and the formation of platelet plugs in

the small vessels that have been damaged. Blood coagulation

involving the activation of thrombin and the accumulation of

fibrin fibrils takes significantly longer.

TMP14 p. 487

58. D) Treating a massive pulmonary embolus is an emergency that

requires lysis of an existing clot. This can be accomplished by

infusing tissue plasminogen activator, which converts

plasminogen to plasmin, which then actively lyses the clot. In this

seing measures that inhibit clot formation are less relevant

because a life-threatening clot already exists.

TMP14 p. 486

59. A) The primary function of heparin is to bind to and activate

antithrombin III.

TMP14 p. 484

60. B) A familial disorder that occurs in males only suggests an Xlinked disorder because males have a single X chromosome, and if

it is defective, then that defect is expressed clinically (rather than

being masked by a normally functioning X chromosome, as occurs

in females). Males receive their X chromosome from their mothers

and their Y chromosome from their fathers. The most common Xlinked bleeding disorders are hemophilia A and hemophilia B,

which are highly penetrant.

TMP14 p. 485


Unit VII: Respiration

1. A healthy 25-year-old medical student participates in a 10-km

charity run for the American Heart Association. Which muscles

does the student use (contract) during expiration?

A) Diaphragm and external intercostals

B) Diaphragm and internal intercostals

C) Diaphragm only

D) Internal intercostals and abdominal recti

E) Scaleni

F) Sternocleidomastoid muscles

2. Several students are trying to see who can generate the highest

expiratory flow. Which muscle is most effective at producing a

maximal effort?

A) Diaphragm

B) Internal intercostals

C) External intercostals

D) Rectus abdominis

E) Sternocleidomastoid

3. The above figure shows three different compliance curves (S, T, and

U) for isolated lungs subjected to various transpulmonary

pressures. Which of the following best describes the relative

compliances for the three curves?

A) S < T < U

B) S < T > U

C) S − T − U

D) S > T < U

E) S > T > U

Questions 4 and 5

Use the figure below to answer Questions 4 and 5.

4. Assuming a respiratory rate of 12 breaths/min, calculate the minute

ventilation.

A) 1 l/min

B) 2 l/min

C) 4 l/min

D) 5 l/min

E) 6 l/min

5. A 22-year-old woman inhales as much air as possible and exhales

as much air as she can, producing the spirogram shown in the

figure. A residual volume of 1.0 liter is determined using the

helium dilution technique. What is her FRC (in liters)?

A) 2.0

B) 2.5

C) 3.0

D) 3.5

E) 4.0

F) 5.0

6. A 22-year-old woman has a pulmonary compliance of 0.2 l/cm H2O

and a pleural pressure of −4 cm H2O. What is the pleural pressure

(in cm H2O) when the woman inhales 1.0 l of air?

A) −6

B) −7

C) −8

D) −9

E) −10

7. A preterm infant has a surfactant deficiency. Without surfactant,

many of the alveoli collapse at the end of each expiration, which in

turn leads to pulmonary failure. Which set of changes is present in

the preterm infant compared with a normal infant?

Alveolar Surface Tension Pulmonary Compliance

A) Decreased Decreased

B) Decreased Increased

C) Decreased No change

D)

Increased Decreased

E)

Increased Increased

F)

Increased No change

G) No change No change

8. A patient has a dead space of 150 ml, FRC of 3 liters, tidal volume

(VT) of 650 ml, expiratory reserve volume (ERV) of 1.5 l, total lung

capacity (TLC) of 8 l, and respiratory rate of 15 breaths/min. What

is the residual volume (RV)?

A) 500 ml

B) 1000 ml

C) 1500 ml

D) 2500 ml

E) 6500 ml

9. The various lung volumes and capacities include the total lung

capacity (TLC), vital capacity (VC), inspiratory capacity (IC), tidal

volume (VT), expiratory capacity (EC), expiratory reserve volume

(ERV), inspiratory reserve volume (IRV), functional residual

capacity (FRC), and residual volume (RV). Which of the following

lung volumes and capacities can be measured using direct

spirometry without additional methods?

TLC VC IC VT EC ERV IRV FRC RV

A) No No Yes No Yes No Yes No No

B) No Yes Yes Yes Yes Yes Yes No No

C) No Yes Yes Yes Yes Yes Yes Yes No

D) Yes Yes Yes Yes Yes Yes Yes No Yes

E) Yes Yes Yes Yes Yes Yes Yes Yes Yes

10. A 34-year-old man sustains a bullet wound to the chest that causes

a pneumothorax. What best describes the changes in lung volume

and thoracic volume in this man compared with normal?

Lung Volume Thoracic Volume

A) Decreased Decreased

B) Decreased Increased

C) Decreased No change

D)

Increased Decreased

E)

Increased Increased

F) No change Decreased

11. A healthy 10-year-old boy breathes quietly under resting

conditions. His tidal volume is 400 ml, and his ventilation

frequency is 12/min. Which of the following best describes the

ventilation of the upper, middle, and lower lung zones in this boy?

Upper Zone Middle Zone Lower Zone

A) Highest Lowest Intermediate

B) Highest Intermediate Lowest

C)

Intermediate Lowest Highest

D) Lowest Intermediate Highest

E) Same Same Same

12. An experiment is conducted in two persons (subjects T and V)

with identical VTs (1000 ml), dead space volumes (200 ml), and

ventilation frequencies (20 breaths/min). Subject T doubles his VT

and reduces his ventilation frequency by 50%. Subject V doubles

his ventilation frequency and reduces his VT by 50%. What best

describes the total ventilation (also called minute ventilation) and

VA of subjects T and V?

Total Ventilation VA

A) T < V T − V

B) T < V T > V

C) T − V T < V

D) T − V T − V

E) T − V T > V

F) T > V T < V

G) T > V T − V

13. A person with normal lungs has an oxygen (O2

) consumption of

750 ml O2

/min. The hemoglobin (Hb) concentration is 15 g/dl. The

mixed venous saturation is 25%. What is the cardiac output?

A) 2500 ml/min

B) 5000 ml/min

C) 7500 ml/min

D) 10,000 ml/min

E) 20,000 ml/min

14. A cardiac catheterization is performed in a healthy adult. The

blood sample withdrawn from the catheter shows 60% O2

saturation, and the pressure recording shows oscillations from a

maximum of 27 mm Hg to a minimum of 12 mm Hg. Where was

the catheter tip located?

A) Ductus arteriosus

B) Foramen ovale

C) Left atrium

D) Pulmonary artery

E) Right atrium

15. A 67-year-old man is admied emergently to the hospital because

of severe chest pain. A Swan-Ganz catheter is floated into the

pulmonary artery, the balloon is inflated, and the pulmonary

wedge pressure is measured. The pulmonary wedge pressure is

used clinically to monitor which pressure?

A) Left atrial pressure

B) Left ventricular pressure

C) Pulmonary artery diastolic pressure

D) Pulmonary artery systolic pressure

E) Pulmonary capillary pressure

16. Which diagram in the above figure best illustrates the pulmonary

vasculature when the cardiac output has increased to a maximum

extent?

A) A

B) B

C) C

D) D

E) E

17. A 19-year-old man sustains a full-thickness burn over 60% of his

body surface area. A systemic Pseudomonas aeruginosa infection

occurs, and severe pulmonary edema follows 7 days later. The

following data are collected from the patient: plasma colloid

osmotic pressure, 19 mm Hg; pulmonary capillary hydrostatic

pressure, 7 mm Hg; and interstitial fluid hydrostatic pressure, 1

mm Hg. Which set of changes has occurred in the lungs of this

patient as a result of the burn and subsequent infection?

Lymph Flow Plasma Colloid Osmotic Pressure Pulmonary Capillary Permeability

A) Decrease Decrease Decrease

B)

Increase Decrease Decrease

C)

Increase Decrease Increase

D)

Increase Increase Decrease

E)

Increase Increase Increase

18. Blood gas measurements are obtained in a resting patient who is

breathing room air. The patient has an arterial content of 19 ml

O2

/min with a PO 2

 of 95. The mixed venous O2

 content is 4 ml

O2

/100 ml blood. Which condition does the patient have?

A) An increase in physiological dead space

B) Pulmonary edema

C) A low Hb concentration

D) A low cardiac output

19. A normal male subject has the following initial conditions (in the

steady state):

Arterial PO 2

 = 92 mm Hg

Arterial O2

 saturation = 97%

Venous O2

 saturation = 20%

Venous PO 2

 = 30 mm Hg

Cardiac output = 5600 ml/min

O2

 consumption = 256 ml/min

Hb concentration = 12 g/dl

If you ignore the contribution of dissolved O2

 to the O2

 content, what

is the venous O2

 content?

A) 2.2 ml O2

/100 ml blood

B) 3.2 ml O2

/100 ml blood

C) 4 ml O2

/100 ml blood

D) 4.6 ml O2

/100 ml blood

E) 6.2 ml O2

/100 ml blood

F) 10.8 ml O2

/100 ml blood

G) 16 ml O2

/100 ml blood

20. A 30-year-old woman performs a Valsalva maneuver about 30

minutes after eating lunch. Which option best describes the

changes in pulmonary and systemic blood volumes that occur in

this woman?

Pulmonary Volume Systemic Volume

A) Decreases Decreases

B) Decreases Increases

C) Decreases No change

D)

Increases Decreases

E)

Increases Increases

F)

Increases No change

G) No change Decreases

H) No change Increases

I) No change No change

21. A child who is eating round candies approximately 1.5 cm in

diameter inhales one down his airway, blocking his left bronchiole.

Which of the following describes the changes that occur?

Left Lung Alveolar PCO 2 Left Lung Alveolar PO 2 Systemic Arterial PO 2

A) ↑ ↑ ↔

B) ↑ ↔ ↑

C) ↓ ↓ ↓

D) ↑ ↑ ↑

E) ↑ ↓ ↓

22. The forces governing the diffusion of a gas through a biological

membrane include the pressure difference across the membrane

(ΔP), the cross-sectional area of the membrane (A), the solubility of

the gas (S), the distance of diffusion (d), and the molecular weight

of the gas (MW). Which changes increase the diffusion of a gas

through a biological membrane?

ΔP A S D MW

A)

Increase Increase Increase Increase Increase

B)

Increase Increase Increase Increase Decrease

C)

Increase Decrease Increase Decrease Decrease

D)

Increase Increase Increase Decrease Increase

E)

Increase Increase Increase Decrease Decrease

23. A person’s normal VT is 400 ml with a dead space of 100 ml. The

respiratory rate is 12 breaths/min. The person undergoes

ventilation during surgery, and the VT is 700 with a rate of 12.

What is the approximate alveolar PCO 2

 for this person?

A) 10

B) 20

C) 30

D) 40

E) 45

24. A 45-year-old man at sea level has an inspired O2

 tension of 149

mm Hg, nitrogen tension of 563 mm Hg, and water vapor pressure

of 47 mm Hg. A small tumor pushes against a pulmonary blood

vessel, completely blocking the blood flow to a small group of

alveoli. What are the O2

 and carbon dioxide (CO2

) tensions of the

alveoli that are not perfused (in mm Hg)?

CO2 O2

A) 0 0

B) 0 149

C) 40 104

D) 47 149

E) 45 149
















guyton and hall physiology review
guyton and hall physiology
guyton and hall physiology review mcq
guyton and hall physiology short notes
guyton and hall physiology review latest edition
guyton and hall physiology review questions pdf
guyton and hall physiology mcqs
guyton and hall physiology review 4th edition
guyton and hall physiology lectures

guyton and hall physiology ppt
guyton and hall physiology anki
guyton and hall physiology anki deck
guyton and hall physiology audiobook
guyton and hall textbook of medical physiology apa citation
guyton and hall medical physiology 13th edition amazon
guyton and hall textbook of medical physiology south asian edition pdf download
guyton and hall textbook of medical physiology 13th edition amazon
guyton and hall textbook of medical physiology 14th edition amazon
guyton and hall physiology south asia edition pdf download
guyton and hall physiology south asia edition pdf
guyton and hall physiology book price
guyton and hall physiology book price in pakistan
guyton and hall physiology book
guyton and hall medical physiology book
guyton hall physiology question book
guyton and hall textbook of medical physiology e-book
physiology by guyton and hall
medical physiology by guyton and hall
guyton and hall textbook of medical physiology book buy
guyton and hall medical physiology review book
guyton and hall physiology chapters
guyton and hall physiology contents
guyton and hall physiology chapter 1
guyton and hall physiology citation
guyton and hall physiology table of contents
guyton and hall textbook of medical physiology citation
guyton and hall textbook of medical physiology chapters
guyton and hall textbook of medical physiology cite
guyton and hall textbook of medical physiology citation apa
guyton and hall textbook of medical physiology content
guyton and hall textbook of medical physiology deutsch
guyton and hall textbook of medical physiology 14th ed
guyton and hall textbook of medical physiology 12th ed citation
guyton and hall medical physiology ebook
guyton and hall human physiology
guyton and hall physiology kit
guyton and hall physiology lecture ppt
guyton and hall physiology latest
guyton and hall textbook of medical physiology latest international edition
guyton and hall physiology medical
guyton and hall medical physiology 14th
guyton and hall medical physiology notes
guyton and hall medical physiology review
guyton and hall textbook of medical physiology 14th edition citation
guyton and hall textbook of medical physiology 12th ed
guyton and hall physiology notes
guyton and hall textbook of medical physiology online
guyton and hall of medical physiology
guyton and hall textbook of medical physiology 14th
guyton and hall textbook of medical physiology 12th edition citation
guyton and hall physiology price
guyton and hall textbook of medical physiology price
guyton and hall textbook of medical physiology ppt
guyton and hall physiology questions
guyton and hall physiology quizlet
guyton and hall physiology review questions
guyton and hall textbook of medical physiology quizlet
guyton and hall textbook of medical physiology quiz
guyton and hall physiology review questions 4th edition pdf
guyton and hall physiology review questions 4th edition
guyton and hall textbook of medical physiology quora
guyton and hall physiology review 3rd edition
guyton and hall physiology review 2nd edition
guyton and hall physiology south asia edition
guyton and hall physiology summary
guyton and hall textbook of medical physiology summary
guyton and hall physiology third south asia edition pdf download
guyton and hall textbook of medical physiology south asian edition
guyton and hall textbook of medical physiology 2nd south asia edition pdf
guyton and hall physiology textbook
guyton and hall medical physiology textbook
guyton and hall physiology videos
guyton and hall textbook of medical physiology videos
guyton and hall textbook of medical physiology tiếng việt
guyton and hall physiology x ray
guyton and hall physiology xr
guyton and hall physiology youtube
guyton and hall physiology zotero
guyton and hall physiology zone
guyton and hall physiology zuid korea
can guyton and hall physiology exam
can guyton and hall physiology edition
can guyton and hall physiology essay
can guyton and hall physiology enzymes
can guyton and hall physiology journal
can guyton and hall physiology jobs
can guyton and hall physiology journal impact factor
can guyton and hall physiology kit
can guyton and hall physiology keys
can guyton and hall physiology of arts
can guyton and hall physiology of medicine
guyton and hall renal physiology
guyton and hall respiratory physiology
can guyton and hall physiology wikipedia
can guyton and hall physiology wiki
can guyton and hall physiology workshop
can guyton and hall physiology x ray
can guyton and hall physiology xr
can guyton and hall physiology youtube
can guyton and hall physiology yacht
can guyton and hall physiology zone
can guyton and hall physiology zurich
can guyton and hall physiology zotero
how guyton and hall physiology essay
how guyton and hall physiology exam
how guyton and hall physiology edition
how guyton and hall physiology endnote
how guyton and hall physiology journal
how guyton and hall physiology journal impact factor
how guyton and hall physiology jobs
how guyton and hall physiology kit
how guyton and hall physiology key
how guyton and hall physiology keys
how guyton and hall physiology of arts
how guyton and hall physiology of medicine
how guyton and hall physiology wikipedia
how guyton and hall physiology wiki
how guyton and hall physiology workshop
how guyton and hall physiology x ray
how guyton and hall physiology youtube
how guyton and hall physiology yacht
how guyton and hall physiology zone
how guyton and hall physiology zurich
how guyton and hall physiology zotero
how guyton and hall physiology zones
introduction to physiology
human physiology
الفيزيولوجيا الطبية
محاضرات physiology
which guyton and hall physiology edition
which guyton and hall physiology exam
which guyton and hall physiology essay
which guyton and hall physiology journal
which guyton and hall physiology jobs
which guyton and hall physiology journal impact factor
which guyton and hall physiology kit
which guyton and hall physiology key
which guyton and hall physiology keys
which guyton and hall physiology of medicine
which guyton and hall physiology of arts

F, Female; M, Male.


No comments:

Post a Comment

اكتب تعليق حول الموضوع

mcq general

 

Search This Blog